SlideShare a Scribd company logo
1 of 62
Download to read offline
CHUYEÂN ÑEÀ 1
TOÏA ÑOÄ PHAÚNG
Trong caùc baøi toaùn veà toïa ñoä trong maët phaúng thöôøng gaëp caùc yeâu caàu nhö tìm toïa ñoä
moät ñieåm, moät vectô, tính ñoä daøi moät ñoaïn thaúng, soá ño goùc giöõa hai vectô, quan heä cuøng
phöông hoaëc vuoâng goùc giöõa hai vectô, 3 ñieåm thaúng haøng.
Ta vaän duïng caùc kieán thöùc cô baûn sau ñaây:
Cho a = ( , = ta coù:b ))1 2a , a ( 1 2b , b
a = b ⇔
1
2
1
2
a = b
a = b
⎧
⎨
⎩
a + = ( , )b 1 1a + b 2 2a + b
a – = ( , )b 1 1a - b 2 2a - b
ka = (k , k ) (k1a 2a ∈ R)
α + = ( +a β b α 1a β 1b , α 2a + β 2b )
a . = +b 1a 1b 2a 2b
. Vôùi caùc quan heä veà ñoä daøi ta coù:
a = ( , )1a 2a ⇒ a = 2 2
1 2a + a
( )
( )
A A
B B
A x , y
B x , y
⎧⎪
⎨
⎪⎩
⇒ AB = ( – , – )Bx Ax By Ay
vaø AB = ( ) ( )
2 2
B A B Ax - x y - y+
. Vôùi quan heä cuøng phöông hoaëc vuoâng goùc ta coù:
a + = 0⊥ b ⇔ 1a 1b 2a 2b
cuøng phöônga b ⇔ sin(a, b) = 0 ⇔ – = 01a 2b 2a 1b
⇔ 1
1
a
b
= 2
2
a
b
( ,1b 2b ≠ 0)
A, B, C thaúng haøng ⇔ AB cuøng phöông AC
⇔
B A B A
C A C A
x - x y - y
x - x y - y
= 0
. Vôùi vieäc tìm goùc cuûa hai vectô ta coù:
- Goùc hình hoïc taïo bôûi hai vectô a, b ñöôïc suy töø coâng thöùc:
cos(a, b ) = 1 1 22a b + a b
a . b
(1)
- Soá ño goùc ñònh höôùng cuûa hai vectô a, b ngoaøi (1) coøn ñöôïc suy theâm töø moät trong
hai coâng thöùc:
sin(a, b) = 1 2 12a b - a b
a . b
tg(a, b) = 1 2 1
1 1 2
2
2
a b - a b
a b + a b
Ngoaøi ra trong caùc baøi toaùn veà toïa ñoä phaúng ta coù theå aùp duïng caùc keát quaû sau ñaây:
. M( , ) laø trung ñieåm cuûa ñoaïn thaúng ABMx My
⇔
2
2
A B
M
A B
M
x + x
x =
y + y
y =
⎧
⎪⎪
⎨
⎪
⎪⎩
. G( , ) laø troïng taâm cuûaGx Gy Δ ABC
⇔
3
3
⎧
⎪⎪
⎨
⎪
⎪⎩
A B
G
A B
G
x + x + x
x =
y + y + y
y =
C
C
. I( , ) vaø J( , ) laø chaân ñöôøng phaân giaùc trong vaø ngoaøi cuûa goùc A trong
ABC thì:
Ix Iy Jx Jy
Δ
IB
IC
= −
JB
JC
= −
AB
AC
. Vôùi A( , ), B( , ), C( , ) thì dieän tích tam giaùc ABC laø:Ax Ay Bx By Cx Cy
S =
1
2
Δ vôùi Δ =
B A B A
C A C A
x - x y - y
x - x y - y
Ví duï 1:
Trong maët phaúng Oxy cho ba ñieåm A(2, –1), B(0, 3), C(4, 2).
a) Tìm toïa ñoä ñieåm D ñoái xöùng vôùi A qua B.
b) Tìm toïa ñoä ñieåm M ñeå 2 + 3AM BM - 4CM = 0
c) Tìm toïa ñoä ñieåm E ñeå ABCE laø hình thang coù moät caïnh ñaùy laø AB vaø E naèm treân
Ox.
d) Tìm toïa ñoä tröïc taâm H, troïng taâm G vaø taâm I ñöôøng troøn ngoaïi tieáp ABC.Δ
e) Chöùng toû H, G, I thaúng haøng.
Giaûi
a) D laø ñieåm ñoái xöùng cuûa A qua B
B laø trung ñieåm cuûa AD⇔
⇔
A D
B
A D
B
x + x
x =
2
y + y
y =
2
⎧
⎪⎪
⎨
⎪
⎪⎩
hay D(–2, 7)⇔
( )
( )
− −⎧⎪
⎨
−⎪⎩
D B A
D B A
x = 2x x = 2 0 2 = 2
y = 2y y = 2 3 + 1 = 7
−
b) Ta coù: 2 + 3 BM – 4CMAM = 0 = ( 0, 0 )
⇔
( ) ( ) ( )
( ) ( ) ( )
− − − −⎧⎪
⎨
− − −⎪⎩
M M M
M M M
2 x 2 + 3 x 0 4 x 4 = 0
2 y + 1 + 3 y 3 4 y 2 = 0
⇔ hay M(–12, –1)
−⎧
⎨
−⎩
M
M
x = 12
y = 1
c) ABCE laø hình thang coù ñaùy AB vaø E naèm treân Ox.
⇔
Ey = 0
CE
⎧⎪
⎨
ΑΒ⎪⎩ //
⇔
E
E E
y = 0
x - 4 y - 2
=
0 - 2 3 + 1
⎧
⎪
⎨
⎪⎩
⇔ hay E(5, 0)
E
E
y = 0
x = 5
⎧
⎨
⎩
d) H laø tröïc taâm cuûa ABCΔ
⇔
AH BC
BH AC
⊥⎧
⎨
⊥⎩
⇔
AH.BC = 0
BH.AC = 0
⎧⎪
⎨
⎪⎩
⇔
( )( ) ( )( )
( )( ) ( )( )
4 1 2
4 2 3 2 1 0
− − + + − =⎧⎪
⎨
− − + − + =⎪⎩
H H
H H
x 2 0 y
x 0 y
3 0
2 3 9
H H
H H
x y
x y
− − =⎧
⎨
+ − =⎩
⇔
4 9 0
0
⇔
18
7
9
7
H
H
x
y
⎧
=⎪⎪
⎨
⎪ =
⎪⎩
hay H
18
7
9
,
7
⎛ ⎞
⎜ ⎟
⎝ ⎠
G laø troïng taâm ABC ta coù:Δ
2 0 4
2
3 3
1 3 2 4
3 3
A B C
G
A B C
G
x x x
x
y y y
y
+ + + +⎧
= =⎪⎪
⎨
+ + − + +⎪ = =
⎪⎩ 3
=
=
hay G
4
2
3
,
⎛ ⎞
⎜ ⎟
⎝ ⎠
+ I laø taâm ñöôøng troøn ngoaïi tieáp Δ ABC
⇔ IA = IB = IC ⇔
2 2
2 2
IA IB
IA IC
⎧ =⎪
⎨
=⎪⎩
⇔
( ) ( ) ( ) (
( ) ( ) ( ) (
2 2 2
2 2 2
2 1 0 3
2 1 4 2
I I I
I I I
x y x
x y x
⎧ − + − − = − + −⎪
⎨
− + − − = − + −⎪⎩
)
)
2
2
I
I
y
y
0
0
⇔
4 8 4
4 6 15
I I
I I
x y
x y
− + − =⎧
⎨
+ − =⎩
⇔
24 12
14 7
19
14
I
I
x
y
⎧
= =⎪⎪
⎨
⎪ =
⎪⎩
hay I
12 19
7 14
,
⎛ ⎞
⎜ ⎟
⎝ ⎠
e) Ta coù : =HG
4 1
7 21
,
⎛ ⎞
−⎜ ⎟
⎝ ⎠
vaø HI =
6 1
7 14
,
⎛ ⎞
−⎜ ⎟
⎝ ⎠
⇒
4
7
6
7
−
−
=
1
21
1
14
=
2
3
⇒ cuøng phöông vôùiHG HI
⇒ H, I, G thaúng haøng.
Ví duï 2:
Trong maët phaúng Oxy cho A(2, 2 3 ), B(1, 3 3 ), C (-1, 3 ) . Tính
cos ( AO, AB) vaø dieän tích tam giaùc ABC.
Giaûi
Ta coù: AO = (–2, –2 3 ), AB = (–1, 3 ) = ( a1;a2 )
cos( AO, AB) =
2 6
4 12 1 3.
−
+ +
=
1
2
−
AC = (–3, – 3 ) = = ( b1; b2 )
⇒ 1 2 2 1
1
2
= −ABCS a b a b =
1
1 3 3 3
2
− − − −( )( ) ( ) = 2 3
* * *
CHUYEÂN ÑEÀ 2
ÑÖÔØNG VAØ PHÖÔNG TRÌNH ÑÖÔØNG
Caùc baøi toaùn veà phaàn ñöôøng vaø phöông trình ñöôøng thöôøng yeâu caàu xaùc ñònh quyõ tích
caùc ñieåm trong maët phaúng toïa ñoä theo nhöõng ñieàu kieän cho tröôùc, quyõ tích naøy laø moät ñöôøng
maø ta phaûi tìm phöông trình cuûa noù döïa vaøo ñònh nghóa:
F(x, y) = 0 laø phöông trình cuûa ñöôøng (L) neáu ta coù :
M( , ) ∈ (L) F( , ) = 0Mx My ⇔ Mx My
Neáu M ∈ (L) vaø M coù toïa ñoä phuï thuoäc tham soá t:
( )
( )
x f t
y g t
=⎧⎪
⎨
=⎪⎩
(t ∈ R)
thì ñoù laø phöông trình tham soá cuûa ñöôøng (L).
Töø phöông trình tham soá, ta khöû t thì coù theå trôû veà daïng
F(x, y) = 0
Löu yù vieäc giôùi haïn cuûa quyõ tích tuyø theo caùc ñieàu kieän ñaõ cho trong ñaàu baøi.
Ví du1:
Trong maët phaúng Oxy cho A(2, 1), B(–3, 2). Tìm quyõ tích ñieåm M ñeå
( MA + MB ) AB = 1
Giaûi
Goïi (L) laø quyõ tích phaûi tìm.
M( , ) ∈ (L)Mx My ⇔ ( MA + MB ) AB = 1
[ (2 – ) + (–3 – ) ] (–3 – 2) + (1 – + 2 – ) (2 – 1) = 1⇔ Mx Mx My My
5 + 10 + 3 – 2 = 1⇔ Mx My
10 – 2 + 7 = 0⇔ Mx My
M( , ) coù toïa ñoä thoûa phöông trình⇔ Mx My
F(x, y) = 10x – 2y + 7 = 0
Vaäy quyõ tích phaûi tìm laø ñöôøng thaúng (L) coù phöông trình
10x – 2y + 7 = 0.
1
Ví duï 2:
Laäp phöông trình quyõ tích taâm cuûa nhöõng ñöôøng troøn tieáp xuùc vôùi truïc Ox vaø ñi qua
ñieåm A(1, 2).
Giaûi
Goïi (L) laø quyõ tích nhöõng taâm ñöôøng troøn tieáp xuùc vôùi truïc Ox vaø ñi qua ñieåm A(1, 2).
I( , ) ∈ (L) I laø taâm ñöôøng troøn qua A(1, 2) vaø tieáp xuùc vôùi Ox taïi MIx Iy ⇔
⇔
IM Ox taïi M
IM = IA
⊥⎧
⎨
⎩
⇔
( ) ( ) ( ) ( )
2 2 2
0 0M I M
M I M I A I A I
x x vaø y
x x y y x x y y
− = =⎧⎪
⎨
− + − = − + −⎪⎩
2
– 2 – 4 + 5 = 0⇔ 2
Ix Ix Iy
I( , ) coù toïa ñoä thoûa phöông trình⇔ Ix Iy
F(x, y) = x2
– 2x – 4y + 5 = 0
Ñoù laø phöông trình cuûa quyõ tích phaûi tìm (Parabol).
* * *
2
CHUYEÂN ÑEÀ 3
ÑÖÔØNG THAÚNG
I. PHÖÔNG TRÌNH ÑÖÔØNG THAÚNG
Trong maët phaúng toïa ñoä Oxy, muoán vieát phöông trình moät ñöôøng thaúng ta caàn
phaûi bieát:
( )Δ
1) ( qua ñieåm M0(x0, y0) vaø coù vectô chæ phöông a)Δ = (a1, a2) seõ coù:
. Phöông trình tham soá : (t
0
0 2
x x ta
y y ta
= +⎧
⎨
= +⎩
1
∈ R)
. Phöông trình chính taéc : 0
1
x x
a
−
= 0
2
y y
a
−
(a1, a2 ≠ 0)
Töø phöông trình chính taéc ta coù theå ñoåi thaønh daïng phöông trình toång quaùt :
Ax + By + C = 0 (A2
+ B2
> 0)
2) ( qua ñieåm M0(x0, y0) vaø coù 1 phaùp veùctô laø (a,b) coù phöông trình : a(x –
x0) + b(y – y0) = 0
)Δ
3) i) Phöông trình ñöôøng thaúng trong maët phaúng coù daïng
Ax + By + C = 0 vôùi A2
+ B2
> 0 (1)
ii) Phöông trình ñöôøng thaúng trong maët phaúng coù daïng
x = x0 hoaëc y = kx + m (2).
Ta deã daøng thaáy (1) vaø (2) laø töông ñöông.
+ (2) ⇔ kx –y + m = 0 ⇒ (2 ) thoûa (1) vôùi A = k, B = - 1 , C = m.
+ Neáu B = 0 ⇒ = −
C
x
A
, coù daïng x = x0 vôùi x0 = −
C
A
. Neáu B ≠ 0 ⇒ = − −
A C
y x
B B
, coù
daïng y = kx + m.
3) ( qua hai ñieåm A(xA, yA), B(xB, yB) coù phöông trình :)Δ
A
B A
x x
x x
−
−
= A
B A
y y
y y
−
−
neáu 0− − ≠B A B A(x x )( y y )
1
Neáu ( qua A(a, 0) ∈ Ox vaø B(0, b))Δ ∈ Oy vôùi a.b ≠ 0; ta noùi ( )Δ coù ñoaïn chaén a, b
vôùi phöông trình:
x
a
+
y
b
= 1
* Ghi chuù:
Neáu ñeà baøi toaùn yeâu caàu ta vieát phöông trình cuûa ñöôøng thaúng, thoâng thöôøng ta neân
vieát phöông trình ôû daïng toång quaùt vaø löu yù :
( )Δ : Ax + By + C = 0 thì ( )Δ coù :
. moät phaùp vectô = (A, B)n
. moät vectô chæ phöông a = (–B, A)
. heä soá goùc k = tg( , ) =Ox Δ
A
B
−
. ( ) (′Δ // ( )Δ ⇒ )′Δ : Ax + By + C0 = 0
. ( ) (′Δ ⊥ ( )Δ ⇒ )′Δ : Bx – Ay + C0 = 0
Ta tìm ñöôïc C0 neáu bieát theâm moät ñieåm naèm treân ( )′Δ .
Ngoaøi ra khi vieát phöông trình cuûa moät ñöôøng thaúng ( )Δ theo heä soá goùc k, baøi toaùn coù
theå bò thieáu nghieäm do tröôøng hôïp ( )Δ ⊥ x′x (heä soá goùc k khoâng toàn taïi), do ñoù ta phaûi xeùt
theâm tröôøng hôïp coù phöông trình x = C ñeå xem ñöôøng thaúng( )Δ ( )Δ naøy coù thoûa maõn ñieàu
kieän cuûa ñaàu baøi khoâng.
Ghi chuù - Neáu n = (A, B) laø 1 phaùp veùc tô cuûa ñöôøng thaúng ( )Δ thì
k.n = (kA, kB) cuõng laø phaùp veùc tô cuûa ( )Δ vôùi moïi soá thöïc k ≠ 0.
- Neáu laø 1 veùc tô chæ phöông cuûa ñöôøng thaúng1 2=a (a ,a ) ( )Δ thì
k. cuõng laø veùc tô chæ phöông cuûa1 2=a (ka ,ka ) ( )Δ vôùi moïi soá thöïc k khaùc 0.
II. VÒ TRÍ TÖÔNG ÑOÁI CUÛA HAI ÑÖÔØNG THAÚNG
Ñeå xeùt vò trí töông ñoái cuûa hai ñöôøng thaúng ta caàn nhôù
Cho (d1) : A1x + B1y + C1 = 0
vaø (d2) : A2x + B2y + C2 = 0
Ñaët :
2
D =
1 1
2 2
A B
A B
; Dx =
1 1
2 2
B C
B C
; Dy =
1
2 2
C A
C A
1
thì :
D ≠ 0 ⇔ (d1) caét (d2) taïi I
1
x
I
y
D
x
D
D
y
D
⎧
=⎪⎪
⎨
⎪ =
⎪⎩
D = 0 vaø Dx 0 hoaëc Dy≠ ≠ 0 ⇔ (d1) // (d2)
D = Dx = Dy = 0 ⇔ (d1) ≡ (d2)
hoaëc vôùi A2, B2, C2 0 ta coù :≠
1
2
A
A
≠ 1
2
B
B
⇔ (d1) caét (d2)
1
2
A
A
= 1
2
B
B
≠ 1
2
C
C
⇔ (d1) // (d2)
1
2
A
A
= 1
2
B
B
= 1
2
C
C
⇔ (d1) ≡ (d2)
Ghi chuù
1 1
2 2
B C
B C
=
1 1
2 2
−
C B
C B
;
1 1
2 2
C A
C A
=
1 1
2 2
−
A C
A C
III. GOÙC GIÖÕA HAI ÑÖÔØNG THAÚNG
Ñeå tìm goùc giöõa hai ñöôøng thaúng, ta goïi α laø goùc nhoïn taïo bôûi hai ñöôøng thaúng
(d1) : A1x + B1y + C1 = 0 (d2) : A2x + B2y + C2 = 0
thì cosα = 1 2 1 2
2 2 2
1 1 2 2
2
A A B B
A B . A B
+
+ +
IV. KHOAÛNG CAÙCH TÖØ MOÄT ÑIEÅM ÑEÁN MOÄT ÑÖÔØNG THAÚNG
Ñeå tìm khoaûng caùch töø ñieåm M(xM, yM) ñeán ñöôøng thaúng
( )Δ : Ax + By + C = 0 ta aùp duïng coâng thöùc :
3
d(M,Δ ) =
2 2
M MAx By C
A B
+ +
+
Khoaûng caùch ñaïi soá töø ñöôøng thaúng ( )Δ ñeán ñieåm M(xM, yM) laø :
t =
2 2
M MAx By
A B
+ +
+
C
Ñaët phaùp vectô = (A, B) coù goác leânn ( )Δ thì :
. t > 0 neáu ñieåm M vaø n naèm cuøng moät beân ñoái vôùi ( )Δ
. t < 0 neáu ñieåm M vaø n naèm khaùc beân ñoái vôùi ( )Δ
Phöông trình ñöôøng phaân giaùc cuûa goùc hôïp bôûi 2 ñöôøng thaúng
(d1) : A1x + B1y + C1 = 0 vaø
(d2) : A2x + B2y + C2 = 0 laø :
1 1
2 2
1 1
1A x B y C
A B
+ +
+
= ± 2 2 2
2 2
2 2
A x B y C
A B
+ +
+
Ví duï 1:
Cho tam giaùc ABC vôùi A(–2, 1), B(4, 3), C(2,–3)
a) Tìm phöông trình tham soá vaø toång quaùt caïnh BC.
b) Tìm phöông trình ñöôøng cao AH.
c) Tìm phöông trình ñöôøng thaúng qua A(–2, 1) vaø song song vôùi BC.
Giaûi
a) Ñöôøng thaúng qua caïnh BC nhaän BC = (–2, –6) hay (1,3) laøm vectô chæ phöông vaø
qua B(4, 3) neân coù phöông trình tham soá :
(t
4
3 3
= +⎧
⎨
= +⎩
x t
y t
∈ R)
⇔
4
1
−x
=
3
3
−y
(phöông trình chính taéc)
⇔ 3x – y – 9 = 0 laø phöông trình toång quaùt cuûa BC.
b) Δ ABC coù ñöôøng cao AH ⊥ BC : 3x – y – 9 = 0
⇒ pt AH : x + 3y + C1 = 0
4
A(–2, 1) ∈ AH –2 + 3(1) + C1 = 0⇔ ⇔ C1 = –1
Vaäy pt AH : x + 3y – 1 = 0
c) Ñöôøng thaúng Au // BC ⇒pt Au : 3x – y + C2 = 0
A(–2, 1) ∈ Au ⇔ 3(–2) – 1 + C2 = 0 ⇔ C2 = 7
Vaäy pt Au : 3x – y + 7 = 0
Ví duï 2:
Cho tam giaùc ABC vôùi A(1, –1), B(–2, 1), C(3, 5).
a) Vieát phöông trình ñöôøng vuoâng goùc AH keû töø A ñeán trung tuyeán BK cuûa tam giaùc
ABC.
b) Tính dieän tích tam giaùc ABK.
Giaûi
a) K laø trung ñieåm cuûa AC ⇔
2
2
2
2
A C
K
A C
K
x x
x
y y
y
+⎧
= =⎪⎪
⎨
+⎪ = =
⎪⎩
hay K(2, 2)
Phöông trình caïnh BK :
2
2 2
x −
− −
=
2
1 2
y −
−
⇔ x – 4y + 6 = 0
AH ⊥ BK pt AH : 4x + y + C0 = 0⇒
A(1, - 1) ∈ AH 4(1) + (–1) + C0 = 0⇔
⇔ C0 = –3 hay AH : 4x + y – 3 = 0
b) Dieän tích tam giaùc ABK laø S =
1
2
AH.BK vôùi
AH = A (BK)d =
1 4 6
17
+ +
S =⇒
1
2
.
11
17
. 2 2
4 1+ =
11
2
( ñvdt ).
Ví duï 3: ( Ñeà döï tröõ khoái A naêm 2005) Trong maët phaúng vôùi heä toïa ñoä Oxy cho tam giaùc
ABC caân taïi ñænh A coù troïng taâm G
4 1
( ; )
3 3
, phöông trình ñöôøng thaúng BC laø vaø
phöông trình ñöôøng thaúng BG laø
2 4x y− − = 0
07 4 8x y− − = .Tìm toïa ñoä caùc ñænh A, B, C.
5
Baøi giaûi
Toïa ñoä ñænh B laø nghieäm cuûa heä pt ( )
− − =⎧
⇒ −⎨
− − =⎩
x 2y 4 0
B 0, 2
7x 4y 8 0
Vì caân taïi A neân AG laø ñöôøng cao cuûaABCΔ ABCΔ
Vì ⇒ pt GA:GA BC⊥ − + − = ⇔ + − =
4 1
2(x ) 1(y ) 0 2x y 3 0
3 3
2x y 3 0⇔ + − =
⇒ = HGA BC∩ ( )
+ − =⎧
⇒ −⎨
− − =⎩
2x y 3 0
H 2, 1
x 2y 4 0
Ta coù H laø trung ñieåm BC ⇒
+ = = − = − =⎧ ⎧
⇒⎨ ⎨
+ = = − = − − − =⎩ ⎩
B C H C H B
B C H C H B
x x 2x x 2x x 2(2) 0 4
y y 2y y 2y y 2( 1) ( 2) 0
)⇒ . Ta coù :(C 4,0
+ + + +
= =A B C A B C
G G
x x x y y y
x vaø y
3 3
⇒ ( )A 0,3
Vaäy ( ) ( ) (A 0,3 ,C 4,0 ,B 0, 2− )
Ví duï 4 ( ÑH KHOÁI A -2002) 1. Trong maët phaúng vôùi heä toïa ñoä Ñeâcac vuoâng goùc Oxy cho
hình chöõ nhaät ABCD coù taâm I
1
;0
2
⎞
⎟
⎝ ⎠
⎛
⎜ ,phöông trình ñöôøng thaúng AB laø
x – 2y + 2 = 0 vaø AB = 2AD .Tìm toïa ñoä caùc ñænh A,B,C,D bieát raèng ñænh A coù hoaønh ñoä aâm .
BAØI GIAÛI: A ∈ ñöôøng thaúng x – 2y + 2 = 0
⇒ A (2a – 2, a) (a < 1)
I laø trung ñieåm AC ⇒ C (3 – 2a, −a)
BC qua C vaø BC ⊥ AB
⇒ pt BC : 2x + y + 5a – 6 = 0
AB ∩ BC = B ⇒ B (2 – 2a, 2 – a)
Ta coù : AB = 2AD ⇔ (1 – a)2
= 1 ⇔ a = 0 hay a = 2 (loaïi)
Vaäy A (−2, 0). B (2, 2), C (3, 0), D (−1, −2)
Ví duï 5 ( ÑH KHOÁI D -2004) Trong maët phaúng vôùi heä toïa ñoä Oxy cho tam giaùc ABC coù caùc ñænh
A (−1; 0); B (4; 0); C (0; m) vôùi m ≠ 0. Tìm toïa ñoä troïng taâm G cuûa tam giaùc ABC theo m.
Xaùc ñònh m ñeå tam giaùc GAB vuoâng taïi G.
BAØI GIAÛI: G
m
1;
3
⎛ ⎞
⎜ ⎟
⎝ ⎠
;
m
GA ( 2; )
3
= − − ;
m
GB (3; )
3
= −
Tam giaùc GAB vuoâng taïi G ⇔ GA.GB 0=
⇔
2
m
6
9
− + = 0 ⇔ m = 3 6± .
Ví duï6 ( ÑH KHOÁI B -2004) Trong maët phaúng vôùi heä toïa ñoä Oxy cho hai ñieåm
A(1; 1), B(4; -3). Tìm ñieåm C thuoäc ñöôøng thaúng 2 1 0x y− − = sao cho khoaûng caùch töø C ñeán
ñöôøng thaúng AB baèng 6.
BAØI GIAÛI: A (1; 1); B (4; −3) ⇒ phöông trình AB:
x 1 y 1
3 4
− −
=
−
⇔ 4x + 3y – 7 = 0 C ∈ ñt : x – 2y – 1 = 0 ⇒ C (2t + 1; t)
6
Ta coù: d (C, AB) = 6 ⇔
8t 4 3t 7
6
5
+ + −
=
⇔ 11t 3 30− = ⇔ ⇔
11t 3 30
11t 3 30
− =⎡
⎢ − = −⎣
t 3
27
t
11
=⎡
⎢
⎢ = −
⎢⎣
Vaäy C (7; 3) hay C
43 27
;
11 11
⎛ ⎞
− −⎜ ⎟
⎝ ⎠
Ví duï7 ( Ñeà DÖÏ TRÖÕ KHOÁI D -2003) Trong maët phaúng vôùi heä toïa ñoä Ñeàcac vuoâng goùc Oxy cho
tam giaùc ABC coù ñænh A (1; 0) vaø hai ñöôøng thaúng laàn löôït chöùa caùc ñöôøng cao veõ töø B vaø C coù
phöông trình töông öùng laø :
x – 2y + 1 = 0 vaø 3x + y – 1 = 0.Tính dieän tích cuûa tam giaùc ABC.
BAØI GIAÛI: Vì AC ⊥ BB' ⇒ phöông trình AC : 2x + y + m = 0
A(1; 0) ∈ AC ⇒ 2 + m = 0 ⇒ m = −2
Phöông trình AC : 2x + y – 2 = 0
Vaäy t ñ C laø nghieäm cuûa
+ − =⎧
⎨
+ − =⎩
2x y 2 0
3x y 1 0
⇒ C(−1; 4)
Vì AB ⊥ CC' ⇒ phöông trình AB : x – 3y + n = 0
A(1; 0) ∈ AB ⇒ 1 + n = 0 ⇒ n = −1
Phöông trình AB : x – 3y – 1 = 0
Vaäy ⇒ B(−5; −2).⇒
x 3y 1 0
B
x 2y 1 0
− − =⎧
⎨
− + =⎩
AB
⎯→
= (−6; −2); AC
⎯→
= (−2; 4)
SΔABC =
− −⎡
⎢−⎣ ⎦
6 21
2 42
⎤
⎥ = 14 (ñvdt).
Ví duï8 ( ÑEÀDÖÏ TRÖÕ KHOÁI B -2004) Trong maët phaúng vôùi heä toïa ñoä Oxy cho ñieåm I (–2; 0) vaø
hai ñöôøng thaúng d1 : 2x – y + 5 = 0, d2 : x + y – 3 = 0. Vieát phöông trình ñöôøng thaúng d ñi qua ñieåm I vaø
caét hai ñöôøng thaúng d1, d2 laàn löôït taïi A, B sao cho : 2
→ →
=IA IB
BAØI GIAÛI: P.trình ñöôøng thaúng d qua I (–2, 0), heä soá goùc k : y = k(x + 2)
⎩
⎨
⎧
⎟
⎠
⎞
⎜
⎝
⎛
−
−
−
−
⇒
=+−
=+−
k
k
,
k
k
A
kykx
yx
A
22
52
02
052
⎩
⎨
⎧
⎟
⎠
⎞
⎜
⎝
⎛
++
−
⇒
=+−
=−+
k
k
,
k
k
B
kykx
yx
B
1
5
1
23
02
03
1 k
IA ;
2 k 2 k
− −⎛ ⎞
= ⎜ ⎟
− −⎝ ⎠
; ⎟
⎠
⎞
⎜
⎝
⎛
++
=
k
k
;
k
IB
1
5
1
5
⇒ ⎟
⎠
⎞
⎜
⎝
⎛
++
=
k
k
;
k
IB
1
10
1
10
2
⎪
⎩
⎪
⎨
⎧
==⇒
+
=
−
−
=⇒
+
=
−
−
⇔=
3
7
0
1
10
2
3
7
1
10
2
1
2
k,k
k
k
k
k
k
kkIBIA
Do ñoù phöông trình ñöôøng thaúng d laø y =
3
7
(x + 2)
7
⇔ 7x – 3y + 14 = 0
* * *
8
CHUYEÂN ÑEÀ 4
ÑÖÔØNG TROØN
1. Ñeå tìm phöông trình cuûa moät ñöôøng troøn ta caàn löu yù:
. Phöông trình cuûa ñöôøng troøn (C) taâm I(a, b) baùn kính R laø :
( ) + ( = R22
x a− )
2
y b−
. Phöông trình cuûa (C) ôû daïng khai trieån :
x2
+ y2
– 2ax – 2by + c = 0 ( hay x2
+ y2
+ 2ax + 2by + c = 0)
vôùi c = a2
+ b2
– R2
R2
=⇔ 2 2
a b c+ −
Do ñoù ta phaûi coù ñieàu kieän a2
+ b2
– c 0≥
. Phöông trình tham soá cuûa ñöôøng troøn taâm I(a, b) baùn kính R laø:
(t
x a R cost
y b R sin t
= +⎧
⎨
= +⎩
∈ R)
2. Ñeå vieát phöông trình tieáp tuyeán vôùi moät ñöôøng troøn ta caàn phaân bieät :
a) Tröôøng hôïp bieát tieáp ñieåm : ta duøng coâng thöùc phaân ñoâi toïa ñoä :
Tieáp tuyeán ( taïi tieáp ñieåm M0(x0, y0) vôùi :)Δ
- ñöôøng troøn (C) : ( ) + = R2
laø
2
x a− (
2
y b− )
)
(x0 – a) (x – a) + (y0 – b) (y – b) = R2
- ñöôøng troøn (C) : x2
+ y2
– 2ax – 2by + c = 0 laø
x0x + y0y – a(x0 + x) – b(y0 + y) + c = 0
b) Tröôøng hôïp khoâng bieát tieáp ñieåm, ta aùp duïng tính chaát :
Ñöôøng thaúng ( tieáp xuùc vôùi ñöôøng troøn taâm I baùn kính R)Δ
⇔ = R.Δd(I, )
c) ñöôøng troøn (C) : ( ) + = R2
coù 2 tieáp tuyeán cuøng phöông vôùi Oy laø x =
a R. Ngoaøi 2 tieáp tuyeán x = a
2
x a− (
2
y b−
± ± R, moïi tieáp tuyeán khaùc vôùi ñöôøng troøn ( C) ñeàu coù
daïng y = kx + m hoaëc daïng y = k ( x –x0 ) + y0 neáu tieáp tuyeán ñi qua ( x0 , y0 ) laø
ñieåm naèm ngoaøi ñöôøng troøn.
Ví duï
1
Trong maët phaúng Oxy cho A(–2, 0), B(0, 4).
a) Vieát phöông trình ñöôøng troøn (C) qua 3 ñieåm O, A, B.
b) Vieát phöông trình caùc tieáp tuyeán vôùi ñöôøng troøn (C) taïi A, B.
c) Vieát phöông trình caùc tieáp tuyeán vôùi (C) phaùt xuaát töø ñieåm M(4, 7)
Giaûi
a) Phöông trình ñöôøng troøn (C) coù daïng :
x2
+ y2
– 2ax – 2by + c = 0
Ñöôøng troøn (C) qua 3 ñieåm O, A, B neân :
0
4 4 0
16 8 0
c
a c
b c
=⎧
⎪
+ + =⎨
⎪ − + =⎩
⇔
0
1
2
c
a
b
=⎧
⎪
= −⎨
⎪ =⎩
Vaäy (C) : x2
+ y2
+ 2x – 4y = 0.
Caùch khaùc: Tam giaùc ABC vuoâng taïi O neân coù taâm laø trung ñieåm cuûa AB vaø ñöôøng kính laø
AB neân pt döôøng troøn (C) laø:
2 2 21 1
1 2 4 16
4 4
+ + − = = + =(x ) ( y ) AB ( ) 5
Caùch khaùc: Tam giaùc ABC vuoâng taïi O neân vôùi ( , ) ( )M x y C∈ ta coù
0=AM.BM . Vaäy pt ñöôøng troøn ( C ) laø 0− − + − − =A B A B(x x )(x x ) ( y y )( y y ) .
b) Phöông trình tieáp tuyeán vôùi (C) taïi :
. Tieáp ñieåm A(–2, 0) laø : –2x + 0.y + (–2 + x) – 2(0 + y) = 0
⇔ x + 2y + 2 = 0
. Tieáp ñieåm B(0, 4) laø : 0.x + 4.y + (0 + x) – 2(4 + y) = 0
⇔ x + 2y – 8 = 0
c) Ñöôøng troøn (C) : x2
+ y2
+ 2x – 4y = 0 coù taâm I(–1, 2) vaø baùn kính R = 2
1 2 0+ − =
5 .Hai tieáp tuyeán cuøng phöông vôùi Oy laø 1= ± = − ±x a R 5 . Hai tieáp tuyeán
naøy khoâng qua M(4, 7)
Vaäy phöông trình tieáp tuyeán qua M(4, 7) coù daïng:
( )Δ : y – 7 = k(x – 4)
⇔ kx – y + 7 – 4k = 0
( )Δ tieáp xuùc vôùi ñöôøng troøn (C) ⇔ Δd(I, ) = R
2
⇔
2
2 7 4
1
k k
k
− − + −
+
= 5 ⇔ 5 5k− = 5 . 2
1k +
⇔ 4k2
– 10k + 4 = 0 ⇔ k = 2 hay k =
1
2
Vaäy coù 2 tieáp tuyeán vôùi ñöôøng troøn (C) phaùt xuaát töø ñieåm M(4, 7) vôùi phöông trình laø :
k = 2 2x – y – 1 = 0⇒
k =
1
2
⇒
1
2
x – y + 5 = 0.
Ví duï (ÑH KHOÁI B-2003)
Trong maët phaúng vôùi heä toïa ñoä Ñeâcac vuoâng goùc Oxy cho tam giaùc ABC coù AB=AC, 0
90BAC = .
Bieát M(1,–1) laø trung ñieåm caïnh BC vaø G(
2
3
; 0) laø troïng taâm tam giaùc ABC. Tìm toïa ñoä caùc
ñænh A , B, C.
G laø troïng taâm ΔABC ⇔ =AG 2GM
⇔
⎧
− = − =⎪
⎨
⎪− = − − = −⎩
A
A
2 2 2
3
=x 2(1 )
3 3
y 2( 1 0) 2
⇔
⎧
⎨
=⎩
A
A
x 0
2
⇔ A (0, 2)
y
PT: BC qua M (1, −1) ⊥ = (1, −3): x – 3y – 4 = 0AM
PT ñ.troøn (C) taâm M, baùn kính R = AM= + =1 9 10
(x – 1)2
+ (y + 1)2
= 10
Toïa ñoä B, C thoûa :
− − =⎧
⎨
− + + =⎩
2 2
x 3y 4 0
(x 1) (y 1) 10
⇔ ⇔
= +⎧
⎨
+ + + = ⇔ + =⎩
2 2 2
x 3y 4
(3y 3) (y 1) 10 (y 1) 1
=⎧
⎨
=⎩
x 4
y 0
∨
= −⎧
⎨
= −⎩
x 2
y 2
Vaäy B (4, 0); C(−2, −2) hay B(−2, −2); C (4, 0)
Ví duï (ÑH KHOÁI D-2003) Trong maët phaúng vôùi heä toïa ñoä Ñecac vuoâng goùc Oxy cho ñöôøng
troøn (C): (x – 1)2
+ (y – 2)2
= 4 vaø ñöôøng thaúng d: x – y – 1 = 0. Vieát phöông trình ñöôøng troøn
(C’) ñoái xöùng vôùi ñöôøng troøn (C) qua ñöôøng thaúng d. Tìm toïa ñoä caùc giao ñieåm (C) vaø (C’)
Giaûi
(C1) coù taâm I (1, 2), R = 2.
Goïi I’ laø ñoái xöùng I qua (d)
Goïi (Δ) laø ñöôøng thaúng qua I vaø (Δ) ⊥ (d)
(Δ) : x + y – 3 = 0. (Δ) ∩ (d) = H(2, 1)
H laø trung ñieåm cuûa II’
Giaû söû I’ (x, y) thì ⇒
+⎧
=⎪⎪
⎨
+⎪ =
⎪⎩
x 1
2
2
y 2
1
2
⇒
=⎧
⎨
=⎩
x 3
y 0
⇒ I’ (3, 0); R’ = R = 2. (C’) : (x – 3)2
+ y2
= 4
3
Giaûi heä
⎧
⇔
− + − =⎪
⎨
− + =⎪⎩
2 2
2 2
(x 1) (y 2) 4
(x 3) y 4
⎧ − + =
⎨
− − =⎩
2 2
(x 3) y 4
x y 1 0
⇔ ⇔ ∨
= +⎧
⎨
− =⎩
2
x y 1
2y 4y 0
=⎧
⎨
=⎩
x 1
y 0
=⎧
⎨
=⎩
x 3
y 2
Vaäy giao ñieåm cuûa (C) vaø (C’) laø A (1, 0) vaø B (3, 2).
Ví duï (ÑH KHOÁI A-2005) Trong maët phaúng vôùi heä toïa ñoä Oxy, cho hai ñöôøng thaúng
d1 : x – y = 0 vaø d2 : 2x + y – 1 = 0.Tìm toïa ñoä caùc ñænh hình vuoâng ABCD bieát raèng ñænh A
thuoäc d1, ñænh C thuoäc d2 vaø caùc ñænh B, D thuoäc truïc hoaønh.
Giaûi
A ∈ d1 ⇔ A (m; m). C ∈ d2 ⇔ C (n; 1 – 2n)
Vì B, D ∈ Ox vaø ABCD laø hình vuoâng neân :
A vaø C ñoái xöùng nhau qua Ox ⇔
m n
m 2n 1
=⎧
⎨
= −⎩
⇔
m 1
n 1
=⎧
⎨
=⎩
Suy ra A(1; 1), C(1; -1). Goïi (C) laø ñöôøng troøn ñöôøng kính AC
⇒ Phöông trình (C) : (x–1)2
+y2
=1. B vaø D laø giao ñieåm (C) vaø Ox neân toïa ñoä cuûa B, D
laø nghieäm cuûa heä :
2 2(x 1) y 1
y 0
⎧⎪ − + =
⎨
=⎪⎩
⇔ . Suy ra B (0; 0), D(2; 0) hay B(2; 0), D(0; 0)
= ∨ =⎧
⎨
=⎩
x 0 x 2
y 0
Vaäy A(1; 1), B (0; 0), C(1; -1), D(2; 0)
hay A(1; 1), B(2; 0), C(1; -1), D(0; 0).
Ví duï (ÑH KHOÁI B-2005)Trong maët phaúng vôùi heä toïa ñoä Oxy, cho hai ñieåm A(2; 0), B(6; 4).
Vieát phöông trình ñöôøng troøn (C) tieáp xuùc vôùi truïc hoaønh taïi ñieåm A vaø khoaûng caùch töø taâm
cuûa (C) ñeán ñieåm B baèng 5.
Giaûi
Goïi I (x; y) laø taâm cuûa (C). Ta coù : (C) tieáp xuùc Ox taïi A ⇒ IA i⊥ = (1; 0) ⇔ x – 2 = 0
⇔ x = 2
IB = 5 ⇔ (x – 6)2
+ (y – 4)2
= 25
⇔ (2 – 6)2
+ (y – 4)2
= 25 ⇔ (y – 4)2
= 9
⇔ y – 4 = ±3 ⇔ y = 7 hay y = 1
Tröôøng hôïp 1: I(2; 7) ⇒ R = d(I, Ox) = 7
Suy ra pt (C) : (x – 2)2
+ (y – 7)2
= 49
Tröôøng hôïp 2: I (2; 1) ⇒ R = d(I, Ox) = 1
⇒ pt (C) : (x – 2)2
+ (y – 1)2
= 1.
Ví duï
(ÑEÀ DÖÏ BÒ KHOÁI A -2002)
Trong maët phaúng vôùi heä toïa ñoä Ñeàcac vuoâng goùc Oxy, cho hai ñöôøng troøn:
(C1) : x2
+ y2
– 10x = 0; (C2) : x2
+ y2
+ 4x – 2y – 20 = 0
4
1) Vieát phöông trình ñöôøng troøn ñi qua caùc giao ñieåm cuûa (C1), (C2) vaø coù taâm naèm treân
ñöôøng thaúng x + 6y – 6 = 0.
2) Vieát phöông trình tieáp tuyeán chung cuûa caùc ñöôøng troøn (C1) vaø (C2).
Giaûi
1) Phöông trình chuøm ñöôøng troøn qua caùc giao ñieåm cuûa (C1), (C2) laø :
m(x2
+ y2
– 10x) + n(x2
+ y2
+ 4x – 2y – 20) = 0 vôùi m2
+ n2
> 0
⇔ (m + n)x2
+ (m + n)y2
+ (4n – 10m)x – 2ny – 20n = 0
⇔ x2
+ y2
+
4n 10m 2n 20n
x y
m n m n m n
−⎛ ⎞
− −⎜ ⎟
+ + +⎝ ⎠
0=
Coù taâm I
5m 2n n
;
m n m n
−⎛ ⎞
⎜ ⎟
+ +⎝ ⎠
Vì taâm I ∈ d : x + 6y – 6 = 0 ⇒
5m 2n 6n 6m 6n
0
m n
− + − −
=
+
⇒ m = −2n . Cho n = 1 ⇒ m = −2
Vaäy phöông trình ñöôøng troøn laø :x2
+ y2
– 24x + 2y + 20 = 0.
2) Vieát phöông trình caùc tieáp tuyeán chung cuûa (C1), (C2).
(C1) coù taâm I1(5; 0), baùn kính R1 = 5 ⇒ I1I2 < R1 + R2
(C2) coù taâm I2(−2; 1), baùn kính R2 = 5
Vì (C1), (C2) caét nhau taïi 2 ñieåm neân coù 2 tieáp tuyeán chung.
Vì x = xo khoâng theå laø tieáp tuyeán chung neân pt tt chung Δ coù daïng :
y = ax + b ⇔ ax – y + b = 0
Δ tieáp xuùc vôùi (C1) ⇔ d(I1, Δ) = R1 ⇔
2
5a b
5
a 1
⏐ + ⏐
=
+
⇔⏐5a + b⏐ = 2
5 a 1+ (1)
Δ tieáp xuùc vôùi (C2) ⇔ d(I2, Δ) = R2 ⇔
2
2a 1 b
a 1
⏐− − + ⏐
+
= 5
⇔ ⏐−2a – 1 + b⏐ = 2
5 a (2)1+
(1) vaø (2) ⇒ ⏐5a + b⏐ = ⏐−2a – 1 + b⏐
⇔ ⇔
5a b 2a 1 b
5a b 2a 1 b
+ = − − +⎡
⎢ + = + + −⎣
1
a
7
3a 1
b
2
⎡
= −⎢
⎢
− +⎢ =
⎢⎣
Theá a =
1
7
− vaøo (1) ta coù : b1 =
5 25 2
7
+
; b2 =
5 25 2
7
−
Vaäy ta coù 2 tieáp tuyeán laø : x + 7y – 5 + 25 2 = 0
x + 7y – 5 − 25 2 = 0.
Caùch khaùc: Vì R = R2 vaø 2 ñöôøng troøn caét nhau neân 2 tieáp tuyeán chung laø 2 ñöôøng
thaúng song song vôùi Vaäy phöông trình 2 tieáp tuyeán coù daïng :
1
1 2I I ( 7;1)= −
x + 7y+m = 0 (Δ)
d(I1, Δ) = 5 ⇔ ⏐5 + m⏐ = +2
5 7 1 ⇔ m = – 5 ± 25 2 Vaäy
phöông trình 2 tieáp tuyeán laø x + 7y – 5 ± 25 2 = 0.
5
GHI CHUÙ :
Baøi ñöôøng troøn trong chöông trình lôùp 12 bao goàm caùc vaán ñeà chính laø : Tìm phöông
trình ñöôøng troøn; caùc baøi toaùn lieân quan ñeán vò trí töông ñoái giöõañöôøng thaúng vaø ñöôøng troøn,
giöõa hai ñöôøng troøn; phöông tích cuûa moät ñieåm ñoái vôùi ñöôøng troøn; truïc ñaúng phöông cuûa hai
ñöôøng troøn khoâng ñoàng taâm. Ngoaøi ra coøn coù moät soá caâu hoûi lieân quan ñeán phöông trình x2
+
y2
+ 2Ax + 2By +C = 0 (1). Chaúng haïn tìm ñieàu kieän ñeå (1) laø phöông trình ñöôøng troøn. Töø
phöông trình (1) tìm taâm vaø baùn kính cuûa ñöôøng troøn, tìm tham soá ñeå baùn kính thoaû moät ñieàu
kieän naøo ñoù . . .
Sau ñaây, chuùng toâi chæ ñeà caäp ñeán caùch tìm phöông trình ñöôøng troøn noäi tieáp tam giaùc
vaø vaøi öùng duïng truïc ñaúng phöông cuûa hai ñöôøng troøn khoâng ñoàng taâm. Ñaây laø vaán ñeá caùc em
thöôøng “ sôï” khi gaëp phaûi.
A/ Caùch tìm phöông trình ñöôøng troøn noäi tieáp tam giaùc ABC :
Tröôùc heát caàn löu yù :
• Taâm ñöôøng troøn noäi tieáp tam giaùc laø giao ñieåm cuûa hai ñöôøng phaân giaùc trong .
• Muoán tìm phöông trình ñöôøng troøn ta tìm taâm I (a ; b) vaø baùn kính R. Khi ñoù phöông trình
ñöôøng troøn coù daïng (x – a)2
+ (y – b)2
= R2
.
• Cho k laø soá thöïc khaùc 1, ta coù :
⎪
⎪
⎩
⎪⎪
⎨
⎧
−
−
=
−
−
=
⇔=
k1
kyy
y
k1
kxx
x
MBkMA
BA
M
BA
M
(I)
1/ Neáu ñeà baøi cho bieát toïa ñoä A, B, C thì :
• Goïi D laø chaân ñöôøng phaân giaùc trong keû töø A cuûa
tam giaùc ABC.
Ta coù : DC
AC
AB
DB −=
Söû duïng coâng thöùc (I) vôùi k =
AC
AB
− ta xaùc ñònh ñöôïc toïa ñoä ñieåm D.
A
B CD
I
• Goïi I laø taâm ñöôøng troøn noäi tieáp tam giaùc ABC thì I chính laø chaân ñöôøng phaân giaùc
trong keû töø B cuûa tam giaùc ABD.
Ta coù : ID
BD
BA
IA −=
Söû duïng coâng thöùc (I) vôùi k =
BD
BA
− laø xaùc ñònh ñöôïc toïa ñoä taâm I.
Coøn baùn kính ñöôøng troøn noäi tieáp tam giaùc chính laø khoaûng caùch töø taâm I ñeán moät
trong 3 caïnh cuûa tam giaùc ABC.
Chuù yù : Neáu moät trong ba ñænh cuûa tam giaùc truøng vôùi goác toïa ñoä vaø hai ñænh coøn laïi
naèm treân hai truïc toïa ñoä thì caùch giaûi ñöôïc thu goïn hôn vì bieát tröôùc ñöôïc 1 ñöôøng phaân giaùc
trong keû töø goác toïa ñoä. Ñöôøng phaân giaùc coøn laïi ñöôïc tìm thoâng qua tìm chaân ñöôøng phaân giaùc
trong nhö ñaõ trình baøy ôû treân.
6
2/ Neáu ñeà baøi cho bieát phöông trình 3 caïnh cuûa tam giaùc ABC thì töø phöông trình 3 caïnh
ñoù, ta tìm ñöôïc toïa ñoä caùc ñieåm A, B, C baèng caùch giaûi heä phöông trình toïa ñoä giao ñieåm vaø
söû duïng caùch giaûi nhö phaàn 1.
Ngoaøi ra coøn coù theå giaûi baèng kieán thöùc mieàn taïo bôûi 1 ñöôøng thaúng vaø khoaûng caùch
ñaïi soá töø moät ñieåm ñeán ñöôøng thaúng.
B/ Truïc ñaúng phöông cuûa hai ñöôøng troøn khoâng ñoàng taâm :
1/ Cho hai ñöôøng troøn khoâng ñoàng taâm :
(C1) : x2
+ y2
+ 2a1x + 2b1y + c1 = 0 (1)
(C2) : x2
+ y2
+ 2a2x + 2b2y + c2 = 0 (2)
Truïc ñaúng phöông cuûa (C1) vaø (C2) laø taäp hôïp caùc ñieåm coù cuøng phöông tích ñoái vôùi
(C1) vaø (C2) vaø coù phöông trình laø :
2(a1 – a2)x + 2(b1 – b2)y + c1 – c2 = 0
2/ ÖÙng duïng :
Trong chöông trình Hình hoïc lôùp 10 ta ñaõ bieát caùch döïng truïc ñaúng phöông cuûa (C1) vaø
(C2).
• Neáu (C1) vaø (C2) caét nhau taïi 2 ñieåm A vaø B thì truïc ñaúng phöông cuûa (C1) vaø (C2) laø
ñöôøng thaúng AB.
• Neáu (C1) vaø (C2) tieáp xuùc nhau (Tieáp xuùc trong hoaëc tieáp xuùc ngoaøi) thì truïc ñaúng
phöông cuûa (C1) vaø (C2) laø tieáp tuyeán chung cuûa (C1) vaø (C2) taïi tieáp ñieåm.
• Neáu (C1) vaø (C2) khoâng caét nhau thì veõ theâm ñöôøng troøn (C3) sao cho caét ñöôïc (C1),
(C2) vaø coù taâm khoâng naèm treân ñöôøng noái taâm cuûa (C1), (C2). Goïi M laø giao ñieåm cuûa hai truïc
ñaúng phöông cuûa (C1) vaø (C3), (C2) vaø (C3). Khi ñoù truïc ñaúng phöông cuûa (C1) vaø (C2) laø ñöôøng
thaúng qua M vaø vuoâng goùc vôùi ñöôøng noái taâm cuûa (C1) vaø (C2).
Baøi toaùn : Cho ñöôøng troøn (C) vaø M laø ñieåm naèm ngoaøi (C). Töø M keû MA vaø MB laø hai tieáp
tuyeán cuûa (C) (A vaø B laø hai tieáp ñieåm). Vieát phöông trình ñöôøng thaúng AB.
Caùch giaûi : Goïi I laø taâm vaø R laø baùn kính cuûa ñöôøng troøn (C).
Goïi (C’) laø ñöôøng troøn taâm M, baùn kính :
R’ = MA = 22
RIM −
Suy ra (C) vaø (C’) caét nhau taïi A vaø B.
Do ñoù ñöôøng thaúng AB chính laø truïc ñaúng phöông cuûa (C)
vaø (C’).
(C) (C’)A
B
MI
Qua keát quaû treân ta ghi nhôù ngay 2 keát quaû :
• Ñöôøng thaúng ñi qua giao ñieåm cuûa hai ñöôøng troøn (C1) vaø (C2) chính laø truïc ñaúng
phöông cuûa (C1) vaø (C2) [Nghóa laø khoâng caàn tìm toïa ñoä giao ñieåm cuûa (C1) vaø (C2)].
• Tieáp tuyeán chung cuûa 2 ñöôøng troøn (C1) vaø (C2) tieáp xuùc nhau taïi tieáp ñieåm chính laø
truïc ñaúng phöông cuûa (C1) vaø (C2).
Sau ñaây, löu yù theâm 2 baøi toaùn thöôøng gaëp :
Baøi 1 : Cho (C1) vaø (C2) ôû ngoaøi nhau. Tìm quyõ tích nhöõng ñieåm M töø ñoù veõ ñöôïc ñeán (C1) vaø
(C2) nhöõng ñoaïn tieáp tuyeán baèng nhau.
Caùch giaûi : Goïi MA vaø MB (nhö hình veõ) laø 2 tieáp tuyeán töø M ñeán (C1) vaø (C2)
Ta coù : MA = MB ⇔ MA2
= MB2
⇔ 1 2/( ) /( )M C M CP P=
Do ñoù quyõ tích M laø truïc ñaúng phöông cuûa (C1) vaø (C2).
•
7
M
• BA •
(C1)
(C2)
Baøi 2 : Tìm tieáp ñieåm M cuûa hai ñöôøng troøn tieáp xuùc nhau (C1) vaø (C2)
Goïi I1 vaø I2 laø taâm cuûa (C1) vaø (C2). Tieáp ñieåm M chính
laø giao ñieåm cuûa truïc ñaúng phöông cuûa (C1) vaø (C2) vôùi
ñöôøng noái taâm I1I2.
(C2)
(C1)
MI1 I2
d
Ví duï (ÑEÀ DÖÏ BÒ KHOÁI B -2005)
Trong maët phaúng vôùi heä toïa ñoä Oxy cho 2 ñöôøng troøn :
(C1 ): x2
+ y2
vaø (C2 ): x2
+ y2
. Vieát phöông trình truïc ñaúng phöông d cuûa
2 ñöôøng troøn (C1) vaø (C2). Chöùng minh raèng neáu K thuoäc d thì khoûang caùch töø K ñeán taâm cuûa
(C1) nhoû hôn khoûang caùch töø K ñeán taâm cuûa ( C2 ).
9= 2 2 23x y− − − = 0
Giaûi:
Ñöôøng troøn ( )1C coù taâm ( )O 0 baùn kính R 3,0 1 =
Ñöôøng troøn ( )2C coù taâm ( )I 1 , baùn kính,1 2R 5=
Phöông trình truïc ñaúng phöông cuûa 2 ñöôøng troøn ( )1C , ( )2C laø
( ) ( )2 2 2 2
x y 9 x y 2x 2y 23+ − − + − − − = 0
x y 7 0⇔ + + = (d)
Goïi ( ) ( )k k k kK x ,y d y x 7∈ ⇔ = − −
( ) ( ) ( )= − + − = + = + − − = + +
2 2 22 2 2 2 2
k k k k k k k kOK x 0 y 0 x y x x 7 2x 14x 49
( ) ( ) ( ) ( )2 2 2 22 2
k k k k k kIK x 1 y 1 x 1 x 8 2x 14x 6= − + − = − + − − = + + 5
Ta xeùt ( ) ( )2 2 2 2
k k k kIK OK 2x 14x 65 2x 14x 49 16 0− = + + − + + = >
K OK IK OK(ñpcm)> ⇔ >Vaäy I 2 2
* * *
8
CHUYEÂN ÑEÀ 5
ELIP
Caùc baøi toaùn veà elip chuû yeáu qui veà vieäc vieát phöông trình chính taéc cuûa elip, xaùc ñònh
caùc phaàn töû cuûa elip (taâm, ñænh, tieâu cöï, ñoä daøi truïc lôùn, truïc nhoû, tieâu ñieåm…), nhaát laø xaùc
ñònh phöông trình cuûa tieáp tuyeán cuøng vôùi toïa ñoä tieáp ñieåm. Trong moïi tröôøng hôïp ta caàn naém
vöõng kieán thöùc cô baûn sau ñaây :
. Elip (E) coù tieâu ñieåm
treân x′x
. Elip (E) coù tieâu ñieåm
treân y′y
Phöông trình
chính taéc
Tieâu cöï
Tieâu ñieåm
Truïc lôùn
Truïc nhoû
Ñænh treân truïc lôùn
Ñænh treân truïc nhoû
Taâm sai
Baùn kính qua tieâu
Ñieåm cuûa M ∈ (E)
Ñöôøng chuaån
(E) :
2
2
x
a
+
2
2
y
b
= 1
a2
> b2
vaø a2
– b2
= c2
2c
F1(–c, 0), F2(c, 0)
Treân Ox, daøi 2a
Treân Oy, daøi 2b
A1(–a, 0), A2(a, 0)
B1(0, –b), B2(0, b)
e =
c
a
1 1
2 2
M
M
r FM a ex
r F M a ex
= = +⎧
⎨
= = −⎩
1 2,Δ : x = ±
a
e
(E) :
2
2
x
a
+
2
2
y
b
= 1
a2
< b2
vaø b2
– a2
= c2
2c
F1(0, –c), F2(0, c)
Treân Oy, daøi 2b
Treân Ox, daøi 2a
A1(0, –b), A2(0, b)
B1(–a, 0), B2(a, 0)
e =
c
b
1 1
2 2
M
M
r FM b ey
r F M b ey
= = +⎧
⎨
= = −⎩
1 2,Δ : y = ±
b
e
* Ghi chuù :
1
Tröôøng hôïp elip coù taâm I( ,α β ) hai truïc cuøng phöông vôùi 2 truïc toïa ñoä thì phöông trình
coù daïng
( )
2
2
x
a
− α
+
( )
2
2
y
b
− β
= 1
Ta dôøi heä truïc toïa ñoä xOy ñeán XIY baèng pheùp tònh tieán theo OI ñeå ñöôïc phöông trình
daïng chính taéc cuûa elip laø
2
2
X
a
+
2
2
Y
b
= 1 vôùi
X x
Y y
= − α⎧
⎨
= − β⎩
ñeå suy ra deã daøng toïa ñoä caùc ñænh vaø tieâu ñieåm.
. Tieáp tuyeán vôùi elip (E) :
2
2
x
a
+
2
2
y
b
= 1 taïi tieáp ñieåm M0(x0, y0) coù phöông trình 0
2
x x
a
+ 0
2
y y
b
= 1
. Tröôøng hôïp khoâng bieát tieáp ñieåm ta aùp duïng tính chaát :
: Ax + By + C = 0 tieáp xuùc vôùi elip( )Δ
(E) :
2
2
x
a
+
2
2
y
b
= 1 a2
A2
+ b2
B2
= C2
⇔
Thöôøng ta vieát phöông trình cuûa ( )Δ theo heä soá goùc ôû daïng
kx – y + c = 0 vaø löu yù tröôøng hôïp ( )Δ ⊥ x′x töùc
( )Δ : x = ± a
. Elip (E) :
2
2
x
a
+
2
2
y
b
= 1 coù 2 tieáp tuyeán cuøng phöông vôùi Oy laø
x = a. Ngoaøi 2 tieáp tuyeán x = a, moïi tieáp tuyeán khaùc vôùi ( E) ñeàu coù daïng± ±
y = kx + m hoaëc daïng y = k ( x –x0 ) + y0 neáu tieáp tuyeán ñi qua ( x0 , y0 ) laø ñieåm naèm ngoaøi
elip.
Ví duï1 :
Cho elip (E) : x2
+ 4y2
– 40 = 0
a) Xaùc ñònh tieâu ñieåm, hai ñænh treân truïc lôùn, 2 ñænh treân truïc nhoû vaø taâm sai cuûa (E).
b) Vieát phöông trình tieáp tuyeán vôùi (E) taïi ñieåm M0(–2, 3).
c) Vieát phöông trình tieáp tuyeán vôùi elip (E) bieát noù xuaát phaùt töø ñieåm M(8, 0).
2
d) Vieát phöông trình tieáp tuyeán vôùi (E) bieát noù vuoâng goùc vôùi ñöôøng thaúng (D) : 2x – 3y
+ 1 = 0, tính toïa ñoä tieáp ñieåm.
Giaûi
a) Tieâu ñieåm, caùc ñænh vaø taâm sai cuûa (E)
(E) : x2
+ 4y2
– 40 = 0
⇔
2
x
40
+
2
10
y
= 1 coù daïng
2
2
x
a
+
2
2
y
b
= 1
vôùi a2
= 40 > b2
= 10 c2
= a2
– b2
= 30⇒
a = 2⇒ 10 , b = 10 , c = 30
Vaäy elip (E) coù truïc lôùn treân Ox, hai tieâu ñieåm naèm treân truïc lôùn laø
F1(– 30 , 0) , F2( 30 , 0).
Hai ñænh treân truïc lôùn laø A1(–2 10 , 0), A2(2 10 , 0)
Truïc nhoû cuûa (E) naèm treân Oy vôùi 2 ñænh laø B1(0, – 10 ), B2(0, 10 ).
Taâm sai cuûa elip (E) laø e =
c
a
=
30
2 10
=
3
2
b) Vieát phöông trình tieáp tuyeán vôùi (E) taïi M0(–2, 3)
Ta coù + 4 – 40 = (2
0x 2
0y )
2
2− + 4 – 40 = 0( )
2
3
M0(–2, 3) ∈ (E) : x2
+ 4y2
– 40 = 0⇒
Phöông trình tieáp tuyeán vôùi (E) taïi tieáp ñieåm M0(–2, 3) seõ laø:⇒
x0x + 4y0y – 40 = 0 ⇔ –2x + 12y – 40 = 0
⇔ x - 6y + 20 = 0
c) Phöông trình tieáp tuyeán vôùi elip phaùt xuaát töø M(8, 0).
(E) coù hai tieáp tuyeán cuøng phöông vôùi 0y laø: x = 2 10± .Hai tieáp tuyeán naøy khoâng ñi qua
M(8,0). Vaäy pt tieáp tuyeán ( qua M(8, 0) coù daïng:)Δ
y= k(x – 8) ⇔ kx – y – 8k = 0
( )Δ tieáp xuùc vôùi elip (E) :
2
x
40
+
2
y
10
= 1
⇔ 40k2
+ 10 = 64k2
3
⇔ k2
=
10
24
=
5
12
⇔ k = ±
5
2 3
= ±
15
6
Vaäy coù 2 tieáp tuyeán vôùi (E) qua M(8, 0) laø :
15
6
x – y – 8
5
6
= 0 ⇔ 15 x – 6y – 8 5 = 0
hay –
15
6
x – y + 8
5
6
= 0 ⇔ 15 x + 6y – 8 5 = 0
d) Phöông trình tieáp tuyeán vôùi (E) vaø vuoâng goùc vôùi (D)
(D) vôùi (D) : 2x – 3y + 1 = 0( )′Δ ⊥
⇒ : 3x + 2y + C = 0( )′Δ
( )′Δ tieáp xuùc (E) :
2
x
40
+
2
y
10
= 1
⇔ 40.9 + 10.4 = C2
⇔ C2
= 400
⇔ C = ± 20
Goïi M0(x0, y0) laø tieáp ñieåm cuûa tieáp tuyeán ( )′Δ vôùi (E) thì ( )′Δ :
0x x
40
+ 0y y
10
= 1 ⇔ x0x + 4y0y – 40 = 0
Vôùi C = 20 : 3x + 2y + 20 = 0⇒ ( )′Δ
⇒ 0x
3
= 04y
2
=
40
20
−
⇔
0
0
x 6
y 1
= −⎧
⎨
= −⎩
hay M0 (–6, –1)
Vôùi C = –20 (⇒ )′Δ : 3x + 2y – 20 = 0
⇒ 0x
3
= 04y
2
=
40
20
−
−
⇔ hay M0(6, 1).
0
0
x 6
y 1
=⎧
⎨
=⎩
4
Ví duï2 :(ÑH KHOÁI D-2005) Trong maët phaúng vôùi heä toïa ñoä Oxy cho ñieåm C (2; 0) vaø elíp
(E) :
2 2
x y
1
4 1
+ = . Tìm toïa ñoä caùc ñieåm A, B thuoäc (E), bieát raèng hai ñieåm A, B ñoái xöùng vôùi
nhau qua truïc hoaønh vaø tam giaùc ABC laø tam giaùc ñeàu
Giaûi
Giaû söû A (a,
2
4 a
2
−
) ∈ (E) ⇒ B (a, −
2
4 a
2
−
) ∈ (E)
Vaø ñieàu kieän: –2 < a < 2. Do A,B ñoái xöùng qua Ox neân ta coù:
ΔCAB ñeàu ⇔ CA2
= AB2
⇔ (a – 2)2
+
2
4 a
4
−
= 4 – a2
⇔ 7a2
– 16a + 4 = 0
⇔ a = 2 (loaïi) hay a = 2
7
. Neân toïa ñoä cuûa A vaø B laø:
A
2 4 3
,
7 7
⎛ ⎞
⎜ ⎟⎜ ⎟
⎝ ⎠
vaø B
2 4 3
,
7 7
⎛
−⎜⎜
⎝ ⎠
⎞
⎟⎟
hoaëc A
2 4 3
,
7 7
⎛ ⎞
−⎜ ⎟⎜ ⎟
⎝ ⎠
vaø B
2 4 3
,
7 7
⎛ ⎞
⎜ ⎟⎜ ⎟
⎝ ⎠
Ví duï3 :(ÑH KHOÁI D-2002) :
Cho(E) :
9
y
16
x 22
+ = 1. Cho M di chuyeån treân tia 0x, N di chuyeån treân tia 0y sao cho ñöôøng
thaúng MN luoân tieáp xuùc (E). Tìm toïa ñoä ñieåm M, N sao cho ñoä daøi ñoaïn MN ngaén nhaát. Tìm
ñoä daøi ñoaïn ngaén nhaát ñoù.
Giaûi
M (m, 0) ∈ tia Ox; N (0, n) ∈ tia Oy ⇒ n, m > 0
(E) :
9
y
16
x 22
+ = 1. MN : nx + my – n.m = 0
(MN) tieáp xuùc (E) ⇔ 1
n
9
m
16
22
=+
Ta coù : MN2
= m2
+ n2
.Theo BÑT BCS ta coù
Ta coù : 7 = MNnm
n
9
m
16
n.
n
3
m.
m
4 22
22
=++≤+
MN nhoû nhaát ⇒
n
3
n
m
4
m
= ⇔
3
n
4
m 22
=
⇔ 3m2
= 4n2
vaø m2
+ n2
= 49 ⇔ m2
= 28 vaø n2
= 21
Do ñoù : MN nhoû nhaát ⇔ m = 72 vaø n = 21 (vì m, n>0)
⇒ M ( 72 , 0); N (0, 21 ). Khi ñoù min MN = 7.
Ví duï4 :(ÑH KHOÁI D-2005) Trong maët phaúng vôùi heä toïa ñoä Ñeàcac vuoâng goùc Oxy, cho elip (E):
1
4
y
9
x 22
=+ vaø ñöôøng thaúng dm : mx – y – 1 = 0.
5
a) Chöùng minh raèng vôùi moïi giaù trò cuûa m, ñöôøng thaúng dm luoân caét elip (E) taïi hai ñieåm phaân
bieät.
b) Vieát phöông trình tieáp tuyeán cuûa (E), bieát raèng tieáp tuyeán ñoù ñi qua ñieåm N (1; −3).
Giaûi
a) (E) :
2 2
x y
1
9 4
+ = ⇔ 4x2
+ 9y2
– 36 = 0
(dm) : mx – y – 1 = 0 ⇔ y = mx – 1
Phöông trình hoaønh ñoä giao ñieåm cuûa (dm) vôùi (E) :
4x2
+ 9(mx – 1)2
– 36 = 0 ⇔ (4 + 9m2
)x2
– 18mx – 25 = 0
coù Δ' = 81m2
+ 25(4 + 9m2
) > 0 ñuùng vôùi moïi m
Vaäy (dm) luoân luoân caét (E) taïi 2 ñieåm phaân bieät.
b) Vieát phöông trình tieáp tuyeán vôùi (E) qua N(1; −3)
2 tieáp tuyeán thaúng ñöùng cuûa (E) laø x = ± 3 ( khoâng qua N )
Goïi Δ laø tieáp tuyeán qua N(1; −3) thì phöông trình Δ coù daïng:
y + 3 = k(x – 1) ⇔ kx – y – 3 – k = 0
(Δ) tieáp xuùc vôùi (E) ⇔ 9k2
+ 4 = (−3 – k)2
= 9 + 6k + k2
⇔ 8k2
– 6k – 5 = 0 ⇔
1
2
1
k
2
5
k
4
⎡
= −⎢
⎢
⎢ =⎢⎣
Δ1 : x + 2y + 5 = 0; Δ2 : 5x – 4y – 17 = 0.
* * *
6
CHUYEÂN ÑEÀ 6
HYPEBOL
Ñeå giaûi caùc baøi toaùn coù lieân quan ñeán ñöôøng hypebol ta caàn naém vöõng caùc vaán ñeà cô
baûn sau:
Hypebol (H) coù taâm O, hai truïc ñoái xöùng laø x′x, y′y.
Phöông trình
chính taéc
. Hypebol coù tieâu ñieåm
treân x′x
2
2
x
a
–
2
2
y
b
= 1
. Hypebol coù tieâu ñieåm t
reân y′y
2
2
x
a
–
2
2
y
b
= –1
vôùi c2
= a2
+ b2
vôùi c2
= a2
+ b2
Tieâu ñieåm
Tieâu cöï
Truïc thöïc, ñoä daøi
Truïc aûo, ñoä daøi
Ñænh
Tieäm caän
Taâm sai
Baùn kính
M(xM, yM) ∈ (H)
F1(–c, 0), F2(c, 0)
2c
Ox, 2a
Oy, 2b
A1(–a, 0), A2(a, 0)
y = ±
b
a
x
e =
c
a
1 1
2 2
M
M
r FM ex a
r F M ex a
= = +⎧
⎨
= = −⎩
(xM a)≥
1
2
M
M
r ex
r ex
a
a
= − −⎧
⎨
= − +⎩
(xM ≤ – a)
F1(0, –c), F2(0, c)
2c
Oy, 2b
Ox, 2a
A1(0, –b), A2(0, b)
y = ±
a
b
x
e =
c
b
1 1
2 2
M
M
r FM ey b
r F M ey b
= = +⎧
⎨
= = −⎩
(yM ≥ b)
1
2
M
M
r ey
r ey
b
b
= − −⎧
⎨
= − +⎩
(yM ≤ – b)
1
Ñöôøng chuaån
Phöông trình tieáp
tuyeán taïi tieáp
ñieåm M0(x0, y0) ∈ (H)
x = ±
a
e
0
2
x x
a
– 0
2
y y
b
= 1
y = ±
b
e
0
2
x x
a
– 0
2
y y
b
= –1
Ngoaøi ra ta cuõng caàn löu yù:
. Ñieàu kieän ñeå:
(D) : Ax + By + C = 0 tieáp xuùc vôùi (H) :
2
2
x
a
–
2
2
y
b
= 1 laø
a2
A2
– b2
B2
= C2
> 0
(D) : Ax + By + C = 0 tieáp xuùc vôùi (H) :
2
2
x
a
–
2
2
y
b
= –1 laø
a2
A2
– b2
B2
= –C2
< 0
Ví duï :
Cho hypebol (H) : 4x2
– y2
= 4
1) Xaùc ñònh tieâu ñieåm, ñænh, taâm sai, caùc ñöôøng tieäm caän vaø ñöôøng chuaån cuûa (H)
2) Vieát phöông trình tieáp tuyeán vôùi (H) taïi ñieåm M(1, 0)
3) Vieát phöông trình tieáp tuyeán vôùi (H) phaùt xuaát töø ñieåm N(1, 4) tìm toïa ñoä tieáp ñieåm.
Giaûi
1) Caùc phaàn töû cuûa hypebol (H)
(H) : 4x2
– y2
= 4 x2
–⇔
2
4
y
= 1 coù daïng
2
2
x
a
–
2
2
y
b
= 1 vôùi
a2
= 1 a = 1, b2
= 4 ⇒ b = 2 vaø c2
= a2
+ b2
= 5⇒
Vaäy hypebol (H) coù 2 tieâu ñieåm F1( 5− , 0), F2( 5 , 0) ; hai ñænh A1(–1, 0), A2(1, 0) ;
taâm sai e =
c
a
= 5 ; hai ñöôøng tieäm caän phöông trình y = ± 2x vaø hai ñöôøng chuaån phöông
trình
x = ±
a
e
= ±
1
5
2
2) Phöông trình tieáp tuyeán vôùi (H) taïi tieáp ñieåm M(1, 0)
Ta coù M(1, 0) ∈ (H) :4x2
– y2
= 4
⇒ Phöông trình tieáp tuyeán vôùi (H) taïi tieáp ñieåm M(1, 0) laø
4xMx – yMy = 4
⇔ 4x – 0y = 4 x = 1⇔
3) Phöông trình tieáp tuyeán vôùi (H) phaùt xuaát töø N(1, 4). Hai tieáp tuyeán cuøng phöông vôùi
0y laø x = a = 1. Vaäy x=1 laø moät tieáp tuyeán qua N(1, 4).± ±
Tieáp tuyeán ( qua N(1, 4) khoâng cuøng phöông vôùi 0y coù daïng:)Δ
: y – 4 = k(x – 1)( )Δ ⇔ kx – y + 4 – k = 0
( )Δ tieáp xuùc vôùi hypebol (H) :
2
1
x
–
2
4
y
= 1
⇔ k2
. 12
– 4(–1)2
= (4 – k)2
⇔ k2
- 4 = 16 – 8k + k2
⇔ k =
20 5
8 2
= .Vaäy :( )Δ
5
2
x – y – 4 –
5
2
= 0
⇔ 5x – 2y – 13 = 0
Toùm laïi coù hai tieáp tuyeán qua ñieåm N(1, 4) laø x = 1, vaø 5x – 2y – 13 = 0.
* * *
3
CHUYEÂN ÑEÀ 7
PARABOL
Caùc baøi toaùn veà parabol thöôøng qui veà vieäc xaùc ñònh caùc yeáu toá cuûa parabol (tieâu
ñieåm, ñöôøng chuaån), laäp phöông trình cuûa parabol vaø caùc vaán ñeà veà tieáp tuyeán cuûa parabol.
Do ñoù ta caàn naém vöõng caùc kieán thöùc cô baûn sau ñaây :
Parabol (P) = { M∈ (Oxy) / MF = ( )M
d Δ }
F laø tieâu ñieåm vaø ( laø ñöôøng chuaån.)Δ
Caùc daïng phöông trình chính taéc :
(P) : y2
= 2px
( )Δ : x =
2
p
−
F 0
2
p
,
⎛ ⎞
⎜ ⎟
⎝ ⎠
M ∈ (P) ⇒ xM 0≥
vaø r = MF = xM +
2
p
(d) : Ax + By + C = 0 tieáp xuùc vôùi (P) ⇔
pB2
= 2AC
Tieáp tuyeán vôùi (P) taïi tieáp ñieåm
(P) : y2
= –2px
y
x
(P)
F
y
( )Δ : x =
2
p
F 0
2
p
,
⎛ ⎞
−⎜ ⎟
⎝ ⎠
M ∈ (P) xM 0⇒ ≤
vaø r = MF = –xM +
2
p
(d) : Ax + By + C = 0 tieáp xuùc vôùi (P) ⇔
pB2
= –2AC
Tieáp tuyeán vôùi (P) taïi tieáp ñieåm
x
(P)
F(P
2
, 0)P
2
−
O
( )Δ
P
2
O
( )Δ
1
M0(x0, y0) coù phöông trình
y0y = p(x0 + x)
(P) : x2
= 2py
( )Δ : y =
2
p
−
F 0
2
p
,
⎛ ⎞
⎜ ⎟
⎝ ⎠
M ∈ (P) ⇒ yM 0≥
vaø r = MF = yM +
2
p
(d) : Ax + By + C = 0 tieáp xuùc vôùi (P) ⇔
pA2
= 2BC
Tieáp tuyeán vôùi (P) taïi tieáp ñieåm
M0(x0, y0) coù phöông trình
x0x = p(y0 + y)
M0(x0, y0) coù phöông trình
y0y = –p(x0 + x)
(P) : x2
= –2py
( )Δ : y =
2
p
F 0
2
p
,
⎛ ⎞
−⎜ ⎟
⎝ ⎠
M ∈ (P) yM 0⇒ ≤
vaø r = MF = –yM +
2
p
(d) : Ax + By + C = 0 tieáp xuùc vôùi (P) ⇔
pA2
= –2BC
Tieáp tuyeán vôùi (P) taïi tieáp ñieåm
M0(x0, y0) coù phöông trình
x0x = –p(y0 + y)
Ví duï1 :
Cho parabol (P) : y2
– 8x = 0
1) Xaùc ñònh tieâu ñieåm F vaø ñöôøng chuaån ( )Δ cuûa (P)
2) Vieát phöông trình tieáp tuyeán vôùi (P) taïi ñieåm M(2; –4)
y
x
(P)
FP
2
O
( )Δ
y
x
(P)
F− P
2
O
( ) P
2
Δ
2
3) Vieát phöông trình tieáp tuyeán vôùi (P) bieát noù song song vôùi ñöôøng thaúng (D) : 2x – y +
5 = 0. Suy ra toïa ñoä tieáp ñieåm.
4) Vieát phöông trình tieáp tuyeán vôùi (P) bieát noù xuaát phaùt töø ñieåm
I(–3, 0), suy ra toïa ñoä tieáp ñieåm.
Giaûi
1) Tieâu ñieåm vaø ñöôøng chuaån
(P) : y2
– 8x = 0 y2
= 8x coù daïng y2
= 2px vôùi p = 4⇔
Tieâu ñieåm F(2, 0) vaø ñöôøng chuaån⇒ ( )Δ : x = –2.
2) Phöông trình tieáp tuyeán vôùi (P) taïi M(2; –4)
Tieáp tuyeán vôùi (P) : y2
= 8x taïi tieáp ñieåm M(2, –4) coù phöông trình cho bôûi coâng thöùc
phaân ñoâi toïa ñoä :
–4(y) = 4(2 + x) ⇔ x + y + 2 = 0
3) Phöông trình tieáp tuyeán vôùi (P) vaø song song vôùi (D)
Ñöôøng thaúng (d) // (D) vôùi (D) : 2x – y + 5 = 0
(d) : 2x – y + C = 0⇒
(d) tieáp xuùc vôùi (P) : y2
= 8x
4 = 2 . 2C = 4C⇔ ⇔ C = 1
Vaäy tieáp tuyeán vôùi (P) phaûi tìm coù phöông trình
2x – y + 1 = 0
Tieáp tuyeán (d) vôùi (P) : y2
= 8x taïi tieáp ñieåm M0(x0, y0) coøn coù phöông trình
y0y = 4(x0 + x) ⇔ 4x – y0y + 4x0 = 0
maø (d) : 2x – y + 1 = 0, do ñoù :
4
2
= 0
1
y
= 04
1
x
⇒ 0
0
1
2
2
x
y
⎧
=⎪
⎨
⎪ =⎩
hay M0
1
2
2
,
⎛ ⎞
⎜ ⎟
⎝ ⎠
4) Phöông trình tieáp tuyeán vôùi (P) xuaát phaùt töø I(–3, 0).
Tieáp tuyeán vôùi (P) vaø cuøng phöông vôùi 0y laø x = 0. Vaäy pt tieáp tuyeán ( ) quad′
I(–3, 0) coù daïng:
(d ) : y – 0 = k(x + 3)′ ⇔ kx – y + 3k = 0
3
( ) tieáp xuùc vôùi (P) : y2
= 8xd′
4 = 2k(3k) = 6k2
k =⇔ ⇔ ±
2
6
= ±
6
3
Vaäy töø ñieåm I(–3, 0) coù 2 tieáp tuyeán vôùi parabol (P) laø:
6
3
x – y + 6 = 0 hay –
6
3
x – y – 6 = 0
6
3
⇔ x – y + 6 = 0 hay 6 x +3 y +3 6 = 0
Tieáp tuyeán (d ) vôùi (P) taïi tieáp ñieåm M0(x0, y0) coù phöông trình′
4x – y0y + 4x0 = 0
Do ñoù vôùi (d ) :′
6
3
x – y + 6 = 0 ⇒
4
6
3
= 0
1
y
= 04
6
x
⇒
0
0
3
12
2 6
6
x
y
=⎧
⎪
⎨
= =⎪
⎩
Vôùi ( ) :d′ 6 x + 3y + 3 6 = 0 ⇒
4
6
= 0
3
y−
= 04
3 6
x
⇒
0
0
3
12
2 6
6
x
y
=⎧
⎪
⎨
= − = −⎪
⎩
Vaäy 2 tieáp ñieåm phaûi tìm laø (3; 2 6 ) vaø (3; –2 6 ).
Ví du2( ÑEÀ DÖÏ TRÖÕKHOÁI A –2003) : Trong maët phaúng vôùi heä toïa ñoä Ñeàcac vuoâng goùc Oxy, cho
parabol (P) coù phöông trình y2
= x vaø ñieåm I (0; 2). Tìm toïa ñoä hai ñieåm M, N thuoäc (P) sao cho
IN4IM = .
Giaûi
Goïi M(m2
; m) ∈ (P), N(n2
; n) ∈ (P)
IM
⎯→
= (m2
; m – 2)
IN
⎯→
= (n2
; n – 2)
IN
⎯→
= (4n2
; 4n – 8)⇒ 4
4
Vì IM
⎯→
= 4 IN
⎯→
⇔
2 2
m 4n
m 2 4n 8
⎧ =⎪
⎨
− = −⎪⎩
⇔ ⇒ ⎢2
m 4n 6
n 4n 3
= −⎧⎪
⎨
− + =⎪⎩ 0 =⎣
1
2
n 1
n 3
=⎡ 1
2
m 2
m 6
⇒ = −
⇒ =
⇒ M1(4; −2), N1(1; 1), M2(36; 6), N2(9; 3)
Ví du 3 ( ÑEÀ DÖÏ TRÖÕKHOÁI A –2003) :Trong maët phaúng vôùi heä toïa ñoä Ñeàcac vuoâng goùc Oxy cho
elip (E): 1
1
y
4
x 22
=+ . M(−2; 3); N(5; n). Vieát phöông trình caùc ñöôøng thaúng d1, d2 qua M vaø tieáp xuùc
vôùi (E). Tìm n ñeå trong soá caùc tieáp tuyeán cuûa (E) ñi qua N coù moät tieáp tuyeán song song vôùi d1 hoaëc d2.
Giaûi
1) Vieát phöông trình caùc ñöôøng thaúng qua M tieáp xuùc vôùi E.
x = 2 laø 2 tieáp tuyeán thaúng ñöùng cuûa (E)±
Vaäy d1 : x = −2 laø 1 tieáp tuyeán cuûa (E) qua M.
Phöông trình tieáp tuyeán d qua M(−2; 3) khaùc döôøng thaúng x = −2
coù daïng : y – 3 = k(x + 2)
O
3
x
y
−2
M⇔ kx – y + 3 + 2k
d tieáp xuùc vôùi (E)
⇔ 4k2
+ 1 = (3 + 2k)2
⇔ 4k2
+ 1 = 9 + 4k2
+ 12k
8 2
12 3
−
= −⇔ k =
d2 : 2x + 3y – 5 = 0
2) deã thaáy tieáp tuyeán d cuûa (E) qua N(5; n) khoâng song song vôùi :
x = −2.
Do ñoù d song song vôùi d2 : 2x + 3y – 5 = 0 vaø qua N(5; n) coù heä soá goùc :
k = −
2
3
= − − +
2
y (x 5)
3
n. Vaäy d : hay
d :
2
− − + n = 0 ⇔ −2x – 3y + 10 + 3n = 0
10
x y
3 3
+
d tieáp xuùc vôùi E ⇔ 4(−2)2
+ 1.(−3)2
= (10 + 3n)2
−
5
3
⇔ 3n2
+ 20n + 25 = 0⇔ n = – 5 hay n=
−
5
3
: loaïi vì khi ñoù d truøng vôùi d1.n =
Vaäy N(5; −5).
* * *
5
CHUYEÂN ÑEÀ 8
VECTÔ TRONG KHOÂNG GIAN
Caùc ñònh nghóa vaø pheùp toaùn cuûa vectô trong khoâng gian cuõng gioáng nhö trong maët
phaúng, ta caàn löu yù ñeán caùc vaán ñeà cô baûn thoâng duïng nhö :
. Qui taéc 3 ñieåm : A, B, C thì∀ AB + BC = AC
. Coäng 2 vectô cuøng goác laø moät vectô cuøng goác vaø laø ñöôøng cheùo hình bình haønh coù 2
caïnh laø 2 vectô ñaõ cho.
. I laø trung ñieåm ñoaïn thaúng AB, vôùi ñieåm M baát kyø naøo ta luoân coù:
MI =
2
MA MB+
. G laø troïng taâm cuûa Δ ABC ⇔ GA + GB + GC = 0 .
Ngoaøi ra ta coøn coù :
. Ba vectô khaùc goïi laø ñoàng phaúng neáu giaù cuûa chuùng cuøng song song hoaëc naèm
trong moät maët phaúng .
0
. Baát kyø vectô a 0 naøo ñoàng phaúng vôùi hai vectô khoâng cuøng phöông , trong
khoâng gian, ñeàu coù theå phaân tích theo
≠ 1e 2e
1e , 2e coù nghóa:
a = α 1e + β 2e (α ,β ∈ R)
vaø söï phaân tích treân laø duy nhaát .
. Baát kyø vectô a naøo trong khoâng gian cuõng coù theå phaân tích ñöôïc theo 3 vectô
khoâng ñoàng phaúng , , coù nghóa :
≠ 0
1e 2e 3e
a = + βα 1e 2e + γ 3e (α ,β , γ ∈ R)
. G ñöôïc goïi laø troïng taâm cuûa töù dieän ABCD
+ + GC +⇔ GA GB GD = 0
Ghi chuù :
1) Neáu moät trong 3 vectô ,a b , c laø 0 thì chuùng ñoàng phaúng.
2) a, b , c ñoàng phaúng ⇔ , . 0a b c⎡ ⎤ =⎣ ⎦
1
3) OA , OB, ñoàng phaúngOC ⇔ O, A, B, C cuøng naèm treân moät maët phaúng.
Ví duï 1:
Cho moät hình laêng truï ABC A′ B′ C′ . Goïi I, I′ laàn löôït laø troïng taâm cuûa Δ ABC vaø
Δ A′ B′ C′ , O laø trung ñieåm cuûa II′ .
a) Chöùng minh raèng
+ + OBOA OA′ + OB′ + OC + OC′ = 0
b) Goïi G laø troïng taâm cuûa hình töù dieän ABCC′ vaø M laø trung ñieåm cuûa A′ B′ . Chöùng
minh raèng O, M, G thaúng haøng.
c) Tính tæ soá
OM
OG
Giaûi
a) + OA + +OA ′ OB OB′ + OC + OC′ = 0
I laø troïng taâm cuûa ABC ⇒Δ IA + IB + IC = 0
( + ) + (IO + OB) + (⇒ IO OA IO + OC) = 0
OA + + OC = 3OI⇒ OB
Töông töï, laø troïng taâm cuûaI′ Δ A′ B′ C′
OA + + OC = 3OI⇒ ′ OB′ ′ ′
Vaäy OA + OA′ + OB + OB′ + OC + OC′ =
= 3OI + 3OI′ = 3(OI + OI′ )
= 0 (vì 0 laø trung ñieåm II′ )
b) O, M, G thaúng haøng
G laø troïng taâm cuûa töù dieän ABCC′
⇒ GA + + GC +GB GC′ = 0
⇒ ( + OA ) + (GO + ) + (GO OB GO + OC) + (GO + OC′ ) = 0
⇒ OA + + OC + OCOB ′ = 4OG
M laø trung ñieåm cuûa A B′ ′
⇒ OA + = 2OM′ OB′
⇒ OA + + OC + OCOB ′ + OA′ + OB′ = 4OG + 2OM
2
⇒ 0 = 4 + 2OMOG
⇒ OM = –2OG
⇒ OM cuøng phöông vôùi OG
⇒ OM, OG cuøng giaù (vì cuøng goác O)
⇒ O, M, G thaúng haøng.
c) Tæ soá
OM
OG
OM = –2OG ⇒
OM
OG
= –2
Ví duï 2:
Cho hình hoäp ABCD. A′ B′ C′ D′ vôùi AA′ = a, AB = b , /
AC = . Haõy bieåu thò caùc
vectô
c
AD , A C′ , , theo caùc vectô aB D′ BD′ , b , c.
Giaûi
Ta coù vôùi hình hoäp ABCD. A′ B′ C′ D′ thì :
AD = AC′ + /
C D′ + D D′
= c – b – a
A C′ = A A′ + /
AC + /
C C
A C′ = –2a + c
B D′ = B B′ + BA + AD
= – a – b + c – –b a
= – 2a – 2 b + c
BD′ = BA + AD + DD′
= – b + (c – – a) +b a
= – 2 b + c
* * *
D′A
B′
′
c
B C
DA
a
C′
b
3
CHUYEÂN ÑEÀ 9
PHÖÔNG PHAÙP TOÏA ÑOÄ TRONG KHOÂNG GIAN
Caùc baøi toaùn veà toïa ñoä trong khoâng gian thöôøng coù caùc yeâu caàu xaùc ñònh toïa ñoä cuûa ñieåm,
vectô, ñoä daøi ñoaïn thaúng, tính goùc 2 vectô, caùc vaán ñeà veà maët phaúng vaø ñöôøng thaúng trong khoâng gian
(phöông trình, vò trí töông ñoái, song song, vuoâng goùc, soá ño goùc, khoaûng caùch,… ). Tuøy theo töøng
tröôøng hôïp ta caàn löu yù vaän duïng caùc kieán thöùc cô baûn sau ñaây :
I. Toaï ñoä ñieåm. Toaï ñoä vectô
Trong khoâng gian toïa ñoä vuoâng goùc Oxyz coù 3 vectô ñôn vò treân ba truïc Ox, Oy, Oz laàn löôït laø
, , .1e 2e 3e
* Cho M(x, y, z) thì OM = x. + y.1e 2e + z. 3e .
* Cho a = (a1, a2, a3) thì a = a1. 1e + a2. 2e + a3. 3e .
II. Caùc pheùp toaùn treân toïa ñoä ñieåm, vectô
1. Caùc pheùp toaùn treân toïa ñoä ñieåm
Cho hai ñieåm A(x1, y1, z1) vaø B(x2, y2, z2). Ta coù nhoùm coâng thöùc tính toïa ñoä vectô AB, khoaûng
caùch giöõa hai ñieåm A, B vaø toïa ñoä ñieåm M laø chia ñoaïn AB theo tæ soá k ≠ 1
* AB = (x2 – x1, y2 – y1, z2 – z1)
* AB = ( ) ( ) ( )
2 2
2 1 2 1 2 1x x y y z z− + − + −
2
* ( x = 1
1
x kx
k
−
−
2
, y = 1
1
y ky
k
−
−
2
, z = 1 2
1
z kz
k
−
−
)
2. Caùc pheùp toaùn treân toïa ñoä vectô
Cho hai vectô a = (a1, a2, a3), = (b1, b2, b3). Vôùib α vaø β laø 2 soá thöïc ta coù caùc coâng thöùc tính
vaø coâng thöùc quan heä sau :
a) Coâng thöùc tính toaùn
. + β . = (α .a1 + .b1, .a2 +α a b β α β .b2, α .a +3 β .b )3
a . b = a1.b1 + a2.b2 + a3 .b3
)cos =(a,b 1 1 2 2 3 3
2 2 2 2 2
1 2 3 1 2 3
a .b a .b a .b
a a a . b b b
+ +
+ + + + 2
b) Coâng thöùc quan heä
1
=a b ⇔
1 1
2 2
3 3
a b
a b
a b
=⎧
⎪
=⎨
⎪ =⎩
cuøng phöônga b ⇔ ( 1
1
a
b
= 2
2
a
b
= 3
3
a
b
) (b1, b2, b3
≠ 0)
⊥ a1.b1 + a2.b2 + a .b = 0a b ⇔ 3 3
Chuù yù :
Goùc hai ñöôøng thaúng cheùo nhau trong khoâng gian laø goùc nhoïn taïo bôûi hai vectô chæ phöông cuûa
2 ñöôøng thaúng ñoù.
MAËT PHAÚNG
I. Phöông trình maët phaúng
1.* Phöông trình tham soá cuûa maët phaúng α qua M(x0, y0, z0) coù caëp vectô chæ phöông a = (a1,
a2, a3
), = (b1, b2, b ) vieát laø :b 3
t1, t2
0 1 1 2 1
0 1 2 2
0 1 3 2 3
x x t a t b
y y t a t b
z z t a t b
= + +⎧
⎪
= + +⎨
⎪ = + +⎩
2 ∈ R
2.* Phöông trình toång quaùt cuûa maët phaúng α laø :
Ax + By + Cz + D = 0vôùi A2
+ B2
+ C2
> 0
Maët phaúng α coù : phaùp vectô : n = (A, B, C)
3.* Phöông trình maët phaúng qua M(x0, y0, z0) vaø vuoâng goùc vôùi vectô
n = (A, B, C) vieát laø : (x – x0)A + (y – y0)B + (z – z0)C = 0
4.* Phöông trình maët phaúng qua M(x0, y0, z0) vaø nhaän 2 vectô chæ phöông
a = (a1, a2, a ), = (b1, b2, b3
) vieát laø3 b
( ) ( ) ( )2 3 3 1 1 2
0 0
2 3 3 1 1 2
0
a a a a a a
x x y y z z
b b b b b b
− + − + − =0 .
5.* Phöông trình maët phaúng caét ba truïc toïa ñoä taïi A(a, 0, 0);
B(0, b, 0); C(0, 0, c) vôùi a.b.c ≠ 0 vieát laø :
x
a
+
y
b
+
z
c
= 1
II. Toaùn treân maët phaúng
1. Khoaûng caùch töø moät ñieåm ñeán moät maët phaúng
Khoaûng caùch töø M(x0, y0, z0) ñeán
2
α : Ax + By + Cz + D = 0 laø :
MH = 0 0 0
2 2 2
Ax By Cz D
A B C
+ + +
+ +
2. Vò trí töông ñoái giöõa hai maët phaúng
Cho hai maët phaúng α , β coù 2 phaùp vectô laàn löôït laø n = (A, B, C),
= (A1, B1, C1)1n
Vò trí giöõa hai maët phaúng , laø vò trí giöõa 2 phaùp vectôα β n , 1n :
// β //α ⇔ n 1n
α ⊥ β ⇔ n ⊥ 1n
caét β khaùc phöôngα ⇔ n 1n
ÑÖÔØNG THAÚNG
I. Phöông trình ñöôøng thaúng
1.* Phöông trình tham soá cuûa ñöôøng thaúng Δ qua
M(x0, y0, z0) coù vectô chæ phöông a = (a1, a2, a ) vieát laø3
0 1
0
0 3
2
x x ta
y y ta
z z ta
= +⎧
⎪
= +⎨
⎪ = +⎩
,t ∈ R (Heä I).
Neáu a1.a2.a3 ≠ 0 ta coù phöông trình chính taéc laø:
x x
a
y y
a
z z
a
−
=
−
=
−0
1
0
2
0
3
2.* Phöông trình toång quaùt cuûa ñöôøng thaúng Δ xaùc ñònh bôûi giao tuyeán 2 maët phaúng α vaø β
vieát laø :
1 1 1 1
0
0
Ax By Cz D ( )
A x B y C z D ( )
+ + + = α⎧
⎨
+ + + =⎩ β
(II)
Ghi chuù:
Cho phöông trình ñöôøng thaúng Δ xaùc ñònh bôûi heä (II). Ñeå vieát thaønh phöông trình tham
soá cuûa ñöôøng thaúng ta coù theå ñaët z = t vaø tính x, y theo t töø heä (II) vaø nhôø heä (I) ta coù ñöôïc vectô
chæ phöông vaø ñieåm cuûa (hoaëc x = t,Δ
hoaëc y = t, neân choïn löïa aån phuï t ñeå pheùp tính hai bieán coøn laïi theo t ñöôïc ñôn giaûn).
3.*Phöông trình maët phaúng chöùa ñöôøng thaúng (d) :
A x B y C z D
A x B y C z D
1 1 1 1
2 2 2 2
0
0
+ + + =
+ + + =
⎧
⎨
⎩
3
Coù daïng : m(A1x + B1y + C1z + D1) + n(A2x + B2y + C2z + D2) = 0 (*) vôùi m, n khoâng ñoàng thôøi
baèng 0. Phöông trình (*) goïi laø phöông trình cuûa chuøm maët phaúng xaùc ñònh bôûi ñöôøng thaúng (d).
Chuù yù :Neáu m= 0 thì n khaùc 0, chia hai veá cuûa (*) cho n ta coù
(*) thaønh A2x + B2y + C2z + D2 = 0
Neáu m khaùc 0 chia hai veá cuûa (*) cho m ta coù:
A1x + B1y + C1z + D1 + h (A2x + B2y + C2z + D2) = 0 vôùi
n
h
m
= .
Vaäy chuøm maët phaúng chöùa ñöôøng thaúng (d) coù daïng:
A1x + B1y + C1z + D1 + h (A2x + B2y + C2z + D2) = 0.
hay A2x + B2y + C2z + D2 = 0.
Vaán ñeà 1
TÌM PHÖÔNG TRÌNH MAËT PHAÚNG
Phöông phaùp :
Thoâng thöôøng ta coù 3 caùch sau :
- Caùch 1 : Tìm moät ñieåm vaø moät caëp vectô chæ phöông cuûa maët phaúng.
- Caùch 2 : Tìm moät ñieåm vaø moät phaùp vectô cuûa maët phaúng.
- Caùch 3 : Duøng phöông trình chuøm maët phaúng.
Vaán ñeà 2 :
TÌM PHÖÔNG TRÌNH ÑÖÔØNG THAÚNG
Phöông phaùp :
Thoâng thöôøng ta coù 2 caùch sau :
- Caùch 1 : Tìm moät ñieåm vaø moät vectô chæ phöông cuûa ñöôøng thaúng.
- Caùch 2 : Tìm phöông trình toång quaùt cuûa 2 maët phaúng phaân bieät cuøng chöùa ñöôøng thaúng caàn tìm.
- Ghi chuù : Trong 2 caùch, thöïc chaát cuûa vieäc tìm phöông trình ñöôøng thaúng laø tìm phöông trình 2 maët
phaúng cuøng chöùa ñöôøng thaúng aáy. Caùi khoù laø phaûi xaùc ñònh ñöôïc 2 maët phaúng phaân bieät naøo cuøng
chöùa ñöôøng thaúng caàn tìm. Thoâng thöôøng ta hay gaëp 3 giaû thuyeát sau :
+ Ñöôøng thaúng (Δ) ñi qua ñieåm A vaø caét ñöôøng thaúng d : Khi ñoù ñöôøng thaúng (Δ) naèm trong maët
phaúng ñi qua A vaø chöùa d.
+ Ñöôøng thaúng (Δ) ñi qua ñieåm A vaø vuoâng goùc vôùi ñöôøng thaúng d : Khi ñoù ñöôøng thaúng (Δ) naèm
trong maët phaúng ñi qua A vaø vuoâng goùc vôùi d.
+ Ñöôøng thaúng (Δ) song song vôùi d1 vaø caét d2 : Khi ñoù ñöôøng thaúng (Δ) naèm trong maët phaúng chöùa d2
vaø song song vôùi d1.
Chaúng haïn :
1. Laäp phöông trình ñöôøng thaúng (Δ) ñi qua ñieåm A, vuoâng goùc vôùi ñöôøng thaúng a vaø caét ñöôøng thaúng
aáy.
Caùch giaûi :
- (Δ) ñi qua A vaø vuoâng goùc vôùi d neân (Δ) naèm trong maët phaúng α ñi qua A vaø vuoâng goùc vôùi d.
- (Δ) ñi qua A vaø caét d neân (Δ) naèm trong maët phaúng β ñi qua A vaø chöùa d. Khi ñoù (Δ) chính laø giao
tuyeán cuûa α vaø β.
2. Laäp phöông trình ñöôøng thaúng (Δ) ñi qua ñieåm A vaø caét caû hai ñöôøng thaúng d1 vaø d2.
Caùch giaûi :
- (Δ) ñi qua A vaø caét d1 neân (Δ) naèm trong maët phaúng α ñi qua A vaø chöùa d1.
4
- (Δ) ñi qua A vaø caét d2 neân (Δ) naèm trong maët phaúng β ñi qua A vaø chöùa d2.
Khi ñoù (Δ) chính laø giao tuyeán cuûa α vaø β.
3. Laäp phöông trình ñöôøng thaúng (Δ) ñi qua giao ñieåm A cuûa ñöôøng thaúng d vaø maët phaúng α, vuoâng
goùc vôùi d vaø naèm trong α.
Caùch giaûi :
- Töø giaû thuyeát ta ñaõ coù (Δ) ⊂ α.
- (Δ) qua A vaø vuoâng goùc vôùi d neân (Δ) naèm trong maët phaúng β ñi qua A vaø vuoâng goùc vôùi d.
Khi ñoù (Δ) chính laø giao tuyeán cuûa α vaø β.
4. Laäp phöông trình ñöôøng thaúng (Δ) song song vôùi ñöôøng thaúng (D) vaø caét 2 ñöôøng thaúng d1 vaø d2.
Caùch giaûi :
- (Δ) song song vôùi (D) vaø caét d1 neân (Δ) naèm trong maët phaúng α chöùa d1 vaø song song vôùi (D).
- (Δ) song song vôùi (D) vaø caét d2 neân (Δ) naèm trong maët phaúng β chöùa d2 vaø song song vôùi (D).
Khi ñoù (Δ) chính laø giao tuyeán cuûa α vaø β.
Vaán ñeà 3
HÌNH CHIEÁU
Baøi toaùn 1 : Tìm hình chieáu vuoâng goùc H cuûa ñieåm A treân ñöôøng thaúng (d)
Phöông phaùp :
(d)
A
H
- Caùch 1 : (d) cho bôûi phöông trình tham soá :
+ H ∈ (d) suy ra daïng toïa ñoä cuûa ñieåm H phuï thuoäc vaøo tham soá t.
+ Tìm tham soá t nhôø ñieàu kieän ⊥aAH
→
d
→
- Caùch 2 : (d) cho bôûi phöông trình chính taéc, goïi H(x, y, z)
+ AH
→
⊥a (*)d
→
+ H ∈ (d) : Bieán ñoåi tæ leä thöùc naøy ñeå duøng ñieàu kieän (*), töø ñoù tìm ñöôïc x, y, z.
- Caùch 3 : (d) cho bôûi phöông trình toång quaùt :
+ Tìm phöông trình maët phaúng α ñi qua A vaø vuoâng goùc vôùi ñöôøng thaúng (d).
+ Giao ñieåm cuûa (d) vaø (α) chính laø hình chieáu H cuûa A treân (d).
Baøi toaùn 2 : Tìm hình chieáu vuoâng goùc H cuûa ñieåm A treân maët phaúng (α)
- Caùch 1 : Goïi H(x, y, z)
+ H ∈ α (*)
+ AH
→
cuøng phöông vôùi : Bieán ñoåi tæ leä thöùc naøy ñeå duøng ñieàu kieän (*), töø ñoù tìm ñöôïc x, y, z.nα
→
- Caùch 2 :
+ Tìm phöông trình ñöôøng thaúng (d) ñi qua A vaø vuoâng goùc vôùi maët phaúng (α).
+ Giao ñieåm cuûa (d) vaø (α) chính laø hình chieáu H cuûa A treân maët phaúng (α).
5
Baøi toaùn 3 : Tìm hình chieáu vuoâng goùc (Δ) cuûa ñöôøng thaúng (d) xuoáng maët phaúng α.
- Tìm phöông trình maët phaúng β chöùa ñöôøng thaúng d vaø vuoâng goùc vôùi maët phaúng α.
- Hình chieáu (Δ) cuûa d xuoáng maët phaúng α chính laø giao tuyeán cuûa α vaø β.
Baøi toaùn 4 : Tìm hình chieáu H cuûa A theo phöông ñöôøng thaúng (d) leân maët phaúng (α).
Phöông phaùp :
- Tìm phöông trình ñöôøng thaúng (Δ) ñi qua A vaø song song vôùi (d).
- Hình chieáu H chính laø giao ñieåm cuûa (Δ) vaø (α).
Baøi toaùn 5 : Tìm hình chieáu (Δ) cuûa ñöôøng thaúng (d) theo phöông cuûa ñöôøng thaúng (D) leân maët
phaúng (α). (Δ)
A
H
(d)
Phöông phaùp :
(D)
d
(Δ)
- Tìm phöông trình maët phaúng (β) chöùa (d) vaø song song vôùi (D)
- Hình chieáu (Δ) chính laø giao tuyeán cuûa (α) vaø (β)
Vaán ñeà4
ÑOÁI XÖÙNG
Baøi toaùn 1 : Tìm ñieåm A’ ñoái xöùng vôùi A qua ñöôøng thaúng d.
Phöông phaùp :
- Tìm hình chieáu H cuûa A treân d.
- H laø trung ñieåm AA’.
Baøi toaùn 2 : Tìm ñieåm A’ ñoái xöùng vôùi A qua maët phaúng α.
Phöông phaùp :
- Tìm hình chieáu H cuûa A treân α.
- H laø trung ñieåm AA’.
Baøi toaùn 3 : Tìm phöông trình ñöôøng thaúng d ñoái xöùng vôùi ñöôøng thaúng (D) qua ñöôøng thaúng (Δ)
Phöông phaùp :
- Tröôøng hôïp 1 : (Δ) vaø (D) caét nhau :
+ Tìm giao ñieåm M cuûa (D) vaø (Δ).
(D)
d
(Δ)M
A
A’
+ Tìm moät ñieåm A treân (D) khaùc vôùi ñieåm M.
+ Tìm ñieåm A’ ñoái xöùng vôùi A qua (Δ)
+ d chính laø ñöôøng thaúng ñi qua 2 ñieåm A’ vaø M.
6
- Tröôøng hôïp 2 : (Δ) vaø (D) song song :
+ Tìm moät ñieåm A treân (D)
+ Tìm ñieåm A’ ñoái xöùng vôùi A qua (Δ)
+ d chính laø ñöôøng thaúng qua A’ vaø song song vôùi (Δ)
- Tröôøng hôïp 3 : (Δ) vaø (D) cheùo nhau :
+ Tìm 2 ñieåm phaân bieät A, B treân (D)
+ Tìm ñieåm A’, B’ laàn löôït laø ñieåm ñoái xöùng cuûa A, B qua (Δ)
+ d chính laø ñöôøng thaúng ñi qua 2 ñieåm A’, B’.
Baøi toaùn 4 : Tìm phöông trình ñöôøng thaúng d ñoái xöùng vôùi ñöôøng thaúng (D) qua maët phaúng α.
Phöông phaùp :
- Tröôøng hôïp 1 : (D) caét α
+ Tìm giao ñieåm M cuûa (D) vaø (α)
+ Tìm moät ñieåm A treân (D)
+ Tìm ñieåm A’ ñoái xöùng vôùi A qua maët phaúng α .
+ d chính laø ñöôøng thaúng ñi qua hai ñieåm A’ vaø M .
- Tröôøng hôïp 2 : (D) song song vôùi α.
A
A’d
(D)
- Tìm moät ñieåm A treân (D)
- Tìm ñieåm A’ ñoái xöùng vôùi A qua maët phaúng α.
- d chính laø ñöôøng thaúng qua A’ vaø song song vôùi (D)
Vaán ñeà 5
KHOAÛNG CAÙCH
Baøi toaùn 1 : Tính khoaûng caùch töø ñieåm M(x0, y0, z0) ñeán maët phaúng α :
Ax + By + Cz + D = 0
Phöông phaùp :
d M
Ax By Cz D
A B C
( , )α =
+ + +
+ +
0 0 0
2 2 2
Baøi toaùn 2 : Tính khoaûng caùch töø ñieåm M ñeán ñöôøng thaúng (Δ)
Phöông phaùp :
- Tìm hình chieáu H cuûa M treân (Δ)
- Khoaûng caùch töø M ñeán (Δ) chính laø ñoä daøi ñoaïn MH.
Baøi toaùn 3 : Tính khoaûng caùch giöõa 2 ñöôøng thaúng song song d1 vaø d2.
Phöông phaùp :
7
- Tìm moät ñieåm A treân d1.
- Khoaûng caùch giöõa d1 vaø d2 chính laø khoaûng caùch töø ñieåm A ñeán d2.
Baøi toaùn 4 : Tính khoaûng caùch giöõa 2 maët phaúng song song α :
Ax + By + Cz + D1 = 0
Vaø β : Ax + By + Cz + D2 = 0
Phöông phaùp :
Khoaûng caùch giöõa α vaø β ñöôïc cho bôûi coâng thöùc : d
D D
A B C
( , )α β =
−
+ +
1 2
2 2 2
Baøi toaùn 5 : Tính khoaûng caùch giöõa 2 ñöôøng thaúng cheùo nhau d1 vaø d2
Phöông phaùp :
- Caùch 1 :
+ Tìm phöông trình maët phaúng α chöùa d1 vaø song song vôùi d2.
+ Tìm moät ñieåm A treân d2.
+ Khi ñoù d(d1, d2) = d(A, α)
- Caùch 2 :
+ Tìm phöông trình maët phaúng α chöùa d1 vaø song song vôùi d2.
+ Tìm phöông trình maët phaúng β chöùa d2 vaø song song vôùi d1.
+ Khi ñoù d(d1, d2) = d(α, β)
Ghi chuù :
Maët phaúng α vaø β chính laø 2 maët phaúng song song vôùi nhau vaø laàn löôït chöùa d1 vaø d2.
- Caùch 3 :
+ Vieát döôùi daïng phöông trình tham soá theo t.
+ Vieát d2 döôùi daïng phöông trình tham soá theo t2.
+ Xem A ∈ d1 ⇒ daïng toïa ñoä A theo t1.
+ Xem B ∈ d2 ⇒ daïng toïa ñoä B theo t2.
+ Tìm vectô chæ phöông laàn löôït cuûa d1 vaø d2.a a1 2
→ →
,
+ AB laø ñoaïn vuoâng goùc chung d1, d2. ⇔ tìm ñöôïc t1 vaø t2
AB a
AB a
→ →
→ →
⊥
⊥
⎧
⎨
⎪
⎩
⎪
1
2
+ Khi ñoù d(d1, d2) = AB
Vaán ñeà 6
GOÙC
Cho 2 ñöôøng thaúng d vaø d’ coù phöông trình :
d :
x x
a
y y
b
z z
c
−
=
−
=
−0 0 0
d’ :
x x
a
y y
b
z z
c
−
=
−
=
−0 0
' '
0
'
Cho 2 maët phaúng α vaø β coù phöông trình :
α : Ax + By + Cz + D = 0 β : A’x + B’y + C’z + D’ = 0
1. Goùc giöõa hai ñöôøng thaúng d vaø d’ :
cos
' ' '
' ' '
ϕ =
+ +
+ + + +
aa bb cc
a b c a b c2 2 2 2 2 2
2. Goùc giöõa hai maët phaúng α vaø β :
8
cos
' '
' '
ϕ =
+ +
+ + + +
AA BB CC'
A B C A B C'2 2 2 2 2 2
3. Goùc giöõa ñöôøng thaúng d vaø maët phaúng α :
sinϕ =
+ +
+ + + +
Aa Bb Cc
A B C a b c2 2 2 2 2 2
Chuù yù : - d ⊥ d’ ⇔ aa’ + bb’ + cc’ = 0
- α ⊥ β ⇔ AA’ + BB’ + CC’ = 0
- d song song (hoaëc naèm treân) maët phaúng α ⇔ aA + bB + cC = 0
Vaán ñeà 7
VÒ TRÍ TÖÔNG ÑOÁI CUÛA HAI MAËT PHAÚNG
Cho hai maët phaúng α vaø β coù phöông trình :
α : A1x + B1y + C1z + D1 = 0 β : A2x + B2y + C2z + D2 = 0
Goïi n A laàn löôït laø phaùp vectô cuûa 2 maët phaúng treân vaø M laø moät ñieåm
treân maët phaúng α.
B C n A B C1 1 1 1 2 2 2
→ →
= =( , , ), ( , , 2 )
- α caét β ⇔ vaø khoâng cuøng phöông.n1
→
n2
→
- α song song β ⇔ n vaø n cuøng phöông
M
1 2
→ →
∉
⎧
⎨
⎪
⎩⎪ β
- α truøng β ⇔ n vaø n cuøng phöông
M
1 2
→ →
∈
⎧
⎨
⎪
⎩⎪ β
Neáu A2, B2, C2, D2 ≠ 0 thì ta coù caùch khaùc :
- α caét β ⇔ A1 : B1 : C1 ≠ A2 : B2 : C2
- α song song β ⇔
A
A
B
B
C
C
D
D
1
2
1
2
1
2
1
2
= = ≠
- α truøng β ⇔
A
A
B
B
C
C
D
D
1
2
1
2
1
2
1
2
= = =
Vaán ñeà 8
VÒ TRÍ TÖÔNG ÑOÁI CUÛA 2 ÑÖÔØNG THAÚNG
- Caùch 1 : Xeùt heä phöông trình toïa ñoä giao ñieåm cuûa hai ñöôøng thaúng d1 vaø d2.
+ Heä coù moät nghieäm duy nhaát : d1 caét d2.
+ Heä coù voâ soá nghieäm : d1 vaø d2 truøng nhau.
+ Heä voâ nghieäm :
cuøng phöông : d1 // d2.a vaøad d1
→ →
2
2
khoâng cuøng phöông : d1 vaø d2 cheùo nhau.a vaøad d1
→ →
- Caùch 2 :
+ Tìm vectô chæ phöông a cuûa d1 vaø d2.ad d1 2
→ →
,
+ Tìm ñieåm A ∈ d1 vaø B ∈ d2.
a) a v cuøng phöôngaøad d1
→ →
2
A d d d
A d d d
∈ ≡
∉
2 1 2
2 1 2
:
: / /
9
b) a v khoâng cuøng phöông ta coù:aøad d1
→ →
2
0
0
i) neáu thì d1,d2 caét nhau.1 2
, .d da a AB⎡ ⎤ =⎣ ⎦
ii) neáu thì d1,d2 cheùo nhau.1 2
, .d da a AB⎡ ⎤ ≠⎣ ⎦
Vaán ñeà 9
VÒ TRÍ TÖÔNG ÑOÁI GIÖÕA ÑÖÔØNG THAÚNG VAØ MAËT PHAÚNG
- Caùch 1 :
Xeùt heä phöông trình toïa ñoä giao ñieåm cuûa ñöôøng thaúng d vaø maët phaúng α.
+ Heä voâ nghieäm : d // α.
+ Heä coù nghieäm duy nhaát : d caét α
+ Heä voâ soá nghieäm : d ⊂ α
- Caùch 2 :
Tìm vectô chæ phöông cuûa d, phaùp vectô cuûa α vaø tìm ñieåm A ∈ d.a
→
n
→
+ a ≠ 0 ( khoâng vuoâng goùc ) : d caét α.n
→ →
. a
→
n
→
+ a = 0 ( )n
→ →
. a n
→ →
⊥
A d
A d
∉
∈ ⊂
α α
α α
: / /
:
Ví duï 1:
Laäp phöông trình maët phaúng chöùa ñöôøng thaúng (D)
2 0
3 2 3
x z
x y z
− =⎧
⎨
− + − =⎩ 0
vaø vuoâng goùc vôùi maët phaúng (P) : x – 2y + z + 5 = 0
Giaûi
Phöông trình tham soá cuûa (D) vieát
2
7 3
2 2
x t
y t
z t
=⎧
⎪
⎪
= −⎨
⎪
=⎪⎩
Maët phaúng (Q) chöùa (D) vaø vuoâng goùc (P) seõ ñi qua ñieåm
M( 0,
3
2
− , 0 ∈ (D) vaø coù caëp vectô chæ phöông laø a) = ( 2,
7
2
, 1 (vectô chæ phöông cuûa (D) vaø
= (1, –2, 1) (phaùp vectô cuûa (P)).
)
n
Do ñoù, moät phaùp veùctô cuûa ( Q) laø 1
2 1 1 2
1 1
2 ; ;7 7
1 21 2
2 2
n
⎛ − − ⎞
⎜ ⎟= =
⎜ ⎟⎜ ⎟
⎝ ⎠
= (– 11, 2, 15)
10
Vaäy phöông trình (Q) vieát
–11x + 2 ( y
3
2
+ ) + 15z = 0 11x – 2y - 15 z – 3 = 0.⇔
Caùch khaùc:
Pt maët phaúng (Q) chöùa (D) vaø vuoâng goùc (P) coù daïng:
x-2z = 0 (loaïi) hay m(x-2z) +3x -2y+z -3= 0.
Vaäy pt (Q) coù daïng: (m+3)x –2y +(1 –2m)z – 3 = 0.
(Q) vuoâng goùc vôùi (P) neân ta coù: m + 3 + 4 + 1- 2 m= 0
⇒ m = 8.
Vaäy pt mp (Q) laø: 11x – 2y - 15 z – 3 = 0.
Ví duï 2:
Xaùc ñònh caùc tham soá m vaø n ñeå maët phaúng 5x + ny + 4z + m = 0 thuoäc chuøm maët phaúng coù
phöông trình :
(3x – 7y + z – 3) + β (x – 9y – 2z + 5) = 0α
Giaûi
Chuøm maët phaúng coù phöông trình
(3x – 7y + z – 3) + β (x – 9y – 2z + 5) = 0α
chöùa ñöôøng thaúng (D) coù phöông trình :
3 7 3
9 2 5
x y z
x y z
− + − =⎧
⎨
− − + =⎩
0
0
Ñeå maët phaúng (P) : 5x + ny + 4z + m = 0 thuoäc chuøm maët phaúng treân thì (P) chöùa (D) nghóa laø
chöùa 2 ñieåm A
1 18
,0,
7 7
⎛ ⎞
⎜ ⎟
⎝ ⎠
, B
31 9
0
10 10
, ,
⎛
⎜
⎝ ⎠
⎞
⎟ ∈ (D). Ñieàu kieän ñeå (P) chöùa A, B thì m, n thoûa heä phöông
trình :
5 18
4 0
7 7
31 9
5 0
10 10
. m
. .n m
⎧
+ + =⎪⎪
⎨
⎪ + + =
⎪⎩
⇒
11
5
m
n
= −⎧
⎨
= −⎩
Ví duï 3: ( ÑH KHOÁI A-2002) Trong khoâng gian vôùi heä toïa ñoä Ñeâcac vuoâng goùc Oxyz cho hai ñöôøng
thaúng:
Δ1 : vaø Δ2 :
⎩
⎨
⎧
=+−+
=−+−
04z2y2x
04zy2x
⎪
⎩
⎪
⎨
⎧
+=
+=
+=
t21z
t2y
t1x
a) Vieát phöông trình maët phaúng (P) chöùa ñöôøng thaúng Δ1 vaø song song vôùi ñöôøng thaúng Δ2.
11
b) Cho ñieåm M (2; 1; 4). Tìm toïa ñoä ñieåm H thuoäc ñöôøng thaúng Δ2 sao cho ñoaïn thaúng MH coù ñoä
daøi nhoû nhaát.
BAØI GIAÛI:
a) (P) chöùa Δ1 vaø // Δ2
1
aΔ = (2, 3, 4); 2aΔ = (1, 1, 2); Δ1 qua M (0, −2, 0)
Maët phaúng (P) coù pvt [ ]21
a,a ΔΔ =(2, 0, −1)
(P) : 2x – z = 0
b) M (2, 1, 4); H ∈ Δ2; MH min ⇔ MH ⊥ Δ2
C1 : Goïi (Q) laø maët phaúng qua M vaø vuoâng goùc vôùi Δ2.
Pt (Q) : x + y + 2z – 11 = 0; {H} = (Q) ∩ Δ2 ⇒ H (2, 3, 3)
C2 : MH = (−1 + t, 1 + t, −3 + 2t), vôùi H ∈ Δ2
Do MH . 2aΔ = 0 ⇒ t = 1. Vaäy ñieåm H (2, 3, 3).
Ví duï 4: ( ÑH KHOÁI B-2002) Cho hình laäp phöông ABCDA1B1C1D1 coù caïnh baèng a.
a) Tính theo a khoaûng caùch giöõa hai ñöôøng thaúng A1B vaø B1D .
b) Goïi M,N,P laàn löôït laø caùc trung ñieåm cuûa caùc caïnh BB1, CD,A1D1 .Tính goùc giöõa hai ñöôøng
thaúng MP vaø C1N .
BAØI GIAÛI:
Choïn heä truïc toïa ñoä Axyz sao cho ta coù :
A (0, 0, 0); A1 (0, 0, a); B (a, 0, 0); B1 (a, 0, a)
C (a, a, 0); C1 (a, a, a); D (0, a, 0); D1 (0, a, a)
Suy ra M (a, 0, 2
a ); N ( 2
a , a, 0); P (0, 2
a , a)
a) BA1 = (a, 0, −a) DB1 = (−a, a, −a)
Goïi (P) laø mp qua B1D vaø (P) // A1B
⇒ (P) coù phaùp vectô n = (1, 2, 1)
⇒ Pt (P) : x + 2y + z – 2a = 0
⇒ d (A1B, B1D) = d (B, (P)) =
6
a
b) MP = (−a, 2
a , 2
a ) . NC1 = (− 2
a , 0, −a)
Ta coù : MP . NC1 = 0 ⇒ MP ⊥ C1N.
Vaäy goùc giöõa MP vaø C1N laø 900
.
Ví duï5 ( ÑH KHOÁI D-2002): Trong khoâng gian vôùi heä toïa ñoä Ñeâcac vuoâng goùc Oxyz, cho maët phaúng
(P): 2x – y + 2 = 0 vaø ñöôøng thaúng dm :
(m laø tham soá)
⎩
⎨
⎧
=++++
=−+−++
02m4z)1m2(mx
01my)m1(x)1m2(
Xaùc ñònh m ñeå ñöôøng thaúng dm song song vôùi maët phaúng (P).
BAØI GIAÛI:
1 vectô chæ phöông cuûa (dm) laø :
a = (−2m2
+ m + 1, −(2m +1)2
, - m(1 – m))
1 pvt cuûa (P) laø n = (2, −1, 0)
ycbt ⇔ a . n = 0 ⇔ −4m2
+ 2m + 2 + (4m2
+ 4m + 1) = 0
⇔ 6m + 3 = 0 ⇔ m =
2
1
−
12
Ví duï 6 ( ÑH KHOÁI A-2003): Trong khoâng gian vôùi heä toïa ñoä Ñeâcac vuoâng goùc Oxyz cho hình hoäp
chöõ nhaät ABCD.A’B’C’D’ coù A truøng vôùi goác toïa ñoä, B(a;0;0),
D(0; a; 0), A’(0; 0; b) ( a > 0, b > 0). Goïi M laø trung ñieåm CC’.
a. Tính theå tích khoái töù dieän BDA’M theo a vaø b.
b. Xaùc ñònh tyû soá
a
b
ñeå hai maët phaúng (A’BD) vaø (MBD) vuoâng goùc vôùi nhau.
BAØI GIAÛI: A (0, 0, 0); B (a, 0, 0); C (a, a, 0); D (0, a, 0)
A’ (0, 0, b); C’ (a, a, b); M (a, a,
b
2
)
a) ; ;= −BD ( a,a,0) = −BA' ( a,0,b) =
b
BM (0,a, )
2
⇒ ⎡ ⎤ =⎣ ⎦
2
BD,BA' (ab,ab,a )
⇒ V= ⎡ ⎤ = +⎣ ⎦
2
21 1 a b
2
BD,BA' .BM (a b )
6 6
= =
2 2
3a b a b
12 4
(ñvtt)
b) (A’BD) coù vectô phaùp tuyeán hay⎡ ⎤ =⎣ ⎦
2
BD,BA' (ab,ab,a ) =n (b,b,a)
(MBD) coù vectô phaùp tuyeán
⎡ ⎤ = −⎣ ⎦
2ab ab
BD,BM ( , , a )
2 2
hay = −m (b,b, 2a)
Ta coù : (A’BD) ⊥ (MBD) ⇔ =m . n 0
⇔ b2
+ b2
– 2a2
= 0 ⇔ a = b (a, b > 0) ⇔ =
a
1
b
Ví duï 7 ( ÑH KHOÁI B-2003): Trong khoâng gian vôùi heä toïa ñoä Ñeâcac vuoâng goùc Oxyz cho hai ñieåm
A(2;0;0), B(0;0;8) vaø ñieåm C sao cho (0;6;0)AC = . Tính khoaûng caùch töø trung ñieåm I cuûa BC ñeán
ñöôøng thaúng OA.
BAØI GIAÛI: A (2; 0; 0); B (0; 0; 8).
= (0; 6; 0) ⇔ ⇔ C (2; 6; 0). I trung ñieåm BC ⇒ I (1; 3; 4)AC
=⎧
⎪
=⎨
⎪ =⎩
C
C
C
x 2
y
z 0
6
Pt tham soá OA :
=⎧
⎪
=⎨
⎪ =⎩
x t
y 0
z 0
(α) qua I ⊥ = (2; 0; 0) : 2(x – 1) = 0 ⇔ x – 1 = 0OA
Toïa ñoä {H} = OA ∩ (α) thoûa :
⇔
= = =⎧
⎨
− =⎩
x t,y 0,z 0
x 1 0
=⎧
⎪
=⎨
⎪ =⎩
x 1
y 0
z 0
. Vaäy H (1; 0; 0).
d(I, OA) = IH = − + − + −2 2
(1 1) (0 3) (0 4)2
= 5.
Ví duï 8 ( ÑH KHOÁI D-2003): Trong khoâng gian vôùi heä toïa ñoä Ñeâcac vuoâng goùc Oxyz cho ñöôøng
thaúng
3 2
:
1 0
x ky z
dk kx y z
+ − + =
− + + =
⎧
⎨
⎩
0
Tìm k ñeå ñöôøng thaúng dk vuoâng goùc vôùi maët phaúng
(P): x – y – 2z + 5 =0
BAØI GIAÛI: 1n = (1, 3k, −1); = (k, −1, 1)2n
13
= (3k – 1, −k – 1, −1 – 3k2
)da
= (1, −1, 2)Pn −
dk ⊥ (P) ⇔ cuøng phöôngda Pn
⇔
− − − − −
= =
− −
2
3k 1 k 1 1 3k
1 1 2
⇔
=⎧
⎪
⎨
= ∨ = −⎪⎩
k 1
1
k 1 k
3
⇔ k = 1
Ví duï9 ( ÑH KHOÁI A-2004): Trong khoâng gian vôùi heä toïa ñoä Oxyz cho hình choùp S.ABCD coù ñaùy
ABCD laø hình thoi, AC caét BD taïi goác toïa ñoä O. Bieát A(2; 0; 0), B(0; 1; 0), S(0; 0; 2 2 ). Goïi M laø
trung ñieåm cuûa caïnh SC.
a) Tính goùc vaø khoaûng caùch hai ñöôøng thaúng SA, BM.
b) Giaû söû maët phaúng (ABM) caét ñöôøng thaúng SD taïi ñieåm N. Tính theå tích khoái choùp S.ABMN.
BAØI GIAÛI: Caùch 1:
S M C
N
H
D O B
A
GT ⇒ SO ⊥ (ABCD); SA = SC = 2 3
a) Ta coù OM // SA ⇒ Goùc (SA, MB) laø OMB
OB ⊥ (SAC) ⇒ OB ⊥ OM ΔOBM coù tg
OB
OMB
OM
=
⇒
1
tgOMB
3
= ⇒ OMB =300
Veõ OH ⊥ SA ⇒ OH ⊥ OM vaø OH ⊥ OB ⇒ OH ⊥ (OMB)
Vì SA // OM ⇒ SA // (OMB)
⇒ d (SA, MB) = d(H, (OMB)) = OH =
2 6
3
.
b) (ABM) ∩ SD = N ⇒ N laø trung ñieåm SD
Ta coù: SBMN
SBCD
V SM SN
.
V SC SD
=
1
4
= ⇒ VSMNB = SBCD SABCD
1 1
V V
4 8
=
Töông töï: VSABN = SABCD
1
V
4
Vaäy: VSABMN = VSMNB + VSABN = SABCD
3
V
8
=
3 1
(ñvtt)
1 1
. . AC.BD.SO .4.2.2 2 2
8 3 2 16
= =
Caùch 2: a) O laø trung ñieåm BD ⇒ D (0; −1; 0)
O laø trung ñieåm AC ⇒ C (−2; 0; 0)
M laø trung ñieåm SC ⇒ M ( 1;0; 2)−
14
=(2; 0;-SA 2 2 ); BM ( 1; 1; 2)= − −
Goïi ϕ laø goùc nhoïn taïo bôûi SA vaø BM
cosϕ =
− + −
+ + +
2 0 4
4 8 1 1 2
=
3
2
⇒ ϕ = 300
Goïi (α) laø mp chöùa SA vaø // BM
⇒ PT (α) : 2x z 2 2 0+ − =
Ta coù d(SA, BM) = d(B, α) =
2 6
3
.
b) Pt mp(ABM): 2x 2 2y 3z 2 2 0+ + − =
Pt tham soá SD:
⎧ =
⎪
= − +⎨
⎪
=⎩
x 0
y 1
z 2 2t
t (t ∈ R).
N laø giao ñieåm cuûa SD vaø mp (ABM) ⇒ N
1
(0; ; 2)
2
−
BS (0; 1;2 2)= − ; BA (2; 1;0)= −
3
BN (0; ; 2)
2
= − ; BM ( 1; 1; 2)= − −
BS,BN (2 2;0;0)⎡ ⎤ =⎣ ⎦ ; BS,BN .BA 4 2⎡ ⎤ =⎣ ⎦
BS.BN .BM 2 2⎡ ⎤ = −⎣ ⎦
VSABMN= VSABN + VSBNM =
1 1
.4 2 .2 2 2
6 6
+ = (ñvtt)
Ví duï 10 ( ÑH KHOÁI D -2004): Trong khoâng gian vôùi heä toïa ñoä Oxyz cho hình laêng truï ñöùng
ABCA1B1C1. Bieát A(a;0;0); B(−a;0;0); C (0; 1; 0); B1(−a; 0; b)
a > 0, b > 0.
a) Tính khoaûng caùch giöõa 2 ñöôøng thaúng B1C vaø AC1 theo a, b.
b) Cho a, b thay ñoåi nhöng luoân thoûa maõn a + b = 4. Tìm a, b ñeå khoaûng caùch giöõa 2 ñöôøng thaúng
B1C vaø AC1 lôùn nhaát.
BAØI GIAÛI: a) C1 (0; 1; b)
Goïi (α) laø maët phaúng chöùa B1C vaø song song vôùi AC1
;1B C (a;1; b)= − 1C A (a; 1; b)= − −
Suy ra: 1 1B C,C A ( 2b;0; 2a)⎡ ⎤ = − −⎣ ⎦
Suy ra ptrình (α): .− + − + − =b(x 0) 0(y 1) a(z 0) 0
⇔ bx + az = 0.
Ta coù: d=d(B1C, AC1)=d(A, α)=
2 2 2 2
ab ab
a b a b
=
+ +
.
b) Caùch 1:
Ta coù: d=
2 2
ab ab ab
2ab 2a b
≤ =
+
a b 4
2
2 2 2 2
+
≤ = =
Max d ⇔ d = 2 ⇔ ⇔ a = b = 2
a b
a b 4
a 0,b 0
=⎧
⎪
+ =⎨
⎪ > >⎩
15
Caùch 2: d =
ab
16 2ab−
, ñaët x = ab, ñk 0 < x ≤ 4.
vì x = ab
2
a b
4
2
+⎛ ⎞
≤ =⎜ ⎟
⎝ ⎠
Xeùt f(x) =
x
16 2x−
f’(x) =
3
16 x
(16 2x)
−
−
> 0 ∀x ∈ (0; 4]
⇒ d ñaït max khi x = ab = 4 ⇒ a = b = 2 (vì a + b = 4)
Ví duï 11 ( ÑH KHOÁI B-2004): Trong khoâng gian vôùi heä toïa ñoä Oxyz cho ñieåm
3 2
: 1
1 4
x t
d y t
z t
= − +⎧
⎪
= −⎨
⎪ = − +⎩
A (-4; -2; 4) vaø ñöôøng thaúng
Vieát phöông trình ñöôøng thaúng Δ ñi qua ñieåm A, caét vaø vuoâng goùc vôùi ñöôøng thaúng d.
BAØI GIAÛI: Caùch 1: A (−4; −2; 4)
(d) :
⎪
⎨
x 3 2t
y 1 t
z 1 4t
= − +⎧
= −
⎪ = − +⎩
Laáy M (−3+2t; 1 – t; −1 + 4t) ∈ (d)
⇒ = (1 + 2t; 3 – t; −5 + 4t)AM
Ta coù: AM ⊥ (d) ⇔ (vôùidAM. a 0= da =(2; −1; 4)).
⇔ 2 + 4t – 3 + t – 20 + 16t = 0 ⇔ 21t = 21 ⇔ t = 1.
Vaäy ñöôøng thaúng caàn tìm laø ñt AM qua A coù VTCP AM =(3;2;−1)
⇒ phöông trình (Δ) :
x+4 y 2 z 4
3 2
+ −
= =
−1
.
Caùch 2: Goïi (α) laø mp qua A chöùa d ,Goïi (β) laø mp qua A
vaø ⊥ d ⇒ d qua B (−3; 1; −1); = (2; −1; 4)da
(α) qua A (−4; −2; 4) (α) coù 1 caëp VTCP :
⇒da (2; 1;4
AB (1;3; 5)
⎧ = −⎪
⎨
= −⎪⎩
)
( )n α = (−7; 14; 7) = −7(1; −2; −1)
Pt mp (α) : x – 2y – z + 4 = 0
( ) d
( ) qua A (-4; -2; 4)
( ) (d) n a (2; 1;4)β
β
β
⎧⎪
⎨
⊥ → = = −⎪⎩
Pt (β) : 2x – y + 4z – 10 = 0 Pt (Δ) :
x 2y z 4 0
2x y 4z 10 0
− − + =⎧
⎨
− + − =⎩
Ví duï 12 ( ÑH KHOÁI A-2005): Trong khoâng gian vôùi heä toïa ñoä Oxyz cho ñöôøng thaúng:
d :
x 1 y 3 z 3
1 2 1
− + −
= =
−
vaø maët phaúng (P) : 2x + y – 2z + 9 = 0
a)Tìm toïa ñoä ñieåm I thuoäc d sao cho khoaûng caùch töø I ñeán maët phaúng (P) baèng 2.
b)Tìm toïa ñoä giao ñieåm A cuûa ñöôøng thaúng d vaø maët phaúng (P). Vieát phöông trình tham soá cuûa ñöôøng thaúng
Δ naèm trong maët phaúng (P), bieát Δ ñi qua A vaø vuoâng goùc vôùi d.
BAØI GIAÛI: a) Phöông trình tham soá cuûa d :
⎧
⎪
⎨
⎪
⎩
= −
= − +
= +
x 1 t
y 3 2t
z 3 t
(t∈ R)
16
I ∈ d ⇔ I (1–t ; –3+2t ; 3+t)
Ta coù : d (I, (P)) = 2 ⇔
− − + − − +
=
+ +
| 2 2t 3 2t 6 2t 9 |
2
4 1 4
⇔ Suy ra : I (3 ; -7 ; 1) hay I (-3 ; 5 ; 7).
t
|1 t | 3
t 4
= −⎡
− = ⇔ ⎢ =⎣
2
b) Theá phöông trình d vaøo phöông trình (P) ta ñöôïc t = 1.
Theá t = 1 vaøo phöông trình d, ta ñöôïc x = 0; y = -1; z = 4
Suy ra A (0; -1 ; 4)
Vectô chæ phöông cuûa d : = −a ( 1;2;1)
Vectô phaùp tuyeán cuûa (P): = −n (2;1; 2)
Suy ra vectô chæ phöông cuûa Δ : = − −[a,n] ( 5; 0; 5) hay (1; 0; 1)
Maët khaùc Δ ñi qua A neân phöông trình tham soá cuûa Δ laø :
⎧
⎪
⎨
⎪
⎩
=
= −
= +
x t'
y 1
z 4 t'
(t’∈ R)
Ví duï 13 ( ÑH KHOÁI B-2005): Trong khoâng gian vôùi heä toïa ñoä Oxyz cho hình laêng truï ñöùng ABC.A1B1C1 vôùi
A(0; -3; 0), B(4; 0; 0), C(0; 3; 0), B1(4; 0; 4).
a) Tìm toïa ñoä caùc ñænh A1, C1. Vieát phöông trình maët caàu coù taâm laø A vaø tieáp xuùc vôùi maët phaúng
(BCC1B1).
b) Goïi M laø trung ñieåm cuûa A1B1 . Vieát phöông trình maët phaúng (P) ñi qua hai ñieåm A, M vaø song song vôùi
BC1. Maët phaúng (P) caét ñöôøng thaúng A1C1 taïi ñieåm N. Tính ñoä daøi MN.
BAØI GIAÛI: a) Hình chieáu cuûa A1 xuoáng mp (Oxy) laø A ⇒ A1(0; -3; 4)
Hình chieáu cuûa C1 xuoáng mp (Oxy) laø C ⇒ C1(0; 3; 4)
Caëp veùc tô chæ phöông cuûa (BCC1B1) laø : BC ( 4;3;0)= −
1BB (0;0;4)=
Suy ra veùc tô phaùp tuyeán cuûa (BCC1B1) laø :
= (12; 16; 0) hay = (3; 4; 0)1n BC,BB⎡= ⎣
⎤
⎦ m
Maët khaùc (BCC1B1) qua B neân coù phöông trình:
3(x – 4) + 4y + 0z = 0 ⇔ 3x + 4y – 12 = 0
Baùn kính maët caàu laø :
R = d (A, (BCC1B1)) =
0 12 12 24
59 16
− −
=
+
Suy ra phöông trình maët caàu laø : x2
+ (y + 3)2
+ z2
=
576
25
b) M laø trung ñieåm cuûa A1B1 ⇒ M (2;
3
2
− ; 4)
Mp (P) coù caëp veùc tô chæ phöông
3
AM (2; ;4)
2
= vaø 1BC ( 4;3;4)= − ⇒ veùc tô phaùp tuyeán cuûa mp (P):
= = (−6; −24; 12) hay (1; 4; −2)Pn 1AM;BC⎡
⎣
⎤
⎦
Maët khaùc (P) ñi qua A neân coù phöông trình : x + 4(y + 3) – 2z = 0
⇔ x + 4y – 2z + 12 = 0
A1C1 ñi qua A1 vaø coù veùc tô chæ phöông 1 1A C = (0; 6;0) hay (0; 1; 0)
neân coù phöông trình : (t ∈ R)
x 0
y 3
z 4
=⎧
⎪
= − +⎨
⎪ =⎩
t
17
Theá phöông trình A1C1 vaøo phöông trình (P) ta ñöôïc t = 2
Theá t = 2 vaøo phöông trình (A1C1) ta ñöôïc x = 0, y = −1, z = 4
⇒ N (0; −1; 4)
vaø MN = 2 2 23 1
(0 2) ( 1 ) (4 4)
2 2
− + − + + − =
7
Ví duï 14 ( ÑH KHOÁI D-2005): Trong khoâng gian vôùi heä toïa ñoä Oxyz cho hai ñöôøng thaúng :
d1 :
x 1 y 2 z 1
3 1 2
− + +
= =
−
vaø d2:
x y z 2 0
x 3y 12 0
+ − − =⎧
⎨
+ − =⎩
a) Chöùng minh raèng d1 vaø d2 song song vôùi nhau. Vieát phöông trình maët phaúng (P) chöùa caû hai ñöôøng
thaúng d1 vaø d2.
b) Maët phaúng toïa ñoä Oxz caét hai ñöôøng thaúng d1, d2 laàn löôït taïi caùc ñieåm A, B. Tính dieän tích tam giaùc
OAB (O laø goác toïa ñoä).
aBAØI GIAÛI: a) d1 qua N (1; −2; −1) vaø coù 1 vectô chæ phöông laø =(3; −1; 2)
bd2 qua B (12; 0; 10) vaø coù 1 vectô chæ phöông laø =(3; −1; 2)
Ta coù : = vaø = (11, 2, 11) khoâng cuøng phöông vôùia b NB a .
Vaäy d1 // d2
Mp (P) qua N vaø coù phaùp vectô : =[n a NB, ] = (−15; −11; 17)
Phöông trình (P) laø: −15(x–1) – 11(y+2) + 17(z+1) = 0
⇔ 15x + 11y – 17z – 10 = 0
b) A(−5, 0, −5); B (12, 0, 10) ⇒ = (0, −10, 0)OA,OB⎡
⎣
⎤
⎦
1
OA,OB
2
⎡
⎣⇒ Dieän tích (ΔOAB) = ⎤
⎦ = 5 (ñvdt).
* * *
18
CHUYEÂN ÑEÀ 10: HÌNH CAÀU
TOÙM TAÉT COÂNG THÖÙC
(1) Phöông trình maët caàu
1) Phöông trình maët caàu (S) coù taâm I(a, b, c) baùn kính R laø
(x – a)2
+ (y – b)2
+ (z – c)2
= R2
2) Daïng toång quaùt cuûa phöông trình maët caàu laø
x2
+ y2
+ z2
– 2ax – 2by – 2cz + d = 0
seõ coù taâm I(a, b, c) baùn kính R = 2 2 2
a b c d+ + − neáu ta coù ñieàu kieän
a2
+ b2
+ c2
– d > 0
3) Ñieàu kieän tieáp xuùc giöõa maët phaúng (P) vaø maët caàu (S) coù taâm I baùn kính R laø khoaûng
caùch töø I ñeán (P) baèng baùn kính R.
Ví duï 1:
Laäp phöông trình maët caàu coù taâm I(2, 3, –1) caét ñöôøng thaúng (d)
5 4 3 20
3 4 8 0
x y z
x y z
− + + =⎧
⎨
− + − =⎩
0
taïi hai ñieåm A vaø B sao cho AB = 16
Giaûi
Goïi (P) laø maët phaúng qua I vaø vuoâng goùc ñöôøng thaúng (d). Ta coù phöông trình tham soá ñöôøng
(d) laø
14
1 2
2 2
x t
y t
z t
= −⎧
⎪
⎪
= −⎨
⎪
= −⎪⎩
5
Goïi (P) laø maët phaúng qua I(2, 3, –1) vaø vuoâng goùc ñöôøng thaúng (d) neân coù phaùp vectô laø a =
1
1, , 1
2
⎛
−⎜
⎝ ⎠
⎞
⎟ . Vaäy phöông trình (P) vieát
(x – 2) +
1
2
(y – 3) - (z + 1) = 0 ⇔ 2x + y – 2z – 9 = 0
Giao ñieåm K giöõa (d) vaø (P) coù toïa ñoä ( t – 14,
1
2
t –
25
2
, –t )
thoûa phöông trình (P). Vaäy ta coù
1
2(t – 14) + (
1
2
t –
25
2
) +2t – 9 = 0
Suy ra t = 11. Vaäy ta coù K (–3, –7, –11).
Khoaûng caùch töø I ñeán (d) laø IK = 25 100 100+ + = 15
Do ñoù baùn kính maët caàu laø R =
2
2
4
AB
IK + = 225 64+
Neân phöông trình maët caàu vieát laø :
(x – 2)2
+ (y – 3)2
+ (z + 1)2
= 289
Ví duï 2:
Laäp phöông trình maët caàu coù taâm thuoäc ñöôøng thaúng (d)
2 4 7 0
4 5 14 0
x y z
x y z
+ − − =⎧
⎨
+ + − =⎩
vaø tieáp xuùc vôùi hai maët phaúng coù phöông trình
(P) : x + 2y – 2z – 2 = 0 ; (Q) : x + 2y – 2z + 4 = 0
Giaûi
Ta coù (P) // (Q) neân khi goïi A, B laø giao ñieåm cuûa (d) vôùi (P) vaø (Q) thì taâm I maët caàu tieáp xuùc
vôùi (P) vaø (Q) phaûi laø trung ñieåm ñoaïn AB vaø baùn kính maët caàu baèng khoaûng caùch töø I ñeán (P).
Ta coù toïa ñoä A laø nghieäm cuûa heä
A(2, 1, 1)
2 4 7 0
4 5 14
2 2 2 0
x y z
x y z
x y z
+ − − =⎧
⎪
+ + − =⎨
⎪ + − − =⎩
0
0
⇒
Ta coù toïa ñoä B laø nghieäm cuûa heä
B(–4, 5, 5)
2 4 7 0
4 5 14
2 2 4 0
x y z
x y z
x y z
+ − − =⎧
⎪
+ + − =⎨
⎪ + − + =⎩
⇒
Vaäy taâm maët caàu laø I(–1, 3, 3) vaø baùn kính R = 1
Neân phöông trình maët caàu vieát thaønh
(x + 1)2
+ (y – 3)2
+ (z – 3)2
= 1.
Ví duï 3 ( ÑH KHOÁI D –2004) Trong khoâng gian vôùi heä toïa ñoä Oxyz cho 3 ñieåm
A (2; 0; 1); B(1;0;0); C (1; 1; 1) vaø maët phaúng (P): x + y + z – 2 = 0. Vieát phöông trình maët caàu ñi qua 3
ñieåm A, B, C vaø coù taâm thuoäc maët phaúng (P).
Giaûi
2
Caùch 1: x2
+ y2
+ z2
+ 2ax + 2by + 2cz + d = 0. Maët caàu qua A, B, C vaø coù taâm thuoäc maët phaúng (P)
neân ta coù:
⇔
+ + = −⎧
⎪ + = −⎪
⎨
+ + + = −⎪
⎪ + + = −⎩
4a 2c d 5
2a d 1
2a 2b 2c d 3
a b c 2
= −⎧
⎪ =⎪
⎨
= −⎪
⎪ =⎩
a 1
b 0
c 1
d 1
⇔ x2
+ y2
+ z2
– 2x – 2z + 1 = 0
Caùch 2: Goïi I(x; y; z) laø taâm maët caàu
Giaû thieát cho:
2 2
IA IB IC
I (P)
⎧ = =⎪
⎨
∈⎪⎩
2
2
⇔
⎧ − + + − = − + +
⎪⎪
− + + = − + − + −⎨
⎪ + + − =
⎪⎩
2 2 2 2 2 2
2 2 2 2 2
(x 2) y (z 1) (x 1) y z
(x 1) y z (x 1) (y 1) (z 1)
x y z 2 0
⇔ ⇔ ⇒ I (1; 0; 1)
+ − =⎧
⎪
+ =⎨
⎪ + + − =⎩
2x 2z 4 0
y z 1
x y z 2 0
x 1
y 0
z 1
=⎧
⎪
=⎨
⎪ =⎩
Baùn kính R = IB = 1
Suy ra phöông trình maët caàu: (x – 1)2
+ y2
+ (z –1)2
=1
Ví duï4 ( Ñeà Döï Tröõ KHOÁI D -2002) Trong khoâng gian vôùi heä toïa ñoä Ñeàcac vuoâng goùc Oxyz cho ñöôøng
thaúng d : vaø maët caàu
⎩
⎨
⎧
=−−+
=+−−
04z2y2x
01zy2x2
(S) : x2
+ y2
+ z2
+ 4x – 6y + m = 0. Tìm m ñeå ñöôøng thaúng d caét maët caàu (S) taïi hai ñieåm M, N sao cho khoaûng
caùch giöõa hai ñieåm ñoù baèng 9.
Giaûi
Phöông trình maët caàu (S) : (x + 2)2
+ (y – 3)2
+ z2
= 13 – m
ÑK : m < 13
(S) coù taâm I(−2; 3; 0), R = 13 m− .
Vì MN = 9 ⇒ HM = HN =
9
2
(IH ⊥ MN)
(d) cho x = 0 ⇒ ⇒
2y z 1 0
2y 2z 4 0
− − + =⎧
⎨
− − =⎩
y 1
z 1
=⎧
⎨
= −⎩
⇒ A(0; 1; −1)
(d) coù ⇒ = 3(2; 1; 2)
1
2
n (2, 2, 1)
n (1,2, 2)
→
→
⎡
= − −⎢
⎢
= −⎢⎣
a
→
AI
⎯→
= (−2; 2; 1), [ AI
⎯→
, ] = (9; 18; − 18) = 9(1; 2; − 2)a
→
IH = d(I, d) =
⎯→ →
→
⏐ ⏐ + +
= =
+ +⏐ ⏐
[ AI ,a] 9 1 4 4
3
3 4 1 4a
.
Δ vuoâng IHN ta coù :
IM2
= IH2
+ HN2
⇔13 – m = 9 +
81 117
4 4
=
3
⇔ m =
65
4
− .
Ví duï 5 ( ÑEÀ DÖÏ TRÖÕ KHOÁI D -2003) Trong khoâng gian vôùi heä toïa ñoä Ñeàcac vuoâng goùc Oxyz cho maët phaúng
(P) : 2x + 2y + z – m2
– 3m = 0 (m laø tham soá) vaø maët caàu
(S) : (x – 1)2
+ (y + 1)2
+ (z – 1)2
= 9
Tìm m ñeå maët phaúng (P) tieáp xuùc vôùi maët caàu (S). Vôùi m tìm ñöôïc haõy xaùc ñònh toïa ñoä tieáp ñieåm cuûa maët
phaúng (P) vaø maët caàu (S).
Giaûi
Maët caàu (S) coù taâm I(1; −1; 1), baùn kính R = 3.
Maët phaúng P tieáp xuùc vôùi (S) ⇔ d(I: P) = R
⇔ 1443m3m122 2
++=−−+−
⇔ m2
+ 3m – 1 = 9 hay m2
+ 3m – 1 = −9
⇔ m2
+ 3m – 10 = 0 hay m2
+ 3m + 8 = 0 (VN)
⇔ m = −5 hay m = 2 ⇒ (P) : 2x + 2y + z – 10 = 0
Phöông trình ñöôøng thaúng Δ qua I vaø ⊥ (P) :
x 1 2t
y 1 2
z 1 t
= +⎧
⎪
Δ = − +⎨
⎪ = +⎩
t
Theá vaøo phöông trình mp (P)
⇒ 2(1 + 2t) + 2(−1 + 2t) + 1 + t – 10 = 0 ⇒ t = 1
⇒ Tieáp ñieåm M cuûa P vaø (S) laø M(3; 1; 2).
Caùch khaùc IM2
= 9 ⇔ 4t2
+ 4t2
+ t2
= 9 ⇒ t = ± 1
⇒ M(3; 1; 2) hay M(-1; -3; 0).Vì M∈ P ⇒ M(3; 1; 2)
PHAÏM HOÀNG DANH-TRAÀN MINH QUANG –TRAÀN VAÊN TOAØN
( TRUNG TAÂM LUYEÄN THI CLC VÓNH VIEÃN )
4

More Related Content

What's hot

Bai toan tim diem tren do thi ham so
Bai toan tim diem tren do thi ham soBai toan tim diem tren do thi ham so
Bai toan tim diem tren do thi ham soVui Lên Bạn Nhé
 
Tiếp tuyến của đồ thị hàm số
Tiếp tuyến của đồ thị hàm sốTiếp tuyến của đồ thị hàm số
Tiếp tuyến của đồ thị hàm sốMinh Thắng Trần
 
Toan pt.de045.2012
Toan pt.de045.2012Toan pt.de045.2012
Toan pt.de045.2012BẢO Hí
 
Tai lieu luyen thi dai hoc de thi dh toan khoi a - nam 2013
Tai lieu luyen thi dai hoc   de thi dh toan khoi a - nam 2013Tai lieu luyen thi dai hoc   de thi dh toan khoi a - nam 2013
Tai lieu luyen thi dai hoc de thi dh toan khoi a - nam 2013Trungtâmluyệnthi Qsc
 
Toan pt.de039.2012
Toan pt.de039.2012Toan pt.de039.2012
Toan pt.de039.2012BẢO Hí
 
Toan pt.de116.2011
Toan pt.de116.2011Toan pt.de116.2011
Toan pt.de116.2011BẢO Hí
 
Dtnc29 cach ung dung loi giai coc chiu luc ngang khi hsn pbo hinh thang
Dtnc29 cach ung dung loi giai coc chiu luc ngang khi hsn pbo hinh thangDtnc29 cach ung dung loi giai coc chiu luc ngang khi hsn pbo hinh thang
Dtnc29 cach ung dung loi giai coc chiu luc ngang khi hsn pbo hinh thangrobinking277
 
Toan pt.de115.2011
Toan pt.de115.2011Toan pt.de115.2011
Toan pt.de115.2011BẢO Hí
 
Toan pt.de111.2011
Toan pt.de111.2011Toan pt.de111.2011
Toan pt.de111.2011BẢO Hí
 
Toan pt.de017.2011
Toan pt.de017.2011Toan pt.de017.2011
Toan pt.de017.2011BẢO Hí
 
Cac huongtuduy phuongphapgiai_hinhhocoxyz
Cac huongtuduy phuongphapgiai_hinhhocoxyzCac huongtuduy phuongphapgiai_hinhhocoxyz
Cac huongtuduy phuongphapgiai_hinhhocoxyzHuynh ICT
 
1.2 tuong giao_cua_do_thi_cac_ham_so
1.2 tuong giao_cua_do_thi_cac_ham_so1.2 tuong giao_cua_do_thi_cac_ham_so
1.2 tuong giao_cua_do_thi_cac_ham_sovanthuan1982
 
Toan pt.de017.2012
Toan pt.de017.2012Toan pt.de017.2012
Toan pt.de017.2012BẢO Hí
 
Toan pt.de101.2011
Toan pt.de101.2011Toan pt.de101.2011
Toan pt.de101.2011BẢO Hí
 
[Vnmath.com] de-thi-lan5-chuyen-vinhphuc-a
[Vnmath.com] de-thi-lan5-chuyen-vinhphuc-a[Vnmath.com] de-thi-lan5-chuyen-vinhphuc-a
[Vnmath.com] de-thi-lan5-chuyen-vinhphuc-aNam Hoài
 

What's hot (19)

Bai toan tim diem tren do thi ham so
Bai toan tim diem tren do thi ham soBai toan tim diem tren do thi ham so
Bai toan tim diem tren do thi ham so
 
10.khaosaths
10.khaosaths10.khaosaths
10.khaosaths
 
Tiếp tuyến của đồ thị hàm số
Tiếp tuyến của đồ thị hàm sốTiếp tuyến của đồ thị hàm số
Tiếp tuyến của đồ thị hàm số
 
Toan pt.de045.2012
Toan pt.de045.2012Toan pt.de045.2012
Toan pt.de045.2012
 
Tai lieu luyen thi dai hoc de thi dh toan khoi a - nam 2013
Tai lieu luyen thi dai hoc   de thi dh toan khoi a - nam 2013Tai lieu luyen thi dai hoc   de thi dh toan khoi a - nam 2013
Tai lieu luyen thi dai hoc de thi dh toan khoi a - nam 2013
 
Toan pt.de039.2012
Toan pt.de039.2012Toan pt.de039.2012
Toan pt.de039.2012
 
Toan pt.de116.2011
Toan pt.de116.2011Toan pt.de116.2011
Toan pt.de116.2011
 
Dtnc29 cach ung dung loi giai coc chiu luc ngang khi hsn pbo hinh thang
Dtnc29 cach ung dung loi giai coc chiu luc ngang khi hsn pbo hinh thangDtnc29 cach ung dung loi giai coc chiu luc ngang khi hsn pbo hinh thang
Dtnc29 cach ung dung loi giai coc chiu luc ngang khi hsn pbo hinh thang
 
Chude1
Chude1Chude1
Chude1
 
Basic số phức cực hay
Basic số phức cực hayBasic số phức cực hay
Basic số phức cực hay
 
Toan pt.de115.2011
Toan pt.de115.2011Toan pt.de115.2011
Toan pt.de115.2011
 
Toan pt.de111.2011
Toan pt.de111.2011Toan pt.de111.2011
Toan pt.de111.2011
 
Toan pt.de017.2011
Toan pt.de017.2011Toan pt.de017.2011
Toan pt.de017.2011
 
Cac huongtuduy phuongphapgiai_hinhhocoxyz
Cac huongtuduy phuongphapgiai_hinhhocoxyzCac huongtuduy phuongphapgiai_hinhhocoxyz
Cac huongtuduy phuongphapgiai_hinhhocoxyz
 
1.2 tuong giao_cua_do_thi_cac_ham_so
1.2 tuong giao_cua_do_thi_cac_ham_so1.2 tuong giao_cua_do_thi_cac_ham_so
1.2 tuong giao_cua_do_thi_cac_ham_so
 
Toan pt.de017.2012
Toan pt.de017.2012Toan pt.de017.2012
Toan pt.de017.2012
 
Toan pt.de101.2011
Toan pt.de101.2011Toan pt.de101.2011
Toan pt.de101.2011
 
[Vnmath.com] de-thi-lan5-chuyen-vinhphuc-a
[Vnmath.com] de-thi-lan5-chuyen-vinhphuc-a[Vnmath.com] de-thi-lan5-chuyen-vinhphuc-a
[Vnmath.com] de-thi-lan5-chuyen-vinhphuc-a
 
Ds10 c4a
Ds10 c4aDs10 c4a
Ds10 c4a
 

Viewers also liked

Target audience questionnaire analysis
Target audience questionnaire analysisTarget audience questionnaire analysis
Target audience questionnaire analysisMeghan Mcardle
 
14.hinhgiaitichphang
14.hinhgiaitichphang14.hinhgiaitichphang
14.hinhgiaitichphanggadaubac2003
 
Teaser trailer power point
Teaser trailer power pointTeaser trailer power point
Teaser trailer power pointMeghan Mcardle
 
Decuonghoa9 10-11-140802224422-phpapp01
Decuonghoa9 10-11-140802224422-phpapp01Decuonghoa9 10-11-140802224422-phpapp01
Decuonghoa9 10-11-140802224422-phpapp01gadaubac2003
 
Baigiang13 duong thang trong mat phang
Baigiang13 duong thang trong mat phangBaigiang13 duong thang trong mat phang
Baigiang13 duong thang trong mat phanggadaubac2003
 
Movie magazine industry
Movie magazine industryMovie magazine industry
Movie magazine industryMeghan Mcardle
 
презентация
презентацияпрезентация
презентацияfondifond
 
Future Talent Council meeting, April 2014, on the campus of Guilford Technica...
Future Talent Council meeting, April 2014, on the campus of Guilford Technica...Future Talent Council meeting, April 2014, on the campus of Guilford Technica...
Future Talent Council meeting, April 2014, on the campus of Guilford Technica...Steve Moore
 
Economic and Educational Profile by Brookings Instiution on Gulford County
Economic and Educational Profile by Brookings Instiution on Gulford CountyEconomic and Educational Profile by Brookings Instiution on Gulford County
Economic and Educational Profile by Brookings Instiution on Gulford CountySteve Moore
 
How is a movie poster made
How is a movie poster madeHow is a movie poster made
How is a movie poster madeMeghan Mcardle
 
Movie poster codes and conventions!
Movie poster codes and conventions!Movie poster codes and conventions!
Movie poster codes and conventions!Meghan Mcardle
 
Chuyen de nc hh 10 nguyen minh ha
Chuyen de nc hh 10 nguyen minh haChuyen de nc hh 10 nguyen minh ha
Chuyen de nc hh 10 nguyen minh hagadaubac2003
 

Viewers also liked (16)

Target audience questionnaire analysis
Target audience questionnaire analysisTarget audience questionnaire analysis
Target audience questionnaire analysis
 
14.hinhgiaitichphang
14.hinhgiaitichphang14.hinhgiaitichphang
14.hinhgiaitichphang
 
Teaser trailer power point
Teaser trailer power pointTeaser trailer power point
Teaser trailer power point
 
Researching genre
Researching genre  Researching genre
Researching genre
 
Decuonghoa9 10-11-140802224422-phpapp01
Decuonghoa9 10-11-140802224422-phpapp01Decuonghoa9 10-11-140802224422-phpapp01
Decuonghoa9 10-11-140802224422-phpapp01
 
Baigiang13 duong thang trong mat phang
Baigiang13 duong thang trong mat phangBaigiang13 duong thang trong mat phang
Baigiang13 duong thang trong mat phang
 
Movie magazine industry
Movie magazine industryMovie magazine industry
Movie magazine industry
 
Researching Genre
Researching GenreResearching Genre
Researching Genre
 
презентация
презентацияпрезентация
презентация
 
Dalin
DalinDalin
Dalin
 
Future Talent Council meeting, April 2014, on the campus of Guilford Technica...
Future Talent Council meeting, April 2014, on the campus of Guilford Technica...Future Talent Council meeting, April 2014, on the campus of Guilford Technica...
Future Talent Council meeting, April 2014, on the campus of Guilford Technica...
 
Economic and Educational Profile by Brookings Instiution on Gulford County
Economic and Educational Profile by Brookings Instiution on Gulford CountyEconomic and Educational Profile by Brookings Instiution on Gulford County
Economic and Educational Profile by Brookings Instiution on Gulford County
 
How is a movie poster made
How is a movie poster madeHow is a movie poster made
How is a movie poster made
 
Movie poster codes and conventions!
Movie poster codes and conventions!Movie poster codes and conventions!
Movie poster codes and conventions!
 
Chuyen de nc hh 10 nguyen minh ha
Chuyen de nc hh 10 nguyen minh haChuyen de nc hh 10 nguyen minh ha
Chuyen de nc hh 10 nguyen minh ha
 
Case law
Case lawCase law
Case law
 

Similar to 10 cd

14.hinhgiaitichphang
14.hinhgiaitichphang14.hinhgiaitichphang
14.hinhgiaitichphanggadaubac2003
 
Toan pt.de128.2011
Toan pt.de128.2011Toan pt.de128.2011
Toan pt.de128.2011BẢO Hí
 
De thi thu dh thpt nam sach hai duong
De thi thu dh thpt nam sach hai duongDe thi thu dh thpt nam sach hai duong
De thi thu dh thpt nam sach hai duongVui Lên Bạn Nhé
 
khao sat ham so và các bài toán liên quan
khao sat ham so và các bài toán liên quankhao sat ham so và các bài toán liên quan
khao sat ham so và các bài toán liên quandinhduysp
 
Toan pt.de015.2011
Toan pt.de015.2011Toan pt.de015.2011
Toan pt.de015.2011BẢO Hí
 
Toan pt.de057.2010
Toan pt.de057.2010Toan pt.de057.2010
Toan pt.de057.2010BẢO Hí
 
Thi thử toán hậu lộc 4 th 2012 lần 1
Thi thử toán hậu lộc 4 th 2012 lần 1Thi thử toán hậu lộc 4 th 2012 lần 1
Thi thử toán hậu lộc 4 th 2012 lần 1Thế Giới Tinh Hoa
 
Cd bptdaiso
Cd bptdaisoCd bptdaiso
Cd bptdaisotonyjony
 
Cac bai toan lien quan den khao sat ham so
Cac bai toan lien quan den khao sat ham soCac bai toan lien quan den khao sat ham so
Cac bai toan lien quan den khao sat ham soHuynh ICT
 
ham-so-on-thi-dh-huynh-bao-toan
 ham-so-on-thi-dh-huynh-bao-toan ham-so-on-thi-dh-huynh-bao-toan
ham-so-on-thi-dh-huynh-bao-toanHuynh ICT
 
Toan pt.de058.2011
Toan pt.de058.2011Toan pt.de058.2011
Toan pt.de058.2011BẢO Hí
 
Cm bat dang thuc bang pp tiep tuyen
Cm bat dang thuc bang pp tiep tuyenCm bat dang thuc bang pp tiep tuyen
Cm bat dang thuc bang pp tiep tuyenVui Lên Bạn Nhé
 
Bai giang 8_tiep_tuyen
Bai giang 8_tiep_tuyenBai giang 8_tiep_tuyen
Bai giang 8_tiep_tuyengadaubac2003
 
Giao trinh phuong phap tinh 2
Giao trinh phuong phap tinh 2 Giao trinh phuong phap tinh 2
Giao trinh phuong phap tinh 2 Tùng Lê
 

Similar to 10 cd (20)

Chde hamsobac4
Chde hamsobac4Chde hamsobac4
Chde hamsobac4
 
Hambac4
Hambac4Hambac4
Hambac4
 
Hàm bậc 4
Hàm bậc 4Hàm bậc 4
Hàm bậc 4
 
10.khaosaths
10.khaosaths10.khaosaths
10.khaosaths
 
10.khaosaths
10.khaosaths10.khaosaths
10.khaosaths
 
14.hinhgiaitichphang
14.hinhgiaitichphang14.hinhgiaitichphang
14.hinhgiaitichphang
 
Toan pt.de128.2011
Toan pt.de128.2011Toan pt.de128.2011
Toan pt.de128.2011
 
De thi thu dh thpt nam sach hai duong
De thi thu dh thpt nam sach hai duongDe thi thu dh thpt nam sach hai duong
De thi thu dh thpt nam sach hai duong
 
khao sat ham so và các bài toán liên quan
khao sat ham so và các bài toán liên quankhao sat ham so và các bài toán liên quan
khao sat ham so và các bài toán liên quan
 
Toan pt.de015.2011
Toan pt.de015.2011Toan pt.de015.2011
Toan pt.de015.2011
 
Toan pt.de057.2010
Toan pt.de057.2010Toan pt.de057.2010
Toan pt.de057.2010
 
Elip
ElipElip
Elip
 
Thi thử toán hậu lộc 4 th 2012 lần 1
Thi thử toán hậu lộc 4 th 2012 lần 1Thi thử toán hậu lộc 4 th 2012 lần 1
Thi thử toán hậu lộc 4 th 2012 lần 1
 
Cd bptdaiso
Cd bptdaisoCd bptdaiso
Cd bptdaiso
 
Cac bai toan lien quan den khao sat ham so
Cac bai toan lien quan den khao sat ham soCac bai toan lien quan den khao sat ham so
Cac bai toan lien quan den khao sat ham so
 
ham-so-on-thi-dh-huynh-bao-toan
 ham-so-on-thi-dh-huynh-bao-toan ham-so-on-thi-dh-huynh-bao-toan
ham-so-on-thi-dh-huynh-bao-toan
 
Toan pt.de058.2011
Toan pt.de058.2011Toan pt.de058.2011
Toan pt.de058.2011
 
Cm bat dang thuc bang pp tiep tuyen
Cm bat dang thuc bang pp tiep tuyenCm bat dang thuc bang pp tiep tuyen
Cm bat dang thuc bang pp tiep tuyen
 
Bai giang 8_tiep_tuyen
Bai giang 8_tiep_tuyenBai giang 8_tiep_tuyen
Bai giang 8_tiep_tuyen
 
Giao trinh phuong phap tinh 2
Giao trinh phuong phap tinh 2 Giao trinh phuong phap tinh 2
Giao trinh phuong phap tinh 2
 

More from gadaubac2003

24hchiase.com toadophang
24hchiase.com toadophang24hchiase.com toadophang
24hchiase.com toadophanggadaubac2003
 
24hchiase.com tuyen-bdt-gtln-gtnn
24hchiase.com tuyen-bdt-gtln-gtnn24hchiase.com tuyen-bdt-gtln-gtnn
24hchiase.com tuyen-bdt-gtln-gtnngadaubac2003
 
10 nhi thuc niuton_mathvn.com
10 nhi thuc niuton_mathvn.com10 nhi thuc niuton_mathvn.com
10 nhi thuc niuton_mathvn.comgadaubac2003
 
Baigiang10 nhi thuc niu ton (1)
Baigiang10 nhi thuc niu ton (1)Baigiang10 nhi thuc niu ton (1)
Baigiang10 nhi thuc niu ton (1)gadaubac2003
 
Baigiang10 nhi thuc niu ton
Baigiang10 nhi thuc niu tonBaigiang10 nhi thuc niu ton
Baigiang10 nhi thuc niu tongadaubac2003
 
Dai so-to-hop-nhi-thuc-niu ton-xac-suat-ltdh
Dai so-to-hop-nhi-thuc-niu ton-xac-suat-ltdhDai so-to-hop-nhi-thuc-niu ton-xac-suat-ltdh
Dai so-to-hop-nhi-thuc-niu ton-xac-suat-ltdhgadaubac2003
 
Toan daisotohop-chuong5(1) (1)
Toan daisotohop-chuong5(1) (1)Toan daisotohop-chuong5(1) (1)
Toan daisotohop-chuong5(1) (1)gadaubac2003
 
Toan daisotohop-chuong5(2)
Toan daisotohop-chuong5(2)Toan daisotohop-chuong5(2)
Toan daisotohop-chuong5(2)gadaubac2003
 
Toa do-trong-mat-phang
Toa do-trong-mat-phangToa do-trong-mat-phang
Toa do-trong-mat-phanggadaubac2003
 
Baigiang14 duong tron
Baigiang14 duong tronBaigiang14 duong tron
Baigiang14 duong trongadaubac2003
 
Baigiang15 ba duong conic
Baigiang15 ba duong conicBaigiang15 ba duong conic
Baigiang15 ba duong conicgadaubac2003
 
Chuyen de-9-hinh-hoc-phang
Chuyen de-9-hinh-hoc-phangChuyen de-9-hinh-hoc-phang
Chuyen de-9-hinh-hoc-phanggadaubac2003
 
Bai tap-axit-h cl-trong-de-thi-dh
Bai tap-axit-h cl-trong-de-thi-dhBai tap-axit-h cl-trong-de-thi-dh
Bai tap-axit-h cl-trong-de-thi-dhgadaubac2003
 
Bai tap-kim-loai-trong-de-thi-dh
Bai tap-kim-loai-trong-de-thi-dhBai tap-kim-loai-trong-de-thi-dh
Bai tap-kim-loai-trong-de-thi-dhgadaubac2003
 
Kimloiaxit 130724215139-phpapp01
Kimloiaxit 130724215139-phpapp01Kimloiaxit 130724215139-phpapp01
Kimloiaxit 130724215139-phpapp01gadaubac2003
 
[Www.pne.edu.vn] bo de thi hoc sinh gioi toan lop 8 tap 2
[Www.pne.edu.vn] bo de thi hoc sinh gioi toan lop 8 tap 2[Www.pne.edu.vn] bo de thi hoc sinh gioi toan lop 8 tap 2
[Www.pne.edu.vn] bo de thi hoc sinh gioi toan lop 8 tap 2gadaubac2003
 
Giao an sinh hoat lop
Giao an sinh hoat lopGiao an sinh hoat lop
Giao an sinh hoat lopgadaubac2003
 

More from gadaubac2003 (20)

24hchiase.com toadophang
24hchiase.com toadophang24hchiase.com toadophang
24hchiase.com toadophang
 
24hchiase.com tuyen-bdt-gtln-gtnn
24hchiase.com tuyen-bdt-gtln-gtnn24hchiase.com tuyen-bdt-gtln-gtnn
24hchiase.com tuyen-bdt-gtln-gtnn
 
1 to hop
1 to hop1 to hop
1 to hop
 
10 nhi thuc niuton_mathvn.com
10 nhi thuc niuton_mathvn.com10 nhi thuc niuton_mathvn.com
10 nhi thuc niuton_mathvn.com
 
Baigiang10 nhi thuc niu ton (1)
Baigiang10 nhi thuc niu ton (1)Baigiang10 nhi thuc niu ton (1)
Baigiang10 nhi thuc niu ton (1)
 
Baigiang10 nhi thuc niu ton
Baigiang10 nhi thuc niu tonBaigiang10 nhi thuc niu ton
Baigiang10 nhi thuc niu ton
 
Dai so-to-hop-nhi-thuc-niu ton-xac-suat-ltdh
Dai so-to-hop-nhi-thuc-niu ton-xac-suat-ltdhDai so-to-hop-nhi-thuc-niu ton-xac-suat-ltdh
Dai so-to-hop-nhi-thuc-niu ton-xac-suat-ltdh
 
Nhi thuc-niuton-2
Nhi thuc-niuton-2Nhi thuc-niuton-2
Nhi thuc-niuton-2
 
Nhịthucniuton
NhịthucniutonNhịthucniuton
Nhịthucniuton
 
Toan daisotohop-chuong5(1) (1)
Toan daisotohop-chuong5(1) (1)Toan daisotohop-chuong5(1) (1)
Toan daisotohop-chuong5(1) (1)
 
Toan daisotohop-chuong5(2)
Toan daisotohop-chuong5(2)Toan daisotohop-chuong5(2)
Toan daisotohop-chuong5(2)
 
Toa do-trong-mat-phang
Toa do-trong-mat-phangToa do-trong-mat-phang
Toa do-trong-mat-phang
 
Baigiang14 duong tron
Baigiang14 duong tronBaigiang14 duong tron
Baigiang14 duong tron
 
Baigiang15 ba duong conic
Baigiang15 ba duong conicBaigiang15 ba duong conic
Baigiang15 ba duong conic
 
Chuyen de-9-hinh-hoc-phang
Chuyen de-9-hinh-hoc-phangChuyen de-9-hinh-hoc-phang
Chuyen de-9-hinh-hoc-phang
 
Bai tap-axit-h cl-trong-de-thi-dh
Bai tap-axit-h cl-trong-de-thi-dhBai tap-axit-h cl-trong-de-thi-dh
Bai tap-axit-h cl-trong-de-thi-dh
 
Bai tap-kim-loai-trong-de-thi-dh
Bai tap-kim-loai-trong-de-thi-dhBai tap-kim-loai-trong-de-thi-dh
Bai tap-kim-loai-trong-de-thi-dh
 
Kimloiaxit 130724215139-phpapp01
Kimloiaxit 130724215139-phpapp01Kimloiaxit 130724215139-phpapp01
Kimloiaxit 130724215139-phpapp01
 
[Www.pne.edu.vn] bo de thi hoc sinh gioi toan lop 8 tap 2
[Www.pne.edu.vn] bo de thi hoc sinh gioi toan lop 8 tap 2[Www.pne.edu.vn] bo de thi hoc sinh gioi toan lop 8 tap 2
[Www.pne.edu.vn] bo de thi hoc sinh gioi toan lop 8 tap 2
 
Giao an sinh hoat lop
Giao an sinh hoat lopGiao an sinh hoat lop
Giao an sinh hoat lop
 

10 cd

  • 1. CHUYEÂN ÑEÀ 1 TOÏA ÑOÄ PHAÚNG Trong caùc baøi toaùn veà toïa ñoä trong maët phaúng thöôøng gaëp caùc yeâu caàu nhö tìm toïa ñoä moät ñieåm, moät vectô, tính ñoä daøi moät ñoaïn thaúng, soá ño goùc giöõa hai vectô, quan heä cuøng phöông hoaëc vuoâng goùc giöõa hai vectô, 3 ñieåm thaúng haøng. Ta vaän duïng caùc kieán thöùc cô baûn sau ñaây: Cho a = ( , = ta coù:b ))1 2a , a ( 1 2b , b a = b ⇔ 1 2 1 2 a = b a = b ⎧ ⎨ ⎩ a + = ( , )b 1 1a + b 2 2a + b a – = ( , )b 1 1a - b 2 2a - b ka = (k , k ) (k1a 2a ∈ R) α + = ( +a β b α 1a β 1b , α 2a + β 2b ) a . = +b 1a 1b 2a 2b . Vôùi caùc quan heä veà ñoä daøi ta coù: a = ( , )1a 2a ⇒ a = 2 2 1 2a + a ( ) ( ) A A B B A x , y B x , y ⎧⎪ ⎨ ⎪⎩ ⇒ AB = ( – , – )Bx Ax By Ay vaø AB = ( ) ( ) 2 2 B A B Ax - x y - y+ . Vôùi quan heä cuøng phöông hoaëc vuoâng goùc ta coù: a + = 0⊥ b ⇔ 1a 1b 2a 2b cuøng phöônga b ⇔ sin(a, b) = 0 ⇔ – = 01a 2b 2a 1b ⇔ 1 1 a b = 2 2 a b ( ,1b 2b ≠ 0) A, B, C thaúng haøng ⇔ AB cuøng phöông AC
  • 2. ⇔ B A B A C A C A x - x y - y x - x y - y = 0 . Vôùi vieäc tìm goùc cuûa hai vectô ta coù: - Goùc hình hoïc taïo bôûi hai vectô a, b ñöôïc suy töø coâng thöùc: cos(a, b ) = 1 1 22a b + a b a . b (1) - Soá ño goùc ñònh höôùng cuûa hai vectô a, b ngoaøi (1) coøn ñöôïc suy theâm töø moät trong hai coâng thöùc: sin(a, b) = 1 2 12a b - a b a . b tg(a, b) = 1 2 1 1 1 2 2 2 a b - a b a b + a b Ngoaøi ra trong caùc baøi toaùn veà toïa ñoä phaúng ta coù theå aùp duïng caùc keát quaû sau ñaây: . M( , ) laø trung ñieåm cuûa ñoaïn thaúng ABMx My ⇔ 2 2 A B M A B M x + x x = y + y y = ⎧ ⎪⎪ ⎨ ⎪ ⎪⎩ . G( , ) laø troïng taâm cuûaGx Gy Δ ABC ⇔ 3 3 ⎧ ⎪⎪ ⎨ ⎪ ⎪⎩ A B G A B G x + x + x x = y + y + y y = C C . I( , ) vaø J( , ) laø chaân ñöôøng phaân giaùc trong vaø ngoaøi cuûa goùc A trong ABC thì: Ix Iy Jx Jy Δ IB IC = − JB JC = − AB AC . Vôùi A( , ), B( , ), C( , ) thì dieän tích tam giaùc ABC laø:Ax Ay Bx By Cx Cy S = 1 2 Δ vôùi Δ = B A B A C A C A x - x y - y x - x y - y Ví duï 1:
  • 3. Trong maët phaúng Oxy cho ba ñieåm A(2, –1), B(0, 3), C(4, 2). a) Tìm toïa ñoä ñieåm D ñoái xöùng vôùi A qua B. b) Tìm toïa ñoä ñieåm M ñeå 2 + 3AM BM - 4CM = 0 c) Tìm toïa ñoä ñieåm E ñeå ABCE laø hình thang coù moät caïnh ñaùy laø AB vaø E naèm treân Ox. d) Tìm toïa ñoä tröïc taâm H, troïng taâm G vaø taâm I ñöôøng troøn ngoaïi tieáp ABC.Δ e) Chöùng toû H, G, I thaúng haøng. Giaûi a) D laø ñieåm ñoái xöùng cuûa A qua B B laø trung ñieåm cuûa AD⇔ ⇔ A D B A D B x + x x = 2 y + y y = 2 ⎧ ⎪⎪ ⎨ ⎪ ⎪⎩ hay D(–2, 7)⇔ ( ) ( ) − −⎧⎪ ⎨ −⎪⎩ D B A D B A x = 2x x = 2 0 2 = 2 y = 2y y = 2 3 + 1 = 7 − b) Ta coù: 2 + 3 BM – 4CMAM = 0 = ( 0, 0 ) ⇔ ( ) ( ) ( ) ( ) ( ) ( ) − − − −⎧⎪ ⎨ − − −⎪⎩ M M M M M M 2 x 2 + 3 x 0 4 x 4 = 0 2 y + 1 + 3 y 3 4 y 2 = 0 ⇔ hay M(–12, –1) −⎧ ⎨ −⎩ M M x = 12 y = 1 c) ABCE laø hình thang coù ñaùy AB vaø E naèm treân Ox. ⇔ Ey = 0 CE ⎧⎪ ⎨ ΑΒ⎪⎩ // ⇔ E E E y = 0 x - 4 y - 2 = 0 - 2 3 + 1 ⎧ ⎪ ⎨ ⎪⎩ ⇔ hay E(5, 0) E E y = 0 x = 5 ⎧ ⎨ ⎩ d) H laø tröïc taâm cuûa ABCΔ ⇔ AH BC BH AC ⊥⎧ ⎨ ⊥⎩ ⇔ AH.BC = 0 BH.AC = 0 ⎧⎪ ⎨ ⎪⎩
  • 4. ⇔ ( )( ) ( )( ) ( )( ) ( )( ) 4 1 2 4 2 3 2 1 0 − − + + − =⎧⎪ ⎨ − − + − + =⎪⎩ H H H H x 2 0 y x 0 y 3 0 2 3 9 H H H H x y x y − − =⎧ ⎨ + − =⎩ ⇔ 4 9 0 0 ⇔ 18 7 9 7 H H x y ⎧ =⎪⎪ ⎨ ⎪ = ⎪⎩ hay H 18 7 9 , 7 ⎛ ⎞ ⎜ ⎟ ⎝ ⎠ G laø troïng taâm ABC ta coù:Δ 2 0 4 2 3 3 1 3 2 4 3 3 A B C G A B C G x x x x y y y y + + + +⎧ = =⎪⎪ ⎨ + + − + +⎪ = = ⎪⎩ 3 = = hay G 4 2 3 , ⎛ ⎞ ⎜ ⎟ ⎝ ⎠ + I laø taâm ñöôøng troøn ngoaïi tieáp Δ ABC ⇔ IA = IB = IC ⇔ 2 2 2 2 IA IB IA IC ⎧ =⎪ ⎨ =⎪⎩ ⇔ ( ) ( ) ( ) ( ( ) ( ) ( ) ( 2 2 2 2 2 2 2 1 0 3 2 1 4 2 I I I I I I x y x x y x ⎧ − + − − = − + −⎪ ⎨ − + − − = − + −⎪⎩ ) ) 2 2 I I y y 0 0 ⇔ 4 8 4 4 6 15 I I I I x y x y − + − =⎧ ⎨ + − =⎩ ⇔ 24 12 14 7 19 14 I I x y ⎧ = =⎪⎪ ⎨ ⎪ = ⎪⎩ hay I 12 19 7 14 , ⎛ ⎞ ⎜ ⎟ ⎝ ⎠ e) Ta coù : =HG 4 1 7 21 , ⎛ ⎞ −⎜ ⎟ ⎝ ⎠ vaø HI = 6 1 7 14 , ⎛ ⎞ −⎜ ⎟ ⎝ ⎠ ⇒ 4 7 6 7 − − = 1 21 1 14 = 2 3 ⇒ cuøng phöông vôùiHG HI ⇒ H, I, G thaúng haøng. Ví duï 2: Trong maët phaúng Oxy cho A(2, 2 3 ), B(1, 3 3 ), C (-1, 3 ) . Tính
  • 5. cos ( AO, AB) vaø dieän tích tam giaùc ABC. Giaûi Ta coù: AO = (–2, –2 3 ), AB = (–1, 3 ) = ( a1;a2 ) cos( AO, AB) = 2 6 4 12 1 3. − + + = 1 2 − AC = (–3, – 3 ) = = ( b1; b2 ) ⇒ 1 2 2 1 1 2 = −ABCS a b a b = 1 1 3 3 3 2 − − − −( )( ) ( ) = 2 3 * * *
  • 6. CHUYEÂN ÑEÀ 2 ÑÖÔØNG VAØ PHÖÔNG TRÌNH ÑÖÔØNG Caùc baøi toaùn veà phaàn ñöôøng vaø phöông trình ñöôøng thöôøng yeâu caàu xaùc ñònh quyõ tích caùc ñieåm trong maët phaúng toïa ñoä theo nhöõng ñieàu kieän cho tröôùc, quyõ tích naøy laø moät ñöôøng maø ta phaûi tìm phöông trình cuûa noù döïa vaøo ñònh nghóa: F(x, y) = 0 laø phöông trình cuûa ñöôøng (L) neáu ta coù : M( , ) ∈ (L) F( , ) = 0Mx My ⇔ Mx My Neáu M ∈ (L) vaø M coù toïa ñoä phuï thuoäc tham soá t: ( ) ( ) x f t y g t =⎧⎪ ⎨ =⎪⎩ (t ∈ R) thì ñoù laø phöông trình tham soá cuûa ñöôøng (L). Töø phöông trình tham soá, ta khöû t thì coù theå trôû veà daïng F(x, y) = 0 Löu yù vieäc giôùi haïn cuûa quyõ tích tuyø theo caùc ñieàu kieän ñaõ cho trong ñaàu baøi. Ví du1: Trong maët phaúng Oxy cho A(2, 1), B(–3, 2). Tìm quyõ tích ñieåm M ñeå ( MA + MB ) AB = 1 Giaûi Goïi (L) laø quyõ tích phaûi tìm. M( , ) ∈ (L)Mx My ⇔ ( MA + MB ) AB = 1 [ (2 – ) + (–3 – ) ] (–3 – 2) + (1 – + 2 – ) (2 – 1) = 1⇔ Mx Mx My My 5 + 10 + 3 – 2 = 1⇔ Mx My 10 – 2 + 7 = 0⇔ Mx My M( , ) coù toïa ñoä thoûa phöông trình⇔ Mx My F(x, y) = 10x – 2y + 7 = 0 Vaäy quyõ tích phaûi tìm laø ñöôøng thaúng (L) coù phöông trình 10x – 2y + 7 = 0. 1
  • 7. Ví duï 2: Laäp phöông trình quyõ tích taâm cuûa nhöõng ñöôøng troøn tieáp xuùc vôùi truïc Ox vaø ñi qua ñieåm A(1, 2). Giaûi Goïi (L) laø quyõ tích nhöõng taâm ñöôøng troøn tieáp xuùc vôùi truïc Ox vaø ñi qua ñieåm A(1, 2). I( , ) ∈ (L) I laø taâm ñöôøng troøn qua A(1, 2) vaø tieáp xuùc vôùi Ox taïi MIx Iy ⇔ ⇔ IM Ox taïi M IM = IA ⊥⎧ ⎨ ⎩ ⇔ ( ) ( ) ( ) ( ) 2 2 2 0 0M I M M I M I A I A I x x vaø y x x y y x x y y − = =⎧⎪ ⎨ − + − = − + −⎪⎩ 2 – 2 – 4 + 5 = 0⇔ 2 Ix Ix Iy I( , ) coù toïa ñoä thoûa phöông trình⇔ Ix Iy F(x, y) = x2 – 2x – 4y + 5 = 0 Ñoù laø phöông trình cuûa quyõ tích phaûi tìm (Parabol). * * * 2
  • 8. CHUYEÂN ÑEÀ 3 ÑÖÔØNG THAÚNG I. PHÖÔNG TRÌNH ÑÖÔØNG THAÚNG Trong maët phaúng toïa ñoä Oxy, muoán vieát phöông trình moät ñöôøng thaúng ta caàn phaûi bieát: ( )Δ 1) ( qua ñieåm M0(x0, y0) vaø coù vectô chæ phöông a)Δ = (a1, a2) seõ coù: . Phöông trình tham soá : (t 0 0 2 x x ta y y ta = +⎧ ⎨ = +⎩ 1 ∈ R) . Phöông trình chính taéc : 0 1 x x a − = 0 2 y y a − (a1, a2 ≠ 0) Töø phöông trình chính taéc ta coù theå ñoåi thaønh daïng phöông trình toång quaùt : Ax + By + C = 0 (A2 + B2 > 0) 2) ( qua ñieåm M0(x0, y0) vaø coù 1 phaùp veùctô laø (a,b) coù phöông trình : a(x – x0) + b(y – y0) = 0 )Δ 3) i) Phöông trình ñöôøng thaúng trong maët phaúng coù daïng Ax + By + C = 0 vôùi A2 + B2 > 0 (1) ii) Phöông trình ñöôøng thaúng trong maët phaúng coù daïng x = x0 hoaëc y = kx + m (2). Ta deã daøng thaáy (1) vaø (2) laø töông ñöông. + (2) ⇔ kx –y + m = 0 ⇒ (2 ) thoûa (1) vôùi A = k, B = - 1 , C = m. + Neáu B = 0 ⇒ = − C x A , coù daïng x = x0 vôùi x0 = − C A . Neáu B ≠ 0 ⇒ = − − A C y x B B , coù daïng y = kx + m. 3) ( qua hai ñieåm A(xA, yA), B(xB, yB) coù phöông trình :)Δ A B A x x x x − − = A B A y y y y − − neáu 0− − ≠B A B A(x x )( y y ) 1
  • 9. Neáu ( qua A(a, 0) ∈ Ox vaø B(0, b))Δ ∈ Oy vôùi a.b ≠ 0; ta noùi ( )Δ coù ñoaïn chaén a, b vôùi phöông trình: x a + y b = 1 * Ghi chuù: Neáu ñeà baøi toaùn yeâu caàu ta vieát phöông trình cuûa ñöôøng thaúng, thoâng thöôøng ta neân vieát phöông trình ôû daïng toång quaùt vaø löu yù : ( )Δ : Ax + By + C = 0 thì ( )Δ coù : . moät phaùp vectô = (A, B)n . moät vectô chæ phöông a = (–B, A) . heä soá goùc k = tg( , ) =Ox Δ A B − . ( ) (′Δ // ( )Δ ⇒ )′Δ : Ax + By + C0 = 0 . ( ) (′Δ ⊥ ( )Δ ⇒ )′Δ : Bx – Ay + C0 = 0 Ta tìm ñöôïc C0 neáu bieát theâm moät ñieåm naèm treân ( )′Δ . Ngoaøi ra khi vieát phöông trình cuûa moät ñöôøng thaúng ( )Δ theo heä soá goùc k, baøi toaùn coù theå bò thieáu nghieäm do tröôøng hôïp ( )Δ ⊥ x′x (heä soá goùc k khoâng toàn taïi), do ñoù ta phaûi xeùt theâm tröôøng hôïp coù phöông trình x = C ñeå xem ñöôøng thaúng( )Δ ( )Δ naøy coù thoûa maõn ñieàu kieän cuûa ñaàu baøi khoâng. Ghi chuù - Neáu n = (A, B) laø 1 phaùp veùc tô cuûa ñöôøng thaúng ( )Δ thì k.n = (kA, kB) cuõng laø phaùp veùc tô cuûa ( )Δ vôùi moïi soá thöïc k ≠ 0. - Neáu laø 1 veùc tô chæ phöông cuûa ñöôøng thaúng1 2=a (a ,a ) ( )Δ thì k. cuõng laø veùc tô chæ phöông cuûa1 2=a (ka ,ka ) ( )Δ vôùi moïi soá thöïc k khaùc 0. II. VÒ TRÍ TÖÔNG ÑOÁI CUÛA HAI ÑÖÔØNG THAÚNG Ñeå xeùt vò trí töông ñoái cuûa hai ñöôøng thaúng ta caàn nhôù Cho (d1) : A1x + B1y + C1 = 0 vaø (d2) : A2x + B2y + C2 = 0 Ñaët : 2
  • 10. D = 1 1 2 2 A B A B ; Dx = 1 1 2 2 B C B C ; Dy = 1 2 2 C A C A 1 thì : D ≠ 0 ⇔ (d1) caét (d2) taïi I 1 x I y D x D D y D ⎧ =⎪⎪ ⎨ ⎪ = ⎪⎩ D = 0 vaø Dx 0 hoaëc Dy≠ ≠ 0 ⇔ (d1) // (d2) D = Dx = Dy = 0 ⇔ (d1) ≡ (d2) hoaëc vôùi A2, B2, C2 0 ta coù :≠ 1 2 A A ≠ 1 2 B B ⇔ (d1) caét (d2) 1 2 A A = 1 2 B B ≠ 1 2 C C ⇔ (d1) // (d2) 1 2 A A = 1 2 B B = 1 2 C C ⇔ (d1) ≡ (d2) Ghi chuù 1 1 2 2 B C B C = 1 1 2 2 − C B C B ; 1 1 2 2 C A C A = 1 1 2 2 − A C A C III. GOÙC GIÖÕA HAI ÑÖÔØNG THAÚNG Ñeå tìm goùc giöõa hai ñöôøng thaúng, ta goïi α laø goùc nhoïn taïo bôûi hai ñöôøng thaúng (d1) : A1x + B1y + C1 = 0 (d2) : A2x + B2y + C2 = 0 thì cosα = 1 2 1 2 2 2 2 1 1 2 2 2 A A B B A B . A B + + + IV. KHOAÛNG CAÙCH TÖØ MOÄT ÑIEÅM ÑEÁN MOÄT ÑÖÔØNG THAÚNG Ñeå tìm khoaûng caùch töø ñieåm M(xM, yM) ñeán ñöôøng thaúng ( )Δ : Ax + By + C = 0 ta aùp duïng coâng thöùc : 3
  • 11. d(M,Δ ) = 2 2 M MAx By C A B + + + Khoaûng caùch ñaïi soá töø ñöôøng thaúng ( )Δ ñeán ñieåm M(xM, yM) laø : t = 2 2 M MAx By A B + + + C Ñaët phaùp vectô = (A, B) coù goác leânn ( )Δ thì : . t > 0 neáu ñieåm M vaø n naèm cuøng moät beân ñoái vôùi ( )Δ . t < 0 neáu ñieåm M vaø n naèm khaùc beân ñoái vôùi ( )Δ Phöông trình ñöôøng phaân giaùc cuûa goùc hôïp bôûi 2 ñöôøng thaúng (d1) : A1x + B1y + C1 = 0 vaø (d2) : A2x + B2y + C2 = 0 laø : 1 1 2 2 1 1 1A x B y C A B + + + = ± 2 2 2 2 2 2 2 A x B y C A B + + + Ví duï 1: Cho tam giaùc ABC vôùi A(–2, 1), B(4, 3), C(2,–3) a) Tìm phöông trình tham soá vaø toång quaùt caïnh BC. b) Tìm phöông trình ñöôøng cao AH. c) Tìm phöông trình ñöôøng thaúng qua A(–2, 1) vaø song song vôùi BC. Giaûi a) Ñöôøng thaúng qua caïnh BC nhaän BC = (–2, –6) hay (1,3) laøm vectô chæ phöông vaø qua B(4, 3) neân coù phöông trình tham soá : (t 4 3 3 = +⎧ ⎨ = +⎩ x t y t ∈ R) ⇔ 4 1 −x = 3 3 −y (phöông trình chính taéc) ⇔ 3x – y – 9 = 0 laø phöông trình toång quaùt cuûa BC. b) Δ ABC coù ñöôøng cao AH ⊥ BC : 3x – y – 9 = 0 ⇒ pt AH : x + 3y + C1 = 0 4
  • 12. A(–2, 1) ∈ AH –2 + 3(1) + C1 = 0⇔ ⇔ C1 = –1 Vaäy pt AH : x + 3y – 1 = 0 c) Ñöôøng thaúng Au // BC ⇒pt Au : 3x – y + C2 = 0 A(–2, 1) ∈ Au ⇔ 3(–2) – 1 + C2 = 0 ⇔ C2 = 7 Vaäy pt Au : 3x – y + 7 = 0 Ví duï 2: Cho tam giaùc ABC vôùi A(1, –1), B(–2, 1), C(3, 5). a) Vieát phöông trình ñöôøng vuoâng goùc AH keû töø A ñeán trung tuyeán BK cuûa tam giaùc ABC. b) Tính dieän tích tam giaùc ABK. Giaûi a) K laø trung ñieåm cuûa AC ⇔ 2 2 2 2 A C K A C K x x x y y y +⎧ = =⎪⎪ ⎨ +⎪ = = ⎪⎩ hay K(2, 2) Phöông trình caïnh BK : 2 2 2 x − − − = 2 1 2 y − − ⇔ x – 4y + 6 = 0 AH ⊥ BK pt AH : 4x + y + C0 = 0⇒ A(1, - 1) ∈ AH 4(1) + (–1) + C0 = 0⇔ ⇔ C0 = –3 hay AH : 4x + y – 3 = 0 b) Dieän tích tam giaùc ABK laø S = 1 2 AH.BK vôùi AH = A (BK)d = 1 4 6 17 + + S =⇒ 1 2 . 11 17 . 2 2 4 1+ = 11 2 ( ñvdt ). Ví duï 3: ( Ñeà döï tröõ khoái A naêm 2005) Trong maët phaúng vôùi heä toïa ñoä Oxy cho tam giaùc ABC caân taïi ñænh A coù troïng taâm G 4 1 ( ; ) 3 3 , phöông trình ñöôøng thaúng BC laø vaø phöông trình ñöôøng thaúng BG laø 2 4x y− − = 0 07 4 8x y− − = .Tìm toïa ñoä caùc ñænh A, B, C. 5
  • 13. Baøi giaûi Toïa ñoä ñænh B laø nghieäm cuûa heä pt ( ) − − =⎧ ⇒ −⎨ − − =⎩ x 2y 4 0 B 0, 2 7x 4y 8 0 Vì caân taïi A neân AG laø ñöôøng cao cuûaABCΔ ABCΔ Vì ⇒ pt GA:GA BC⊥ − + − = ⇔ + − = 4 1 2(x ) 1(y ) 0 2x y 3 0 3 3 2x y 3 0⇔ + − = ⇒ = HGA BC∩ ( ) + − =⎧ ⇒ −⎨ − − =⎩ 2x y 3 0 H 2, 1 x 2y 4 0 Ta coù H laø trung ñieåm BC ⇒ + = = − = − =⎧ ⎧ ⇒⎨ ⎨ + = = − = − − − =⎩ ⎩ B C H C H B B C H C H B x x 2x x 2x x 2(2) 0 4 y y 2y y 2y y 2( 1) ( 2) 0 )⇒ . Ta coù :(C 4,0 + + + + = =A B C A B C G G x x x y y y x vaø y 3 3 ⇒ ( )A 0,3 Vaäy ( ) ( ) (A 0,3 ,C 4,0 ,B 0, 2− ) Ví duï 4 ( ÑH KHOÁI A -2002) 1. Trong maët phaúng vôùi heä toïa ñoä Ñeâcac vuoâng goùc Oxy cho hình chöõ nhaät ABCD coù taâm I 1 ;0 2 ⎞ ⎟ ⎝ ⎠ ⎛ ⎜ ,phöông trình ñöôøng thaúng AB laø x – 2y + 2 = 0 vaø AB = 2AD .Tìm toïa ñoä caùc ñænh A,B,C,D bieát raèng ñænh A coù hoaønh ñoä aâm . BAØI GIAÛI: A ∈ ñöôøng thaúng x – 2y + 2 = 0 ⇒ A (2a – 2, a) (a < 1) I laø trung ñieåm AC ⇒ C (3 – 2a, −a) BC qua C vaø BC ⊥ AB ⇒ pt BC : 2x + y + 5a – 6 = 0 AB ∩ BC = B ⇒ B (2 – 2a, 2 – a) Ta coù : AB = 2AD ⇔ (1 – a)2 = 1 ⇔ a = 0 hay a = 2 (loaïi) Vaäy A (−2, 0). B (2, 2), C (3, 0), D (−1, −2) Ví duï 5 ( ÑH KHOÁI D -2004) Trong maët phaúng vôùi heä toïa ñoä Oxy cho tam giaùc ABC coù caùc ñænh A (−1; 0); B (4; 0); C (0; m) vôùi m ≠ 0. Tìm toïa ñoä troïng taâm G cuûa tam giaùc ABC theo m. Xaùc ñònh m ñeå tam giaùc GAB vuoâng taïi G. BAØI GIAÛI: G m 1; 3 ⎛ ⎞ ⎜ ⎟ ⎝ ⎠ ; m GA ( 2; ) 3 = − − ; m GB (3; ) 3 = − Tam giaùc GAB vuoâng taïi G ⇔ GA.GB 0= ⇔ 2 m 6 9 − + = 0 ⇔ m = 3 6± . Ví duï6 ( ÑH KHOÁI B -2004) Trong maët phaúng vôùi heä toïa ñoä Oxy cho hai ñieåm A(1; 1), B(4; -3). Tìm ñieåm C thuoäc ñöôøng thaúng 2 1 0x y− − = sao cho khoaûng caùch töø C ñeán ñöôøng thaúng AB baèng 6. BAØI GIAÛI: A (1; 1); B (4; −3) ⇒ phöông trình AB: x 1 y 1 3 4 − − = − ⇔ 4x + 3y – 7 = 0 C ∈ ñt : x – 2y – 1 = 0 ⇒ C (2t + 1; t) 6
  • 14. Ta coù: d (C, AB) = 6 ⇔ 8t 4 3t 7 6 5 + + − = ⇔ 11t 3 30− = ⇔ ⇔ 11t 3 30 11t 3 30 − =⎡ ⎢ − = −⎣ t 3 27 t 11 =⎡ ⎢ ⎢ = − ⎢⎣ Vaäy C (7; 3) hay C 43 27 ; 11 11 ⎛ ⎞ − −⎜ ⎟ ⎝ ⎠ Ví duï7 ( Ñeà DÖÏ TRÖÕ KHOÁI D -2003) Trong maët phaúng vôùi heä toïa ñoä Ñeàcac vuoâng goùc Oxy cho tam giaùc ABC coù ñænh A (1; 0) vaø hai ñöôøng thaúng laàn löôït chöùa caùc ñöôøng cao veõ töø B vaø C coù phöông trình töông öùng laø : x – 2y + 1 = 0 vaø 3x + y – 1 = 0.Tính dieän tích cuûa tam giaùc ABC. BAØI GIAÛI: Vì AC ⊥ BB' ⇒ phöông trình AC : 2x + y + m = 0 A(1; 0) ∈ AC ⇒ 2 + m = 0 ⇒ m = −2 Phöông trình AC : 2x + y – 2 = 0 Vaäy t ñ C laø nghieäm cuûa + − =⎧ ⎨ + − =⎩ 2x y 2 0 3x y 1 0 ⇒ C(−1; 4) Vì AB ⊥ CC' ⇒ phöông trình AB : x – 3y + n = 0 A(1; 0) ∈ AB ⇒ 1 + n = 0 ⇒ n = −1 Phöông trình AB : x – 3y – 1 = 0 Vaäy ⇒ B(−5; −2).⇒ x 3y 1 0 B x 2y 1 0 − − =⎧ ⎨ − + =⎩ AB ⎯→ = (−6; −2); AC ⎯→ = (−2; 4) SΔABC = − −⎡ ⎢−⎣ ⎦ 6 21 2 42 ⎤ ⎥ = 14 (ñvdt). Ví duï8 ( ÑEÀDÖÏ TRÖÕ KHOÁI B -2004) Trong maët phaúng vôùi heä toïa ñoä Oxy cho ñieåm I (–2; 0) vaø hai ñöôøng thaúng d1 : 2x – y + 5 = 0, d2 : x + y – 3 = 0. Vieát phöông trình ñöôøng thaúng d ñi qua ñieåm I vaø caét hai ñöôøng thaúng d1, d2 laàn löôït taïi A, B sao cho : 2 → → =IA IB BAØI GIAÛI: P.trình ñöôøng thaúng d qua I (–2, 0), heä soá goùc k : y = k(x + 2) ⎩ ⎨ ⎧ ⎟ ⎠ ⎞ ⎜ ⎝ ⎛ − − − − ⇒ =+− =+− k k , k k A kykx yx A 22 52 02 052 ⎩ ⎨ ⎧ ⎟ ⎠ ⎞ ⎜ ⎝ ⎛ ++ − ⇒ =+− =−+ k k , k k B kykx yx B 1 5 1 23 02 03 1 k IA ; 2 k 2 k − −⎛ ⎞ = ⎜ ⎟ − −⎝ ⎠ ; ⎟ ⎠ ⎞ ⎜ ⎝ ⎛ ++ = k k ; k IB 1 5 1 5 ⇒ ⎟ ⎠ ⎞ ⎜ ⎝ ⎛ ++ = k k ; k IB 1 10 1 10 2 ⎪ ⎩ ⎪ ⎨ ⎧ ==⇒ + = − − =⇒ + = − − ⇔= 3 7 0 1 10 2 3 7 1 10 2 1 2 k,k k k k k k kkIBIA Do ñoù phöông trình ñöôøng thaúng d laø y = 3 7 (x + 2) 7
  • 15. ⇔ 7x – 3y + 14 = 0 * * * 8
  • 16. CHUYEÂN ÑEÀ 4 ÑÖÔØNG TROØN 1. Ñeå tìm phöông trình cuûa moät ñöôøng troøn ta caàn löu yù: . Phöông trình cuûa ñöôøng troøn (C) taâm I(a, b) baùn kính R laø : ( ) + ( = R22 x a− ) 2 y b− . Phöông trình cuûa (C) ôû daïng khai trieån : x2 + y2 – 2ax – 2by + c = 0 ( hay x2 + y2 + 2ax + 2by + c = 0) vôùi c = a2 + b2 – R2 R2 =⇔ 2 2 a b c+ − Do ñoù ta phaûi coù ñieàu kieän a2 + b2 – c 0≥ . Phöông trình tham soá cuûa ñöôøng troøn taâm I(a, b) baùn kính R laø: (t x a R cost y b R sin t = +⎧ ⎨ = +⎩ ∈ R) 2. Ñeå vieát phöông trình tieáp tuyeán vôùi moät ñöôøng troøn ta caàn phaân bieät : a) Tröôøng hôïp bieát tieáp ñieåm : ta duøng coâng thöùc phaân ñoâi toïa ñoä : Tieáp tuyeán ( taïi tieáp ñieåm M0(x0, y0) vôùi :)Δ - ñöôøng troøn (C) : ( ) + = R2 laø 2 x a− ( 2 y b− ) ) (x0 – a) (x – a) + (y0 – b) (y – b) = R2 - ñöôøng troøn (C) : x2 + y2 – 2ax – 2by + c = 0 laø x0x + y0y – a(x0 + x) – b(y0 + y) + c = 0 b) Tröôøng hôïp khoâng bieát tieáp ñieåm, ta aùp duïng tính chaát : Ñöôøng thaúng ( tieáp xuùc vôùi ñöôøng troøn taâm I baùn kính R)Δ ⇔ = R.Δd(I, ) c) ñöôøng troøn (C) : ( ) + = R2 coù 2 tieáp tuyeán cuøng phöông vôùi Oy laø x = a R. Ngoaøi 2 tieáp tuyeán x = a 2 x a− ( 2 y b− ± ± R, moïi tieáp tuyeán khaùc vôùi ñöôøng troøn ( C) ñeàu coù daïng y = kx + m hoaëc daïng y = k ( x –x0 ) + y0 neáu tieáp tuyeán ñi qua ( x0 , y0 ) laø ñieåm naèm ngoaøi ñöôøng troøn. Ví duï 1
  • 17. Trong maët phaúng Oxy cho A(–2, 0), B(0, 4). a) Vieát phöông trình ñöôøng troøn (C) qua 3 ñieåm O, A, B. b) Vieát phöông trình caùc tieáp tuyeán vôùi ñöôøng troøn (C) taïi A, B. c) Vieát phöông trình caùc tieáp tuyeán vôùi (C) phaùt xuaát töø ñieåm M(4, 7) Giaûi a) Phöông trình ñöôøng troøn (C) coù daïng : x2 + y2 – 2ax – 2by + c = 0 Ñöôøng troøn (C) qua 3 ñieåm O, A, B neân : 0 4 4 0 16 8 0 c a c b c =⎧ ⎪ + + =⎨ ⎪ − + =⎩ ⇔ 0 1 2 c a b =⎧ ⎪ = −⎨ ⎪ =⎩ Vaäy (C) : x2 + y2 + 2x – 4y = 0. Caùch khaùc: Tam giaùc ABC vuoâng taïi O neân coù taâm laø trung ñieåm cuûa AB vaø ñöôøng kính laø AB neân pt döôøng troøn (C) laø: 2 2 21 1 1 2 4 16 4 4 + + − = = + =(x ) ( y ) AB ( ) 5 Caùch khaùc: Tam giaùc ABC vuoâng taïi O neân vôùi ( , ) ( )M x y C∈ ta coù 0=AM.BM . Vaäy pt ñöôøng troøn ( C ) laø 0− − + − − =A B A B(x x )(x x ) ( y y )( y y ) . b) Phöông trình tieáp tuyeán vôùi (C) taïi : . Tieáp ñieåm A(–2, 0) laø : –2x + 0.y + (–2 + x) – 2(0 + y) = 0 ⇔ x + 2y + 2 = 0 . Tieáp ñieåm B(0, 4) laø : 0.x + 4.y + (0 + x) – 2(4 + y) = 0 ⇔ x + 2y – 8 = 0 c) Ñöôøng troøn (C) : x2 + y2 + 2x – 4y = 0 coù taâm I(–1, 2) vaø baùn kính R = 2 1 2 0+ − = 5 .Hai tieáp tuyeán cuøng phöông vôùi Oy laø 1= ± = − ±x a R 5 . Hai tieáp tuyeán naøy khoâng qua M(4, 7) Vaäy phöông trình tieáp tuyeán qua M(4, 7) coù daïng: ( )Δ : y – 7 = k(x – 4) ⇔ kx – y + 7 – 4k = 0 ( )Δ tieáp xuùc vôùi ñöôøng troøn (C) ⇔ Δd(I, ) = R 2
  • 18. ⇔ 2 2 7 4 1 k k k − − + − + = 5 ⇔ 5 5k− = 5 . 2 1k + ⇔ 4k2 – 10k + 4 = 0 ⇔ k = 2 hay k = 1 2 Vaäy coù 2 tieáp tuyeán vôùi ñöôøng troøn (C) phaùt xuaát töø ñieåm M(4, 7) vôùi phöông trình laø : k = 2 2x – y – 1 = 0⇒ k = 1 2 ⇒ 1 2 x – y + 5 = 0. Ví duï (ÑH KHOÁI B-2003) Trong maët phaúng vôùi heä toïa ñoä Ñeâcac vuoâng goùc Oxy cho tam giaùc ABC coù AB=AC, 0 90BAC = . Bieát M(1,–1) laø trung ñieåm caïnh BC vaø G( 2 3 ; 0) laø troïng taâm tam giaùc ABC. Tìm toïa ñoä caùc ñænh A , B, C. G laø troïng taâm ΔABC ⇔ =AG 2GM ⇔ ⎧ − = − =⎪ ⎨ ⎪− = − − = −⎩ A A 2 2 2 3 =x 2(1 ) 3 3 y 2( 1 0) 2 ⇔ ⎧ ⎨ =⎩ A A x 0 2 ⇔ A (0, 2) y PT: BC qua M (1, −1) ⊥ = (1, −3): x – 3y – 4 = 0AM PT ñ.troøn (C) taâm M, baùn kính R = AM= + =1 9 10 (x – 1)2 + (y + 1)2 = 10 Toïa ñoä B, C thoûa : − − =⎧ ⎨ − + + =⎩ 2 2 x 3y 4 0 (x 1) (y 1) 10 ⇔ ⇔ = +⎧ ⎨ + + + = ⇔ + =⎩ 2 2 2 x 3y 4 (3y 3) (y 1) 10 (y 1) 1 =⎧ ⎨ =⎩ x 4 y 0 ∨ = −⎧ ⎨ = −⎩ x 2 y 2 Vaäy B (4, 0); C(−2, −2) hay B(−2, −2); C (4, 0) Ví duï (ÑH KHOÁI D-2003) Trong maët phaúng vôùi heä toïa ñoä Ñecac vuoâng goùc Oxy cho ñöôøng troøn (C): (x – 1)2 + (y – 2)2 = 4 vaø ñöôøng thaúng d: x – y – 1 = 0. Vieát phöông trình ñöôøng troøn (C’) ñoái xöùng vôùi ñöôøng troøn (C) qua ñöôøng thaúng d. Tìm toïa ñoä caùc giao ñieåm (C) vaø (C’) Giaûi (C1) coù taâm I (1, 2), R = 2. Goïi I’ laø ñoái xöùng I qua (d) Goïi (Δ) laø ñöôøng thaúng qua I vaø (Δ) ⊥ (d) (Δ) : x + y – 3 = 0. (Δ) ∩ (d) = H(2, 1) H laø trung ñieåm cuûa II’ Giaû söû I’ (x, y) thì ⇒ +⎧ =⎪⎪ ⎨ +⎪ = ⎪⎩ x 1 2 2 y 2 1 2 ⇒ =⎧ ⎨ =⎩ x 3 y 0 ⇒ I’ (3, 0); R’ = R = 2. (C’) : (x – 3)2 + y2 = 4 3
  • 19. Giaûi heä ⎧ ⇔ − + − =⎪ ⎨ − + =⎪⎩ 2 2 2 2 (x 1) (y 2) 4 (x 3) y 4 ⎧ − + = ⎨ − − =⎩ 2 2 (x 3) y 4 x y 1 0 ⇔ ⇔ ∨ = +⎧ ⎨ − =⎩ 2 x y 1 2y 4y 0 =⎧ ⎨ =⎩ x 1 y 0 =⎧ ⎨ =⎩ x 3 y 2 Vaäy giao ñieåm cuûa (C) vaø (C’) laø A (1, 0) vaø B (3, 2). Ví duï (ÑH KHOÁI A-2005) Trong maët phaúng vôùi heä toïa ñoä Oxy, cho hai ñöôøng thaúng d1 : x – y = 0 vaø d2 : 2x + y – 1 = 0.Tìm toïa ñoä caùc ñænh hình vuoâng ABCD bieát raèng ñænh A thuoäc d1, ñænh C thuoäc d2 vaø caùc ñænh B, D thuoäc truïc hoaønh. Giaûi A ∈ d1 ⇔ A (m; m). C ∈ d2 ⇔ C (n; 1 – 2n) Vì B, D ∈ Ox vaø ABCD laø hình vuoâng neân : A vaø C ñoái xöùng nhau qua Ox ⇔ m n m 2n 1 =⎧ ⎨ = −⎩ ⇔ m 1 n 1 =⎧ ⎨ =⎩ Suy ra A(1; 1), C(1; -1). Goïi (C) laø ñöôøng troøn ñöôøng kính AC ⇒ Phöông trình (C) : (x–1)2 +y2 =1. B vaø D laø giao ñieåm (C) vaø Ox neân toïa ñoä cuûa B, D laø nghieäm cuûa heä : 2 2(x 1) y 1 y 0 ⎧⎪ − + = ⎨ =⎪⎩ ⇔ . Suy ra B (0; 0), D(2; 0) hay B(2; 0), D(0; 0) = ∨ =⎧ ⎨ =⎩ x 0 x 2 y 0 Vaäy A(1; 1), B (0; 0), C(1; -1), D(2; 0) hay A(1; 1), B(2; 0), C(1; -1), D(0; 0). Ví duï (ÑH KHOÁI B-2005)Trong maët phaúng vôùi heä toïa ñoä Oxy, cho hai ñieåm A(2; 0), B(6; 4). Vieát phöông trình ñöôøng troøn (C) tieáp xuùc vôùi truïc hoaønh taïi ñieåm A vaø khoaûng caùch töø taâm cuûa (C) ñeán ñieåm B baèng 5. Giaûi Goïi I (x; y) laø taâm cuûa (C). Ta coù : (C) tieáp xuùc Ox taïi A ⇒ IA i⊥ = (1; 0) ⇔ x – 2 = 0 ⇔ x = 2 IB = 5 ⇔ (x – 6)2 + (y – 4)2 = 25 ⇔ (2 – 6)2 + (y – 4)2 = 25 ⇔ (y – 4)2 = 9 ⇔ y – 4 = ±3 ⇔ y = 7 hay y = 1 Tröôøng hôïp 1: I(2; 7) ⇒ R = d(I, Ox) = 7 Suy ra pt (C) : (x – 2)2 + (y – 7)2 = 49 Tröôøng hôïp 2: I (2; 1) ⇒ R = d(I, Ox) = 1 ⇒ pt (C) : (x – 2)2 + (y – 1)2 = 1. Ví duï (ÑEÀ DÖÏ BÒ KHOÁI A -2002) Trong maët phaúng vôùi heä toïa ñoä Ñeàcac vuoâng goùc Oxy, cho hai ñöôøng troøn: (C1) : x2 + y2 – 10x = 0; (C2) : x2 + y2 + 4x – 2y – 20 = 0 4
  • 20. 1) Vieát phöông trình ñöôøng troøn ñi qua caùc giao ñieåm cuûa (C1), (C2) vaø coù taâm naèm treân ñöôøng thaúng x + 6y – 6 = 0. 2) Vieát phöông trình tieáp tuyeán chung cuûa caùc ñöôøng troøn (C1) vaø (C2). Giaûi 1) Phöông trình chuøm ñöôøng troøn qua caùc giao ñieåm cuûa (C1), (C2) laø : m(x2 + y2 – 10x) + n(x2 + y2 + 4x – 2y – 20) = 0 vôùi m2 + n2 > 0 ⇔ (m + n)x2 + (m + n)y2 + (4n – 10m)x – 2ny – 20n = 0 ⇔ x2 + y2 + 4n 10m 2n 20n x y m n m n m n −⎛ ⎞ − −⎜ ⎟ + + +⎝ ⎠ 0= Coù taâm I 5m 2n n ; m n m n −⎛ ⎞ ⎜ ⎟ + +⎝ ⎠ Vì taâm I ∈ d : x + 6y – 6 = 0 ⇒ 5m 2n 6n 6m 6n 0 m n − + − − = + ⇒ m = −2n . Cho n = 1 ⇒ m = −2 Vaäy phöông trình ñöôøng troøn laø :x2 + y2 – 24x + 2y + 20 = 0. 2) Vieát phöông trình caùc tieáp tuyeán chung cuûa (C1), (C2). (C1) coù taâm I1(5; 0), baùn kính R1 = 5 ⇒ I1I2 < R1 + R2 (C2) coù taâm I2(−2; 1), baùn kính R2 = 5 Vì (C1), (C2) caét nhau taïi 2 ñieåm neân coù 2 tieáp tuyeán chung. Vì x = xo khoâng theå laø tieáp tuyeán chung neân pt tt chung Δ coù daïng : y = ax + b ⇔ ax – y + b = 0 Δ tieáp xuùc vôùi (C1) ⇔ d(I1, Δ) = R1 ⇔ 2 5a b 5 a 1 ⏐ + ⏐ = + ⇔⏐5a + b⏐ = 2 5 a 1+ (1) Δ tieáp xuùc vôùi (C2) ⇔ d(I2, Δ) = R2 ⇔ 2 2a 1 b a 1 ⏐− − + ⏐ + = 5 ⇔ ⏐−2a – 1 + b⏐ = 2 5 a (2)1+ (1) vaø (2) ⇒ ⏐5a + b⏐ = ⏐−2a – 1 + b⏐ ⇔ ⇔ 5a b 2a 1 b 5a b 2a 1 b + = − − +⎡ ⎢ + = + + −⎣ 1 a 7 3a 1 b 2 ⎡ = −⎢ ⎢ − +⎢ = ⎢⎣ Theá a = 1 7 − vaøo (1) ta coù : b1 = 5 25 2 7 + ; b2 = 5 25 2 7 − Vaäy ta coù 2 tieáp tuyeán laø : x + 7y – 5 + 25 2 = 0 x + 7y – 5 − 25 2 = 0. Caùch khaùc: Vì R = R2 vaø 2 ñöôøng troøn caét nhau neân 2 tieáp tuyeán chung laø 2 ñöôøng thaúng song song vôùi Vaäy phöông trình 2 tieáp tuyeán coù daïng : 1 1 2I I ( 7;1)= − x + 7y+m = 0 (Δ) d(I1, Δ) = 5 ⇔ ⏐5 + m⏐ = +2 5 7 1 ⇔ m = – 5 ± 25 2 Vaäy phöông trình 2 tieáp tuyeán laø x + 7y – 5 ± 25 2 = 0. 5
  • 21. GHI CHUÙ : Baøi ñöôøng troøn trong chöông trình lôùp 12 bao goàm caùc vaán ñeà chính laø : Tìm phöông trình ñöôøng troøn; caùc baøi toaùn lieân quan ñeán vò trí töông ñoái giöõañöôøng thaúng vaø ñöôøng troøn, giöõa hai ñöôøng troøn; phöông tích cuûa moät ñieåm ñoái vôùi ñöôøng troøn; truïc ñaúng phöông cuûa hai ñöôøng troøn khoâng ñoàng taâm. Ngoaøi ra coøn coù moät soá caâu hoûi lieân quan ñeán phöông trình x2 + y2 + 2Ax + 2By +C = 0 (1). Chaúng haïn tìm ñieàu kieän ñeå (1) laø phöông trình ñöôøng troøn. Töø phöông trình (1) tìm taâm vaø baùn kính cuûa ñöôøng troøn, tìm tham soá ñeå baùn kính thoaû moät ñieàu kieän naøo ñoù . . . Sau ñaây, chuùng toâi chæ ñeà caäp ñeán caùch tìm phöông trình ñöôøng troøn noäi tieáp tam giaùc vaø vaøi öùng duïng truïc ñaúng phöông cuûa hai ñöôøng troøn khoâng ñoàng taâm. Ñaây laø vaán ñeá caùc em thöôøng “ sôï” khi gaëp phaûi. A/ Caùch tìm phöông trình ñöôøng troøn noäi tieáp tam giaùc ABC : Tröôùc heát caàn löu yù : • Taâm ñöôøng troøn noäi tieáp tam giaùc laø giao ñieåm cuûa hai ñöôøng phaân giaùc trong . • Muoán tìm phöông trình ñöôøng troøn ta tìm taâm I (a ; b) vaø baùn kính R. Khi ñoù phöông trình ñöôøng troøn coù daïng (x – a)2 + (y – b)2 = R2 . • Cho k laø soá thöïc khaùc 1, ta coù : ⎪ ⎪ ⎩ ⎪⎪ ⎨ ⎧ − − = − − = ⇔= k1 kyy y k1 kxx x MBkMA BA M BA M (I) 1/ Neáu ñeà baøi cho bieát toïa ñoä A, B, C thì : • Goïi D laø chaân ñöôøng phaân giaùc trong keû töø A cuûa tam giaùc ABC. Ta coù : DC AC AB DB −= Söû duïng coâng thöùc (I) vôùi k = AC AB − ta xaùc ñònh ñöôïc toïa ñoä ñieåm D. A B CD I • Goïi I laø taâm ñöôøng troøn noäi tieáp tam giaùc ABC thì I chính laø chaân ñöôøng phaân giaùc trong keû töø B cuûa tam giaùc ABD. Ta coù : ID BD BA IA −= Söû duïng coâng thöùc (I) vôùi k = BD BA − laø xaùc ñònh ñöôïc toïa ñoä taâm I. Coøn baùn kính ñöôøng troøn noäi tieáp tam giaùc chính laø khoaûng caùch töø taâm I ñeán moät trong 3 caïnh cuûa tam giaùc ABC. Chuù yù : Neáu moät trong ba ñænh cuûa tam giaùc truøng vôùi goác toïa ñoä vaø hai ñænh coøn laïi naèm treân hai truïc toïa ñoä thì caùch giaûi ñöôïc thu goïn hôn vì bieát tröôùc ñöôïc 1 ñöôøng phaân giaùc trong keû töø goác toïa ñoä. Ñöôøng phaân giaùc coøn laïi ñöôïc tìm thoâng qua tìm chaân ñöôøng phaân giaùc trong nhö ñaõ trình baøy ôû treân. 6
  • 22. 2/ Neáu ñeà baøi cho bieát phöông trình 3 caïnh cuûa tam giaùc ABC thì töø phöông trình 3 caïnh ñoù, ta tìm ñöôïc toïa ñoä caùc ñieåm A, B, C baèng caùch giaûi heä phöông trình toïa ñoä giao ñieåm vaø söû duïng caùch giaûi nhö phaàn 1. Ngoaøi ra coøn coù theå giaûi baèng kieán thöùc mieàn taïo bôûi 1 ñöôøng thaúng vaø khoaûng caùch ñaïi soá töø moät ñieåm ñeán ñöôøng thaúng. B/ Truïc ñaúng phöông cuûa hai ñöôøng troøn khoâng ñoàng taâm : 1/ Cho hai ñöôøng troøn khoâng ñoàng taâm : (C1) : x2 + y2 + 2a1x + 2b1y + c1 = 0 (1) (C2) : x2 + y2 + 2a2x + 2b2y + c2 = 0 (2) Truïc ñaúng phöông cuûa (C1) vaø (C2) laø taäp hôïp caùc ñieåm coù cuøng phöông tích ñoái vôùi (C1) vaø (C2) vaø coù phöông trình laø : 2(a1 – a2)x + 2(b1 – b2)y + c1 – c2 = 0 2/ ÖÙng duïng : Trong chöông trình Hình hoïc lôùp 10 ta ñaõ bieát caùch döïng truïc ñaúng phöông cuûa (C1) vaø (C2). • Neáu (C1) vaø (C2) caét nhau taïi 2 ñieåm A vaø B thì truïc ñaúng phöông cuûa (C1) vaø (C2) laø ñöôøng thaúng AB. • Neáu (C1) vaø (C2) tieáp xuùc nhau (Tieáp xuùc trong hoaëc tieáp xuùc ngoaøi) thì truïc ñaúng phöông cuûa (C1) vaø (C2) laø tieáp tuyeán chung cuûa (C1) vaø (C2) taïi tieáp ñieåm. • Neáu (C1) vaø (C2) khoâng caét nhau thì veõ theâm ñöôøng troøn (C3) sao cho caét ñöôïc (C1), (C2) vaø coù taâm khoâng naèm treân ñöôøng noái taâm cuûa (C1), (C2). Goïi M laø giao ñieåm cuûa hai truïc ñaúng phöông cuûa (C1) vaø (C3), (C2) vaø (C3). Khi ñoù truïc ñaúng phöông cuûa (C1) vaø (C2) laø ñöôøng thaúng qua M vaø vuoâng goùc vôùi ñöôøng noái taâm cuûa (C1) vaø (C2). Baøi toaùn : Cho ñöôøng troøn (C) vaø M laø ñieåm naèm ngoaøi (C). Töø M keû MA vaø MB laø hai tieáp tuyeán cuûa (C) (A vaø B laø hai tieáp ñieåm). Vieát phöông trình ñöôøng thaúng AB. Caùch giaûi : Goïi I laø taâm vaø R laø baùn kính cuûa ñöôøng troøn (C). Goïi (C’) laø ñöôøng troøn taâm M, baùn kính : R’ = MA = 22 RIM − Suy ra (C) vaø (C’) caét nhau taïi A vaø B. Do ñoù ñöôøng thaúng AB chính laø truïc ñaúng phöông cuûa (C) vaø (C’). (C) (C’)A B MI Qua keát quaû treân ta ghi nhôù ngay 2 keát quaû : • Ñöôøng thaúng ñi qua giao ñieåm cuûa hai ñöôøng troøn (C1) vaø (C2) chính laø truïc ñaúng phöông cuûa (C1) vaø (C2) [Nghóa laø khoâng caàn tìm toïa ñoä giao ñieåm cuûa (C1) vaø (C2)]. • Tieáp tuyeán chung cuûa 2 ñöôøng troøn (C1) vaø (C2) tieáp xuùc nhau taïi tieáp ñieåm chính laø truïc ñaúng phöông cuûa (C1) vaø (C2). Sau ñaây, löu yù theâm 2 baøi toaùn thöôøng gaëp : Baøi 1 : Cho (C1) vaø (C2) ôû ngoaøi nhau. Tìm quyõ tích nhöõng ñieåm M töø ñoù veõ ñöôïc ñeán (C1) vaø (C2) nhöõng ñoaïn tieáp tuyeán baèng nhau. Caùch giaûi : Goïi MA vaø MB (nhö hình veõ) laø 2 tieáp tuyeán töø M ñeán (C1) vaø (C2) Ta coù : MA = MB ⇔ MA2 = MB2 ⇔ 1 2/( ) /( )M C M CP P= Do ñoù quyõ tích M laø truïc ñaúng phöông cuûa (C1) vaø (C2). • 7 M • BA • (C1) (C2)
  • 23. Baøi 2 : Tìm tieáp ñieåm M cuûa hai ñöôøng troøn tieáp xuùc nhau (C1) vaø (C2) Goïi I1 vaø I2 laø taâm cuûa (C1) vaø (C2). Tieáp ñieåm M chính laø giao ñieåm cuûa truïc ñaúng phöông cuûa (C1) vaø (C2) vôùi ñöôøng noái taâm I1I2. (C2) (C1) MI1 I2 d Ví duï (ÑEÀ DÖÏ BÒ KHOÁI B -2005) Trong maët phaúng vôùi heä toïa ñoä Oxy cho 2 ñöôøng troøn : (C1 ): x2 + y2 vaø (C2 ): x2 + y2 . Vieát phöông trình truïc ñaúng phöông d cuûa 2 ñöôøng troøn (C1) vaø (C2). Chöùng minh raèng neáu K thuoäc d thì khoûang caùch töø K ñeán taâm cuûa (C1) nhoû hôn khoûang caùch töø K ñeán taâm cuûa ( C2 ). 9= 2 2 23x y− − − = 0 Giaûi: Ñöôøng troøn ( )1C coù taâm ( )O 0 baùn kính R 3,0 1 = Ñöôøng troøn ( )2C coù taâm ( )I 1 , baùn kính,1 2R 5= Phöông trình truïc ñaúng phöông cuûa 2 ñöôøng troøn ( )1C , ( )2C laø ( ) ( )2 2 2 2 x y 9 x y 2x 2y 23+ − − + − − − = 0 x y 7 0⇔ + + = (d) Goïi ( ) ( )k k k kK x ,y d y x 7∈ ⇔ = − − ( ) ( ) ( )= − + − = + = + − − = + + 2 2 22 2 2 2 2 k k k k k k k kOK x 0 y 0 x y x x 7 2x 14x 49 ( ) ( ) ( ) ( )2 2 2 22 2 k k k k k kIK x 1 y 1 x 1 x 8 2x 14x 6= − + − = − + − − = + + 5 Ta xeùt ( ) ( )2 2 2 2 k k k kIK OK 2x 14x 65 2x 14x 49 16 0− = + + − + + = > K OK IK OK(ñpcm)> ⇔ >Vaäy I 2 2 * * * 8
  • 24. CHUYEÂN ÑEÀ 5 ELIP Caùc baøi toaùn veà elip chuû yeáu qui veà vieäc vieát phöông trình chính taéc cuûa elip, xaùc ñònh caùc phaàn töû cuûa elip (taâm, ñænh, tieâu cöï, ñoä daøi truïc lôùn, truïc nhoû, tieâu ñieåm…), nhaát laø xaùc ñònh phöông trình cuûa tieáp tuyeán cuøng vôùi toïa ñoä tieáp ñieåm. Trong moïi tröôøng hôïp ta caàn naém vöõng kieán thöùc cô baûn sau ñaây : . Elip (E) coù tieâu ñieåm treân x′x . Elip (E) coù tieâu ñieåm treân y′y Phöông trình chính taéc Tieâu cöï Tieâu ñieåm Truïc lôùn Truïc nhoû Ñænh treân truïc lôùn Ñænh treân truïc nhoû Taâm sai Baùn kính qua tieâu Ñieåm cuûa M ∈ (E) Ñöôøng chuaån (E) : 2 2 x a + 2 2 y b = 1 a2 > b2 vaø a2 – b2 = c2 2c F1(–c, 0), F2(c, 0) Treân Ox, daøi 2a Treân Oy, daøi 2b A1(–a, 0), A2(a, 0) B1(0, –b), B2(0, b) e = c a 1 1 2 2 M M r FM a ex r F M a ex = = +⎧ ⎨ = = −⎩ 1 2,Δ : x = ± a e (E) : 2 2 x a + 2 2 y b = 1 a2 < b2 vaø b2 – a2 = c2 2c F1(0, –c), F2(0, c) Treân Oy, daøi 2b Treân Ox, daøi 2a A1(0, –b), A2(0, b) B1(–a, 0), B2(a, 0) e = c b 1 1 2 2 M M r FM b ey r F M b ey = = +⎧ ⎨ = = −⎩ 1 2,Δ : y = ± b e * Ghi chuù : 1
  • 25. Tröôøng hôïp elip coù taâm I( ,α β ) hai truïc cuøng phöông vôùi 2 truïc toïa ñoä thì phöông trình coù daïng ( ) 2 2 x a − α + ( ) 2 2 y b − β = 1 Ta dôøi heä truïc toïa ñoä xOy ñeán XIY baèng pheùp tònh tieán theo OI ñeå ñöôïc phöông trình daïng chính taéc cuûa elip laø 2 2 X a + 2 2 Y b = 1 vôùi X x Y y = − α⎧ ⎨ = − β⎩ ñeå suy ra deã daøng toïa ñoä caùc ñænh vaø tieâu ñieåm. . Tieáp tuyeán vôùi elip (E) : 2 2 x a + 2 2 y b = 1 taïi tieáp ñieåm M0(x0, y0) coù phöông trình 0 2 x x a + 0 2 y y b = 1 . Tröôøng hôïp khoâng bieát tieáp ñieåm ta aùp duïng tính chaát : : Ax + By + C = 0 tieáp xuùc vôùi elip( )Δ (E) : 2 2 x a + 2 2 y b = 1 a2 A2 + b2 B2 = C2 ⇔ Thöôøng ta vieát phöông trình cuûa ( )Δ theo heä soá goùc ôû daïng kx – y + c = 0 vaø löu yù tröôøng hôïp ( )Δ ⊥ x′x töùc ( )Δ : x = ± a . Elip (E) : 2 2 x a + 2 2 y b = 1 coù 2 tieáp tuyeán cuøng phöông vôùi Oy laø x = a. Ngoaøi 2 tieáp tuyeán x = a, moïi tieáp tuyeán khaùc vôùi ( E) ñeàu coù daïng± ± y = kx + m hoaëc daïng y = k ( x –x0 ) + y0 neáu tieáp tuyeán ñi qua ( x0 , y0 ) laø ñieåm naèm ngoaøi elip. Ví duï1 : Cho elip (E) : x2 + 4y2 – 40 = 0 a) Xaùc ñònh tieâu ñieåm, hai ñænh treân truïc lôùn, 2 ñænh treân truïc nhoû vaø taâm sai cuûa (E). b) Vieát phöông trình tieáp tuyeán vôùi (E) taïi ñieåm M0(–2, 3). c) Vieát phöông trình tieáp tuyeán vôùi elip (E) bieát noù xuaát phaùt töø ñieåm M(8, 0). 2
  • 26. d) Vieát phöông trình tieáp tuyeán vôùi (E) bieát noù vuoâng goùc vôùi ñöôøng thaúng (D) : 2x – 3y + 1 = 0, tính toïa ñoä tieáp ñieåm. Giaûi a) Tieâu ñieåm, caùc ñænh vaø taâm sai cuûa (E) (E) : x2 + 4y2 – 40 = 0 ⇔ 2 x 40 + 2 10 y = 1 coù daïng 2 2 x a + 2 2 y b = 1 vôùi a2 = 40 > b2 = 10 c2 = a2 – b2 = 30⇒ a = 2⇒ 10 , b = 10 , c = 30 Vaäy elip (E) coù truïc lôùn treân Ox, hai tieâu ñieåm naèm treân truïc lôùn laø F1(– 30 , 0) , F2( 30 , 0). Hai ñænh treân truïc lôùn laø A1(–2 10 , 0), A2(2 10 , 0) Truïc nhoû cuûa (E) naèm treân Oy vôùi 2 ñænh laø B1(0, – 10 ), B2(0, 10 ). Taâm sai cuûa elip (E) laø e = c a = 30 2 10 = 3 2 b) Vieát phöông trình tieáp tuyeán vôùi (E) taïi M0(–2, 3) Ta coù + 4 – 40 = (2 0x 2 0y ) 2 2− + 4 – 40 = 0( ) 2 3 M0(–2, 3) ∈ (E) : x2 + 4y2 – 40 = 0⇒ Phöông trình tieáp tuyeán vôùi (E) taïi tieáp ñieåm M0(–2, 3) seõ laø:⇒ x0x + 4y0y – 40 = 0 ⇔ –2x + 12y – 40 = 0 ⇔ x - 6y + 20 = 0 c) Phöông trình tieáp tuyeán vôùi elip phaùt xuaát töø M(8, 0). (E) coù hai tieáp tuyeán cuøng phöông vôùi 0y laø: x = 2 10± .Hai tieáp tuyeán naøy khoâng ñi qua M(8,0). Vaäy pt tieáp tuyeán ( qua M(8, 0) coù daïng:)Δ y= k(x – 8) ⇔ kx – y – 8k = 0 ( )Δ tieáp xuùc vôùi elip (E) : 2 x 40 + 2 y 10 = 1 ⇔ 40k2 + 10 = 64k2 3
  • 27. ⇔ k2 = 10 24 = 5 12 ⇔ k = ± 5 2 3 = ± 15 6 Vaäy coù 2 tieáp tuyeán vôùi (E) qua M(8, 0) laø : 15 6 x – y – 8 5 6 = 0 ⇔ 15 x – 6y – 8 5 = 0 hay – 15 6 x – y + 8 5 6 = 0 ⇔ 15 x + 6y – 8 5 = 0 d) Phöông trình tieáp tuyeán vôùi (E) vaø vuoâng goùc vôùi (D) (D) vôùi (D) : 2x – 3y + 1 = 0( )′Δ ⊥ ⇒ : 3x + 2y + C = 0( )′Δ ( )′Δ tieáp xuùc (E) : 2 x 40 + 2 y 10 = 1 ⇔ 40.9 + 10.4 = C2 ⇔ C2 = 400 ⇔ C = ± 20 Goïi M0(x0, y0) laø tieáp ñieåm cuûa tieáp tuyeán ( )′Δ vôùi (E) thì ( )′Δ : 0x x 40 + 0y y 10 = 1 ⇔ x0x + 4y0y – 40 = 0 Vôùi C = 20 : 3x + 2y + 20 = 0⇒ ( )′Δ ⇒ 0x 3 = 04y 2 = 40 20 − ⇔ 0 0 x 6 y 1 = −⎧ ⎨ = −⎩ hay M0 (–6, –1) Vôùi C = –20 (⇒ )′Δ : 3x + 2y – 20 = 0 ⇒ 0x 3 = 04y 2 = 40 20 − − ⇔ hay M0(6, 1). 0 0 x 6 y 1 =⎧ ⎨ =⎩ 4
  • 28. Ví duï2 :(ÑH KHOÁI D-2005) Trong maët phaúng vôùi heä toïa ñoä Oxy cho ñieåm C (2; 0) vaø elíp (E) : 2 2 x y 1 4 1 + = . Tìm toïa ñoä caùc ñieåm A, B thuoäc (E), bieát raèng hai ñieåm A, B ñoái xöùng vôùi nhau qua truïc hoaønh vaø tam giaùc ABC laø tam giaùc ñeàu Giaûi Giaû söû A (a, 2 4 a 2 − ) ∈ (E) ⇒ B (a, − 2 4 a 2 − ) ∈ (E) Vaø ñieàu kieän: –2 < a < 2. Do A,B ñoái xöùng qua Ox neân ta coù: ΔCAB ñeàu ⇔ CA2 = AB2 ⇔ (a – 2)2 + 2 4 a 4 − = 4 – a2 ⇔ 7a2 – 16a + 4 = 0 ⇔ a = 2 (loaïi) hay a = 2 7 . Neân toïa ñoä cuûa A vaø B laø: A 2 4 3 , 7 7 ⎛ ⎞ ⎜ ⎟⎜ ⎟ ⎝ ⎠ vaø B 2 4 3 , 7 7 ⎛ −⎜⎜ ⎝ ⎠ ⎞ ⎟⎟ hoaëc A 2 4 3 , 7 7 ⎛ ⎞ −⎜ ⎟⎜ ⎟ ⎝ ⎠ vaø B 2 4 3 , 7 7 ⎛ ⎞ ⎜ ⎟⎜ ⎟ ⎝ ⎠ Ví duï3 :(ÑH KHOÁI D-2002) : Cho(E) : 9 y 16 x 22 + = 1. Cho M di chuyeån treân tia 0x, N di chuyeån treân tia 0y sao cho ñöôøng thaúng MN luoân tieáp xuùc (E). Tìm toïa ñoä ñieåm M, N sao cho ñoä daøi ñoaïn MN ngaén nhaát. Tìm ñoä daøi ñoaïn ngaén nhaát ñoù. Giaûi M (m, 0) ∈ tia Ox; N (0, n) ∈ tia Oy ⇒ n, m > 0 (E) : 9 y 16 x 22 + = 1. MN : nx + my – n.m = 0 (MN) tieáp xuùc (E) ⇔ 1 n 9 m 16 22 =+ Ta coù : MN2 = m2 + n2 .Theo BÑT BCS ta coù Ta coù : 7 = MNnm n 9 m 16 n. n 3 m. m 4 22 22 =++≤+ MN nhoû nhaát ⇒ n 3 n m 4 m = ⇔ 3 n 4 m 22 = ⇔ 3m2 = 4n2 vaø m2 + n2 = 49 ⇔ m2 = 28 vaø n2 = 21 Do ñoù : MN nhoû nhaát ⇔ m = 72 vaø n = 21 (vì m, n>0) ⇒ M ( 72 , 0); N (0, 21 ). Khi ñoù min MN = 7. Ví duï4 :(ÑH KHOÁI D-2005) Trong maët phaúng vôùi heä toïa ñoä Ñeàcac vuoâng goùc Oxy, cho elip (E): 1 4 y 9 x 22 =+ vaø ñöôøng thaúng dm : mx – y – 1 = 0. 5
  • 29. a) Chöùng minh raèng vôùi moïi giaù trò cuûa m, ñöôøng thaúng dm luoân caét elip (E) taïi hai ñieåm phaân bieät. b) Vieát phöông trình tieáp tuyeán cuûa (E), bieát raèng tieáp tuyeán ñoù ñi qua ñieåm N (1; −3). Giaûi a) (E) : 2 2 x y 1 9 4 + = ⇔ 4x2 + 9y2 – 36 = 0 (dm) : mx – y – 1 = 0 ⇔ y = mx – 1 Phöông trình hoaønh ñoä giao ñieåm cuûa (dm) vôùi (E) : 4x2 + 9(mx – 1)2 – 36 = 0 ⇔ (4 + 9m2 )x2 – 18mx – 25 = 0 coù Δ' = 81m2 + 25(4 + 9m2 ) > 0 ñuùng vôùi moïi m Vaäy (dm) luoân luoân caét (E) taïi 2 ñieåm phaân bieät. b) Vieát phöông trình tieáp tuyeán vôùi (E) qua N(1; −3) 2 tieáp tuyeán thaúng ñöùng cuûa (E) laø x = ± 3 ( khoâng qua N ) Goïi Δ laø tieáp tuyeán qua N(1; −3) thì phöông trình Δ coù daïng: y + 3 = k(x – 1) ⇔ kx – y – 3 – k = 0 (Δ) tieáp xuùc vôùi (E) ⇔ 9k2 + 4 = (−3 – k)2 = 9 + 6k + k2 ⇔ 8k2 – 6k – 5 = 0 ⇔ 1 2 1 k 2 5 k 4 ⎡ = −⎢ ⎢ ⎢ =⎢⎣ Δ1 : x + 2y + 5 = 0; Δ2 : 5x – 4y – 17 = 0. * * * 6
  • 30. CHUYEÂN ÑEÀ 6 HYPEBOL Ñeå giaûi caùc baøi toaùn coù lieân quan ñeán ñöôøng hypebol ta caàn naém vöõng caùc vaán ñeà cô baûn sau: Hypebol (H) coù taâm O, hai truïc ñoái xöùng laø x′x, y′y. Phöông trình chính taéc . Hypebol coù tieâu ñieåm treân x′x 2 2 x a – 2 2 y b = 1 . Hypebol coù tieâu ñieåm t reân y′y 2 2 x a – 2 2 y b = –1 vôùi c2 = a2 + b2 vôùi c2 = a2 + b2 Tieâu ñieåm Tieâu cöï Truïc thöïc, ñoä daøi Truïc aûo, ñoä daøi Ñænh Tieäm caän Taâm sai Baùn kính M(xM, yM) ∈ (H) F1(–c, 0), F2(c, 0) 2c Ox, 2a Oy, 2b A1(–a, 0), A2(a, 0) y = ± b a x e = c a 1 1 2 2 M M r FM ex a r F M ex a = = +⎧ ⎨ = = −⎩ (xM a)≥ 1 2 M M r ex r ex a a = − −⎧ ⎨ = − +⎩ (xM ≤ – a) F1(0, –c), F2(0, c) 2c Oy, 2b Ox, 2a A1(0, –b), A2(0, b) y = ± a b x e = c b 1 1 2 2 M M r FM ey b r F M ey b = = +⎧ ⎨ = = −⎩ (yM ≥ b) 1 2 M M r ey r ey b b = − −⎧ ⎨ = − +⎩ (yM ≤ – b) 1
  • 31. Ñöôøng chuaån Phöông trình tieáp tuyeán taïi tieáp ñieåm M0(x0, y0) ∈ (H) x = ± a e 0 2 x x a – 0 2 y y b = 1 y = ± b e 0 2 x x a – 0 2 y y b = –1 Ngoaøi ra ta cuõng caàn löu yù: . Ñieàu kieän ñeå: (D) : Ax + By + C = 0 tieáp xuùc vôùi (H) : 2 2 x a – 2 2 y b = 1 laø a2 A2 – b2 B2 = C2 > 0 (D) : Ax + By + C = 0 tieáp xuùc vôùi (H) : 2 2 x a – 2 2 y b = –1 laø a2 A2 – b2 B2 = –C2 < 0 Ví duï : Cho hypebol (H) : 4x2 – y2 = 4 1) Xaùc ñònh tieâu ñieåm, ñænh, taâm sai, caùc ñöôøng tieäm caän vaø ñöôøng chuaån cuûa (H) 2) Vieát phöông trình tieáp tuyeán vôùi (H) taïi ñieåm M(1, 0) 3) Vieát phöông trình tieáp tuyeán vôùi (H) phaùt xuaát töø ñieåm N(1, 4) tìm toïa ñoä tieáp ñieåm. Giaûi 1) Caùc phaàn töû cuûa hypebol (H) (H) : 4x2 – y2 = 4 x2 –⇔ 2 4 y = 1 coù daïng 2 2 x a – 2 2 y b = 1 vôùi a2 = 1 a = 1, b2 = 4 ⇒ b = 2 vaø c2 = a2 + b2 = 5⇒ Vaäy hypebol (H) coù 2 tieâu ñieåm F1( 5− , 0), F2( 5 , 0) ; hai ñænh A1(–1, 0), A2(1, 0) ; taâm sai e = c a = 5 ; hai ñöôøng tieäm caän phöông trình y = ± 2x vaø hai ñöôøng chuaån phöông trình x = ± a e = ± 1 5 2
  • 32. 2) Phöông trình tieáp tuyeán vôùi (H) taïi tieáp ñieåm M(1, 0) Ta coù M(1, 0) ∈ (H) :4x2 – y2 = 4 ⇒ Phöông trình tieáp tuyeán vôùi (H) taïi tieáp ñieåm M(1, 0) laø 4xMx – yMy = 4 ⇔ 4x – 0y = 4 x = 1⇔ 3) Phöông trình tieáp tuyeán vôùi (H) phaùt xuaát töø N(1, 4). Hai tieáp tuyeán cuøng phöông vôùi 0y laø x = a = 1. Vaäy x=1 laø moät tieáp tuyeán qua N(1, 4).± ± Tieáp tuyeán ( qua N(1, 4) khoâng cuøng phöông vôùi 0y coù daïng:)Δ : y – 4 = k(x – 1)( )Δ ⇔ kx – y + 4 – k = 0 ( )Δ tieáp xuùc vôùi hypebol (H) : 2 1 x – 2 4 y = 1 ⇔ k2 . 12 – 4(–1)2 = (4 – k)2 ⇔ k2 - 4 = 16 – 8k + k2 ⇔ k = 20 5 8 2 = .Vaäy :( )Δ 5 2 x – y – 4 – 5 2 = 0 ⇔ 5x – 2y – 13 = 0 Toùm laïi coù hai tieáp tuyeán qua ñieåm N(1, 4) laø x = 1, vaø 5x – 2y – 13 = 0. * * * 3
  • 33. CHUYEÂN ÑEÀ 7 PARABOL Caùc baøi toaùn veà parabol thöôøng qui veà vieäc xaùc ñònh caùc yeáu toá cuûa parabol (tieâu ñieåm, ñöôøng chuaån), laäp phöông trình cuûa parabol vaø caùc vaán ñeà veà tieáp tuyeán cuûa parabol. Do ñoù ta caàn naém vöõng caùc kieán thöùc cô baûn sau ñaây : Parabol (P) = { M∈ (Oxy) / MF = ( )M d Δ } F laø tieâu ñieåm vaø ( laø ñöôøng chuaån.)Δ Caùc daïng phöông trình chính taéc : (P) : y2 = 2px ( )Δ : x = 2 p − F 0 2 p , ⎛ ⎞ ⎜ ⎟ ⎝ ⎠ M ∈ (P) ⇒ xM 0≥ vaø r = MF = xM + 2 p (d) : Ax + By + C = 0 tieáp xuùc vôùi (P) ⇔ pB2 = 2AC Tieáp tuyeán vôùi (P) taïi tieáp ñieåm (P) : y2 = –2px y x (P) F y ( )Δ : x = 2 p F 0 2 p , ⎛ ⎞ −⎜ ⎟ ⎝ ⎠ M ∈ (P) xM 0⇒ ≤ vaø r = MF = –xM + 2 p (d) : Ax + By + C = 0 tieáp xuùc vôùi (P) ⇔ pB2 = –2AC Tieáp tuyeán vôùi (P) taïi tieáp ñieåm x (P) F(P 2 , 0)P 2 − O ( )Δ P 2 O ( )Δ 1
  • 34. M0(x0, y0) coù phöông trình y0y = p(x0 + x) (P) : x2 = 2py ( )Δ : y = 2 p − F 0 2 p , ⎛ ⎞ ⎜ ⎟ ⎝ ⎠ M ∈ (P) ⇒ yM 0≥ vaø r = MF = yM + 2 p (d) : Ax + By + C = 0 tieáp xuùc vôùi (P) ⇔ pA2 = 2BC Tieáp tuyeán vôùi (P) taïi tieáp ñieåm M0(x0, y0) coù phöông trình x0x = p(y0 + y) M0(x0, y0) coù phöông trình y0y = –p(x0 + x) (P) : x2 = –2py ( )Δ : y = 2 p F 0 2 p , ⎛ ⎞ −⎜ ⎟ ⎝ ⎠ M ∈ (P) yM 0⇒ ≤ vaø r = MF = –yM + 2 p (d) : Ax + By + C = 0 tieáp xuùc vôùi (P) ⇔ pA2 = –2BC Tieáp tuyeán vôùi (P) taïi tieáp ñieåm M0(x0, y0) coù phöông trình x0x = –p(y0 + y) Ví duï1 : Cho parabol (P) : y2 – 8x = 0 1) Xaùc ñònh tieâu ñieåm F vaø ñöôøng chuaån ( )Δ cuûa (P) 2) Vieát phöông trình tieáp tuyeán vôùi (P) taïi ñieåm M(2; –4) y x (P) FP 2 O ( )Δ y x (P) F− P 2 O ( ) P 2 Δ 2
  • 35. 3) Vieát phöông trình tieáp tuyeán vôùi (P) bieát noù song song vôùi ñöôøng thaúng (D) : 2x – y + 5 = 0. Suy ra toïa ñoä tieáp ñieåm. 4) Vieát phöông trình tieáp tuyeán vôùi (P) bieát noù xuaát phaùt töø ñieåm I(–3, 0), suy ra toïa ñoä tieáp ñieåm. Giaûi 1) Tieâu ñieåm vaø ñöôøng chuaån (P) : y2 – 8x = 0 y2 = 8x coù daïng y2 = 2px vôùi p = 4⇔ Tieâu ñieåm F(2, 0) vaø ñöôøng chuaån⇒ ( )Δ : x = –2. 2) Phöông trình tieáp tuyeán vôùi (P) taïi M(2; –4) Tieáp tuyeán vôùi (P) : y2 = 8x taïi tieáp ñieåm M(2, –4) coù phöông trình cho bôûi coâng thöùc phaân ñoâi toïa ñoä : –4(y) = 4(2 + x) ⇔ x + y + 2 = 0 3) Phöông trình tieáp tuyeán vôùi (P) vaø song song vôùi (D) Ñöôøng thaúng (d) // (D) vôùi (D) : 2x – y + 5 = 0 (d) : 2x – y + C = 0⇒ (d) tieáp xuùc vôùi (P) : y2 = 8x 4 = 2 . 2C = 4C⇔ ⇔ C = 1 Vaäy tieáp tuyeán vôùi (P) phaûi tìm coù phöông trình 2x – y + 1 = 0 Tieáp tuyeán (d) vôùi (P) : y2 = 8x taïi tieáp ñieåm M0(x0, y0) coøn coù phöông trình y0y = 4(x0 + x) ⇔ 4x – y0y + 4x0 = 0 maø (d) : 2x – y + 1 = 0, do ñoù : 4 2 = 0 1 y = 04 1 x ⇒ 0 0 1 2 2 x y ⎧ =⎪ ⎨ ⎪ =⎩ hay M0 1 2 2 , ⎛ ⎞ ⎜ ⎟ ⎝ ⎠ 4) Phöông trình tieáp tuyeán vôùi (P) xuaát phaùt töø I(–3, 0). Tieáp tuyeán vôùi (P) vaø cuøng phöông vôùi 0y laø x = 0. Vaäy pt tieáp tuyeán ( ) quad′ I(–3, 0) coù daïng: (d ) : y – 0 = k(x + 3)′ ⇔ kx – y + 3k = 0 3
  • 36. ( ) tieáp xuùc vôùi (P) : y2 = 8xd′ 4 = 2k(3k) = 6k2 k =⇔ ⇔ ± 2 6 = ± 6 3 Vaäy töø ñieåm I(–3, 0) coù 2 tieáp tuyeán vôùi parabol (P) laø: 6 3 x – y + 6 = 0 hay – 6 3 x – y – 6 = 0 6 3 ⇔ x – y + 6 = 0 hay 6 x +3 y +3 6 = 0 Tieáp tuyeán (d ) vôùi (P) taïi tieáp ñieåm M0(x0, y0) coù phöông trình′ 4x – y0y + 4x0 = 0 Do ñoù vôùi (d ) :′ 6 3 x – y + 6 = 0 ⇒ 4 6 3 = 0 1 y = 04 6 x ⇒ 0 0 3 12 2 6 6 x y =⎧ ⎪ ⎨ = =⎪ ⎩ Vôùi ( ) :d′ 6 x + 3y + 3 6 = 0 ⇒ 4 6 = 0 3 y− = 04 3 6 x ⇒ 0 0 3 12 2 6 6 x y =⎧ ⎪ ⎨ = − = −⎪ ⎩ Vaäy 2 tieáp ñieåm phaûi tìm laø (3; 2 6 ) vaø (3; –2 6 ). Ví du2( ÑEÀ DÖÏ TRÖÕKHOÁI A –2003) : Trong maët phaúng vôùi heä toïa ñoä Ñeàcac vuoâng goùc Oxy, cho parabol (P) coù phöông trình y2 = x vaø ñieåm I (0; 2). Tìm toïa ñoä hai ñieåm M, N thuoäc (P) sao cho IN4IM = . Giaûi Goïi M(m2 ; m) ∈ (P), N(n2 ; n) ∈ (P) IM ⎯→ = (m2 ; m – 2) IN ⎯→ = (n2 ; n – 2) IN ⎯→ = (4n2 ; 4n – 8)⇒ 4 4
  • 37. Vì IM ⎯→ = 4 IN ⎯→ ⇔ 2 2 m 4n m 2 4n 8 ⎧ =⎪ ⎨ − = −⎪⎩ ⇔ ⇒ ⎢2 m 4n 6 n 4n 3 = −⎧⎪ ⎨ − + =⎪⎩ 0 =⎣ 1 2 n 1 n 3 =⎡ 1 2 m 2 m 6 ⇒ = − ⇒ = ⇒ M1(4; −2), N1(1; 1), M2(36; 6), N2(9; 3) Ví du 3 ( ÑEÀ DÖÏ TRÖÕKHOÁI A –2003) :Trong maët phaúng vôùi heä toïa ñoä Ñeàcac vuoâng goùc Oxy cho elip (E): 1 1 y 4 x 22 =+ . M(−2; 3); N(5; n). Vieát phöông trình caùc ñöôøng thaúng d1, d2 qua M vaø tieáp xuùc vôùi (E). Tìm n ñeå trong soá caùc tieáp tuyeán cuûa (E) ñi qua N coù moät tieáp tuyeán song song vôùi d1 hoaëc d2. Giaûi 1) Vieát phöông trình caùc ñöôøng thaúng qua M tieáp xuùc vôùi E. x = 2 laø 2 tieáp tuyeán thaúng ñöùng cuûa (E)± Vaäy d1 : x = −2 laø 1 tieáp tuyeán cuûa (E) qua M. Phöông trình tieáp tuyeán d qua M(−2; 3) khaùc döôøng thaúng x = −2 coù daïng : y – 3 = k(x + 2) O 3 x y −2 M⇔ kx – y + 3 + 2k d tieáp xuùc vôùi (E) ⇔ 4k2 + 1 = (3 + 2k)2 ⇔ 4k2 + 1 = 9 + 4k2 + 12k 8 2 12 3 − = −⇔ k = d2 : 2x + 3y – 5 = 0 2) deã thaáy tieáp tuyeán d cuûa (E) qua N(5; n) khoâng song song vôùi : x = −2. Do ñoù d song song vôùi d2 : 2x + 3y – 5 = 0 vaø qua N(5; n) coù heä soá goùc : k = − 2 3 = − − + 2 y (x 5) 3 n. Vaäy d : hay d : 2 − − + n = 0 ⇔ −2x – 3y + 10 + 3n = 0 10 x y 3 3 + d tieáp xuùc vôùi E ⇔ 4(−2)2 + 1.(−3)2 = (10 + 3n)2 − 5 3 ⇔ 3n2 + 20n + 25 = 0⇔ n = – 5 hay n= − 5 3 : loaïi vì khi ñoù d truøng vôùi d1.n = Vaäy N(5; −5). * * * 5
  • 38. CHUYEÂN ÑEÀ 8 VECTÔ TRONG KHOÂNG GIAN Caùc ñònh nghóa vaø pheùp toaùn cuûa vectô trong khoâng gian cuõng gioáng nhö trong maët phaúng, ta caàn löu yù ñeán caùc vaán ñeà cô baûn thoâng duïng nhö : . Qui taéc 3 ñieåm : A, B, C thì∀ AB + BC = AC . Coäng 2 vectô cuøng goác laø moät vectô cuøng goác vaø laø ñöôøng cheùo hình bình haønh coù 2 caïnh laø 2 vectô ñaõ cho. . I laø trung ñieåm ñoaïn thaúng AB, vôùi ñieåm M baát kyø naøo ta luoân coù: MI = 2 MA MB+ . G laø troïng taâm cuûa Δ ABC ⇔ GA + GB + GC = 0 . Ngoaøi ra ta coøn coù : . Ba vectô khaùc goïi laø ñoàng phaúng neáu giaù cuûa chuùng cuøng song song hoaëc naèm trong moät maët phaúng . 0 . Baát kyø vectô a 0 naøo ñoàng phaúng vôùi hai vectô khoâng cuøng phöông , trong khoâng gian, ñeàu coù theå phaân tích theo ≠ 1e 2e 1e , 2e coù nghóa: a = α 1e + β 2e (α ,β ∈ R) vaø söï phaân tích treân laø duy nhaát . . Baát kyø vectô a naøo trong khoâng gian cuõng coù theå phaân tích ñöôïc theo 3 vectô khoâng ñoàng phaúng , , coù nghóa : ≠ 0 1e 2e 3e a = + βα 1e 2e + γ 3e (α ,β , γ ∈ R) . G ñöôïc goïi laø troïng taâm cuûa töù dieän ABCD + + GC +⇔ GA GB GD = 0 Ghi chuù : 1) Neáu moät trong 3 vectô ,a b , c laø 0 thì chuùng ñoàng phaúng. 2) a, b , c ñoàng phaúng ⇔ , . 0a b c⎡ ⎤ =⎣ ⎦ 1
  • 39. 3) OA , OB, ñoàng phaúngOC ⇔ O, A, B, C cuøng naèm treân moät maët phaúng. Ví duï 1: Cho moät hình laêng truï ABC A′ B′ C′ . Goïi I, I′ laàn löôït laø troïng taâm cuûa Δ ABC vaø Δ A′ B′ C′ , O laø trung ñieåm cuûa II′ . a) Chöùng minh raèng + + OBOA OA′ + OB′ + OC + OC′ = 0 b) Goïi G laø troïng taâm cuûa hình töù dieän ABCC′ vaø M laø trung ñieåm cuûa A′ B′ . Chöùng minh raèng O, M, G thaúng haøng. c) Tính tæ soá OM OG Giaûi a) + OA + +OA ′ OB OB′ + OC + OC′ = 0 I laø troïng taâm cuûa ABC ⇒Δ IA + IB + IC = 0 ( + ) + (IO + OB) + (⇒ IO OA IO + OC) = 0 OA + + OC = 3OI⇒ OB Töông töï, laø troïng taâm cuûaI′ Δ A′ B′ C′ OA + + OC = 3OI⇒ ′ OB′ ′ ′ Vaäy OA + OA′ + OB + OB′ + OC + OC′ = = 3OI + 3OI′ = 3(OI + OI′ ) = 0 (vì 0 laø trung ñieåm II′ ) b) O, M, G thaúng haøng G laø troïng taâm cuûa töù dieän ABCC′ ⇒ GA + + GC +GB GC′ = 0 ⇒ ( + OA ) + (GO + ) + (GO OB GO + OC) + (GO + OC′ ) = 0 ⇒ OA + + OC + OCOB ′ = 4OG M laø trung ñieåm cuûa A B′ ′ ⇒ OA + = 2OM′ OB′ ⇒ OA + + OC + OCOB ′ + OA′ + OB′ = 4OG + 2OM 2
  • 40. ⇒ 0 = 4 + 2OMOG ⇒ OM = –2OG ⇒ OM cuøng phöông vôùi OG ⇒ OM, OG cuøng giaù (vì cuøng goác O) ⇒ O, M, G thaúng haøng. c) Tæ soá OM OG OM = –2OG ⇒ OM OG = –2 Ví duï 2: Cho hình hoäp ABCD. A′ B′ C′ D′ vôùi AA′ = a, AB = b , / AC = . Haõy bieåu thò caùc vectô c AD , A C′ , , theo caùc vectô aB D′ BD′ , b , c. Giaûi Ta coù vôùi hình hoäp ABCD. A′ B′ C′ D′ thì : AD = AC′ + / C D′ + D D′ = c – b – a A C′ = A A′ + / AC + / C C A C′ = –2a + c B D′ = B B′ + BA + AD = – a – b + c – –b a = – 2a – 2 b + c BD′ = BA + AD + DD′ = – b + (c – – a) +b a = – 2 b + c * * * D′A B′ ′ c B C DA a C′ b 3
  • 41. CHUYEÂN ÑEÀ 9 PHÖÔNG PHAÙP TOÏA ÑOÄ TRONG KHOÂNG GIAN Caùc baøi toaùn veà toïa ñoä trong khoâng gian thöôøng coù caùc yeâu caàu xaùc ñònh toïa ñoä cuûa ñieåm, vectô, ñoä daøi ñoaïn thaúng, tính goùc 2 vectô, caùc vaán ñeà veà maët phaúng vaø ñöôøng thaúng trong khoâng gian (phöông trình, vò trí töông ñoái, song song, vuoâng goùc, soá ño goùc, khoaûng caùch,… ). Tuøy theo töøng tröôøng hôïp ta caàn löu yù vaän duïng caùc kieán thöùc cô baûn sau ñaây : I. Toaï ñoä ñieåm. Toaï ñoä vectô Trong khoâng gian toïa ñoä vuoâng goùc Oxyz coù 3 vectô ñôn vò treân ba truïc Ox, Oy, Oz laàn löôït laø , , .1e 2e 3e * Cho M(x, y, z) thì OM = x. + y.1e 2e + z. 3e . * Cho a = (a1, a2, a3) thì a = a1. 1e + a2. 2e + a3. 3e . II. Caùc pheùp toaùn treân toïa ñoä ñieåm, vectô 1. Caùc pheùp toaùn treân toïa ñoä ñieåm Cho hai ñieåm A(x1, y1, z1) vaø B(x2, y2, z2). Ta coù nhoùm coâng thöùc tính toïa ñoä vectô AB, khoaûng caùch giöõa hai ñieåm A, B vaø toïa ñoä ñieåm M laø chia ñoaïn AB theo tæ soá k ≠ 1 * AB = (x2 – x1, y2 – y1, z2 – z1) * AB = ( ) ( ) ( ) 2 2 2 1 2 1 2 1x x y y z z− + − + − 2 * ( x = 1 1 x kx k − − 2 , y = 1 1 y ky k − − 2 , z = 1 2 1 z kz k − − ) 2. Caùc pheùp toaùn treân toïa ñoä vectô Cho hai vectô a = (a1, a2, a3), = (b1, b2, b3). Vôùib α vaø β laø 2 soá thöïc ta coù caùc coâng thöùc tính vaø coâng thöùc quan heä sau : a) Coâng thöùc tính toaùn . + β . = (α .a1 + .b1, .a2 +α a b β α β .b2, α .a +3 β .b )3 a . b = a1.b1 + a2.b2 + a3 .b3 )cos =(a,b 1 1 2 2 3 3 2 2 2 2 2 1 2 3 1 2 3 a .b a .b a .b a a a . b b b + + + + + + 2 b) Coâng thöùc quan heä 1
  • 42. =a b ⇔ 1 1 2 2 3 3 a b a b a b =⎧ ⎪ =⎨ ⎪ =⎩ cuøng phöônga b ⇔ ( 1 1 a b = 2 2 a b = 3 3 a b ) (b1, b2, b3 ≠ 0) ⊥ a1.b1 + a2.b2 + a .b = 0a b ⇔ 3 3 Chuù yù : Goùc hai ñöôøng thaúng cheùo nhau trong khoâng gian laø goùc nhoïn taïo bôûi hai vectô chæ phöông cuûa 2 ñöôøng thaúng ñoù. MAËT PHAÚNG I. Phöông trình maët phaúng 1.* Phöông trình tham soá cuûa maët phaúng α qua M(x0, y0, z0) coù caëp vectô chæ phöông a = (a1, a2, a3 ), = (b1, b2, b ) vieát laø :b 3 t1, t2 0 1 1 2 1 0 1 2 2 0 1 3 2 3 x x t a t b y y t a t b z z t a t b = + +⎧ ⎪ = + +⎨ ⎪ = + +⎩ 2 ∈ R 2.* Phöông trình toång quaùt cuûa maët phaúng α laø : Ax + By + Cz + D = 0vôùi A2 + B2 + C2 > 0 Maët phaúng α coù : phaùp vectô : n = (A, B, C) 3.* Phöông trình maët phaúng qua M(x0, y0, z0) vaø vuoâng goùc vôùi vectô n = (A, B, C) vieát laø : (x – x0)A + (y – y0)B + (z – z0)C = 0 4.* Phöông trình maët phaúng qua M(x0, y0, z0) vaø nhaän 2 vectô chæ phöông a = (a1, a2, a ), = (b1, b2, b3 ) vieát laø3 b ( ) ( ) ( )2 3 3 1 1 2 0 0 2 3 3 1 1 2 0 a a a a a a x x y y z z b b b b b b − + − + − =0 . 5.* Phöông trình maët phaúng caét ba truïc toïa ñoä taïi A(a, 0, 0); B(0, b, 0); C(0, 0, c) vôùi a.b.c ≠ 0 vieát laø : x a + y b + z c = 1 II. Toaùn treân maët phaúng 1. Khoaûng caùch töø moät ñieåm ñeán moät maët phaúng Khoaûng caùch töø M(x0, y0, z0) ñeán 2
  • 43. α : Ax + By + Cz + D = 0 laø : MH = 0 0 0 2 2 2 Ax By Cz D A B C + + + + + 2. Vò trí töông ñoái giöõa hai maët phaúng Cho hai maët phaúng α , β coù 2 phaùp vectô laàn löôït laø n = (A, B, C), = (A1, B1, C1)1n Vò trí giöõa hai maët phaúng , laø vò trí giöõa 2 phaùp vectôα β n , 1n : // β //α ⇔ n 1n α ⊥ β ⇔ n ⊥ 1n caét β khaùc phöôngα ⇔ n 1n ÑÖÔØNG THAÚNG I. Phöông trình ñöôøng thaúng 1.* Phöông trình tham soá cuûa ñöôøng thaúng Δ qua M(x0, y0, z0) coù vectô chæ phöông a = (a1, a2, a ) vieát laø3 0 1 0 0 3 2 x x ta y y ta z z ta = +⎧ ⎪ = +⎨ ⎪ = +⎩ ,t ∈ R (Heä I). Neáu a1.a2.a3 ≠ 0 ta coù phöông trình chính taéc laø: x x a y y a z z a − = − = −0 1 0 2 0 3 2.* Phöông trình toång quaùt cuûa ñöôøng thaúng Δ xaùc ñònh bôûi giao tuyeán 2 maët phaúng α vaø β vieát laø : 1 1 1 1 0 0 Ax By Cz D ( ) A x B y C z D ( ) + + + = α⎧ ⎨ + + + =⎩ β (II) Ghi chuù: Cho phöông trình ñöôøng thaúng Δ xaùc ñònh bôûi heä (II). Ñeå vieát thaønh phöông trình tham soá cuûa ñöôøng thaúng ta coù theå ñaët z = t vaø tính x, y theo t töø heä (II) vaø nhôø heä (I) ta coù ñöôïc vectô chæ phöông vaø ñieåm cuûa (hoaëc x = t,Δ hoaëc y = t, neân choïn löïa aån phuï t ñeå pheùp tính hai bieán coøn laïi theo t ñöôïc ñôn giaûn). 3.*Phöông trình maët phaúng chöùa ñöôøng thaúng (d) : A x B y C z D A x B y C z D 1 1 1 1 2 2 2 2 0 0 + + + = + + + = ⎧ ⎨ ⎩ 3
  • 44. Coù daïng : m(A1x + B1y + C1z + D1) + n(A2x + B2y + C2z + D2) = 0 (*) vôùi m, n khoâng ñoàng thôøi baèng 0. Phöông trình (*) goïi laø phöông trình cuûa chuøm maët phaúng xaùc ñònh bôûi ñöôøng thaúng (d). Chuù yù :Neáu m= 0 thì n khaùc 0, chia hai veá cuûa (*) cho n ta coù (*) thaønh A2x + B2y + C2z + D2 = 0 Neáu m khaùc 0 chia hai veá cuûa (*) cho m ta coù: A1x + B1y + C1z + D1 + h (A2x + B2y + C2z + D2) = 0 vôùi n h m = . Vaäy chuøm maët phaúng chöùa ñöôøng thaúng (d) coù daïng: A1x + B1y + C1z + D1 + h (A2x + B2y + C2z + D2) = 0. hay A2x + B2y + C2z + D2 = 0. Vaán ñeà 1 TÌM PHÖÔNG TRÌNH MAËT PHAÚNG Phöông phaùp : Thoâng thöôøng ta coù 3 caùch sau : - Caùch 1 : Tìm moät ñieåm vaø moät caëp vectô chæ phöông cuûa maët phaúng. - Caùch 2 : Tìm moät ñieåm vaø moät phaùp vectô cuûa maët phaúng. - Caùch 3 : Duøng phöông trình chuøm maët phaúng. Vaán ñeà 2 : TÌM PHÖÔNG TRÌNH ÑÖÔØNG THAÚNG Phöông phaùp : Thoâng thöôøng ta coù 2 caùch sau : - Caùch 1 : Tìm moät ñieåm vaø moät vectô chæ phöông cuûa ñöôøng thaúng. - Caùch 2 : Tìm phöông trình toång quaùt cuûa 2 maët phaúng phaân bieät cuøng chöùa ñöôøng thaúng caàn tìm. - Ghi chuù : Trong 2 caùch, thöïc chaát cuûa vieäc tìm phöông trình ñöôøng thaúng laø tìm phöông trình 2 maët phaúng cuøng chöùa ñöôøng thaúng aáy. Caùi khoù laø phaûi xaùc ñònh ñöôïc 2 maët phaúng phaân bieät naøo cuøng chöùa ñöôøng thaúng caàn tìm. Thoâng thöôøng ta hay gaëp 3 giaû thuyeát sau : + Ñöôøng thaúng (Δ) ñi qua ñieåm A vaø caét ñöôøng thaúng d : Khi ñoù ñöôøng thaúng (Δ) naèm trong maët phaúng ñi qua A vaø chöùa d. + Ñöôøng thaúng (Δ) ñi qua ñieåm A vaø vuoâng goùc vôùi ñöôøng thaúng d : Khi ñoù ñöôøng thaúng (Δ) naèm trong maët phaúng ñi qua A vaø vuoâng goùc vôùi d. + Ñöôøng thaúng (Δ) song song vôùi d1 vaø caét d2 : Khi ñoù ñöôøng thaúng (Δ) naèm trong maët phaúng chöùa d2 vaø song song vôùi d1. Chaúng haïn : 1. Laäp phöông trình ñöôøng thaúng (Δ) ñi qua ñieåm A, vuoâng goùc vôùi ñöôøng thaúng a vaø caét ñöôøng thaúng aáy. Caùch giaûi : - (Δ) ñi qua A vaø vuoâng goùc vôùi d neân (Δ) naèm trong maët phaúng α ñi qua A vaø vuoâng goùc vôùi d. - (Δ) ñi qua A vaø caét d neân (Δ) naèm trong maët phaúng β ñi qua A vaø chöùa d. Khi ñoù (Δ) chính laø giao tuyeán cuûa α vaø β. 2. Laäp phöông trình ñöôøng thaúng (Δ) ñi qua ñieåm A vaø caét caû hai ñöôøng thaúng d1 vaø d2. Caùch giaûi : - (Δ) ñi qua A vaø caét d1 neân (Δ) naèm trong maët phaúng α ñi qua A vaø chöùa d1. 4
  • 45. - (Δ) ñi qua A vaø caét d2 neân (Δ) naèm trong maët phaúng β ñi qua A vaø chöùa d2. Khi ñoù (Δ) chính laø giao tuyeán cuûa α vaø β. 3. Laäp phöông trình ñöôøng thaúng (Δ) ñi qua giao ñieåm A cuûa ñöôøng thaúng d vaø maët phaúng α, vuoâng goùc vôùi d vaø naèm trong α. Caùch giaûi : - Töø giaû thuyeát ta ñaõ coù (Δ) ⊂ α. - (Δ) qua A vaø vuoâng goùc vôùi d neân (Δ) naèm trong maët phaúng β ñi qua A vaø vuoâng goùc vôùi d. Khi ñoù (Δ) chính laø giao tuyeán cuûa α vaø β. 4. Laäp phöông trình ñöôøng thaúng (Δ) song song vôùi ñöôøng thaúng (D) vaø caét 2 ñöôøng thaúng d1 vaø d2. Caùch giaûi : - (Δ) song song vôùi (D) vaø caét d1 neân (Δ) naèm trong maët phaúng α chöùa d1 vaø song song vôùi (D). - (Δ) song song vôùi (D) vaø caét d2 neân (Δ) naèm trong maët phaúng β chöùa d2 vaø song song vôùi (D). Khi ñoù (Δ) chính laø giao tuyeán cuûa α vaø β. Vaán ñeà 3 HÌNH CHIEÁU Baøi toaùn 1 : Tìm hình chieáu vuoâng goùc H cuûa ñieåm A treân ñöôøng thaúng (d) Phöông phaùp : (d) A H - Caùch 1 : (d) cho bôûi phöông trình tham soá : + H ∈ (d) suy ra daïng toïa ñoä cuûa ñieåm H phuï thuoäc vaøo tham soá t. + Tìm tham soá t nhôø ñieàu kieän ⊥aAH → d → - Caùch 2 : (d) cho bôûi phöông trình chính taéc, goïi H(x, y, z) + AH → ⊥a (*)d → + H ∈ (d) : Bieán ñoåi tæ leä thöùc naøy ñeå duøng ñieàu kieän (*), töø ñoù tìm ñöôïc x, y, z. - Caùch 3 : (d) cho bôûi phöông trình toång quaùt : + Tìm phöông trình maët phaúng α ñi qua A vaø vuoâng goùc vôùi ñöôøng thaúng (d). + Giao ñieåm cuûa (d) vaø (α) chính laø hình chieáu H cuûa A treân (d). Baøi toaùn 2 : Tìm hình chieáu vuoâng goùc H cuûa ñieåm A treân maët phaúng (α) - Caùch 1 : Goïi H(x, y, z) + H ∈ α (*) + AH → cuøng phöông vôùi : Bieán ñoåi tæ leä thöùc naøy ñeå duøng ñieàu kieän (*), töø ñoù tìm ñöôïc x, y, z.nα → - Caùch 2 : + Tìm phöông trình ñöôøng thaúng (d) ñi qua A vaø vuoâng goùc vôùi maët phaúng (α). + Giao ñieåm cuûa (d) vaø (α) chính laø hình chieáu H cuûa A treân maët phaúng (α). 5
  • 46. Baøi toaùn 3 : Tìm hình chieáu vuoâng goùc (Δ) cuûa ñöôøng thaúng (d) xuoáng maët phaúng α. - Tìm phöông trình maët phaúng β chöùa ñöôøng thaúng d vaø vuoâng goùc vôùi maët phaúng α. - Hình chieáu (Δ) cuûa d xuoáng maët phaúng α chính laø giao tuyeán cuûa α vaø β. Baøi toaùn 4 : Tìm hình chieáu H cuûa A theo phöông ñöôøng thaúng (d) leân maët phaúng (α). Phöông phaùp : - Tìm phöông trình ñöôøng thaúng (Δ) ñi qua A vaø song song vôùi (d). - Hình chieáu H chính laø giao ñieåm cuûa (Δ) vaø (α). Baøi toaùn 5 : Tìm hình chieáu (Δ) cuûa ñöôøng thaúng (d) theo phöông cuûa ñöôøng thaúng (D) leân maët phaúng (α). (Δ) A H (d) Phöông phaùp : (D) d (Δ) - Tìm phöông trình maët phaúng (β) chöùa (d) vaø song song vôùi (D) - Hình chieáu (Δ) chính laø giao tuyeán cuûa (α) vaø (β) Vaán ñeà4 ÑOÁI XÖÙNG Baøi toaùn 1 : Tìm ñieåm A’ ñoái xöùng vôùi A qua ñöôøng thaúng d. Phöông phaùp : - Tìm hình chieáu H cuûa A treân d. - H laø trung ñieåm AA’. Baøi toaùn 2 : Tìm ñieåm A’ ñoái xöùng vôùi A qua maët phaúng α. Phöông phaùp : - Tìm hình chieáu H cuûa A treân α. - H laø trung ñieåm AA’. Baøi toaùn 3 : Tìm phöông trình ñöôøng thaúng d ñoái xöùng vôùi ñöôøng thaúng (D) qua ñöôøng thaúng (Δ) Phöông phaùp : - Tröôøng hôïp 1 : (Δ) vaø (D) caét nhau : + Tìm giao ñieåm M cuûa (D) vaø (Δ). (D) d (Δ)M A A’ + Tìm moät ñieåm A treân (D) khaùc vôùi ñieåm M. + Tìm ñieåm A’ ñoái xöùng vôùi A qua (Δ) + d chính laø ñöôøng thaúng ñi qua 2 ñieåm A’ vaø M. 6
  • 47. - Tröôøng hôïp 2 : (Δ) vaø (D) song song : + Tìm moät ñieåm A treân (D) + Tìm ñieåm A’ ñoái xöùng vôùi A qua (Δ) + d chính laø ñöôøng thaúng qua A’ vaø song song vôùi (Δ) - Tröôøng hôïp 3 : (Δ) vaø (D) cheùo nhau : + Tìm 2 ñieåm phaân bieät A, B treân (D) + Tìm ñieåm A’, B’ laàn löôït laø ñieåm ñoái xöùng cuûa A, B qua (Δ) + d chính laø ñöôøng thaúng ñi qua 2 ñieåm A’, B’. Baøi toaùn 4 : Tìm phöông trình ñöôøng thaúng d ñoái xöùng vôùi ñöôøng thaúng (D) qua maët phaúng α. Phöông phaùp : - Tröôøng hôïp 1 : (D) caét α + Tìm giao ñieåm M cuûa (D) vaø (α) + Tìm moät ñieåm A treân (D) + Tìm ñieåm A’ ñoái xöùng vôùi A qua maët phaúng α . + d chính laø ñöôøng thaúng ñi qua hai ñieåm A’ vaø M . - Tröôøng hôïp 2 : (D) song song vôùi α. A A’d (D) - Tìm moät ñieåm A treân (D) - Tìm ñieåm A’ ñoái xöùng vôùi A qua maët phaúng α. - d chính laø ñöôøng thaúng qua A’ vaø song song vôùi (D) Vaán ñeà 5 KHOAÛNG CAÙCH Baøi toaùn 1 : Tính khoaûng caùch töø ñieåm M(x0, y0, z0) ñeán maët phaúng α : Ax + By + Cz + D = 0 Phöông phaùp : d M Ax By Cz D A B C ( , )α = + + + + + 0 0 0 2 2 2 Baøi toaùn 2 : Tính khoaûng caùch töø ñieåm M ñeán ñöôøng thaúng (Δ) Phöông phaùp : - Tìm hình chieáu H cuûa M treân (Δ) - Khoaûng caùch töø M ñeán (Δ) chính laø ñoä daøi ñoaïn MH. Baøi toaùn 3 : Tính khoaûng caùch giöõa 2 ñöôøng thaúng song song d1 vaø d2. Phöông phaùp : 7
  • 48. - Tìm moät ñieåm A treân d1. - Khoaûng caùch giöõa d1 vaø d2 chính laø khoaûng caùch töø ñieåm A ñeán d2. Baøi toaùn 4 : Tính khoaûng caùch giöõa 2 maët phaúng song song α : Ax + By + Cz + D1 = 0 Vaø β : Ax + By + Cz + D2 = 0 Phöông phaùp : Khoaûng caùch giöõa α vaø β ñöôïc cho bôûi coâng thöùc : d D D A B C ( , )α β = − + + 1 2 2 2 2 Baøi toaùn 5 : Tính khoaûng caùch giöõa 2 ñöôøng thaúng cheùo nhau d1 vaø d2 Phöông phaùp : - Caùch 1 : + Tìm phöông trình maët phaúng α chöùa d1 vaø song song vôùi d2. + Tìm moät ñieåm A treân d2. + Khi ñoù d(d1, d2) = d(A, α) - Caùch 2 : + Tìm phöông trình maët phaúng α chöùa d1 vaø song song vôùi d2. + Tìm phöông trình maët phaúng β chöùa d2 vaø song song vôùi d1. + Khi ñoù d(d1, d2) = d(α, β) Ghi chuù : Maët phaúng α vaø β chính laø 2 maët phaúng song song vôùi nhau vaø laàn löôït chöùa d1 vaø d2. - Caùch 3 : + Vieát döôùi daïng phöông trình tham soá theo t. + Vieát d2 döôùi daïng phöông trình tham soá theo t2. + Xem A ∈ d1 ⇒ daïng toïa ñoä A theo t1. + Xem B ∈ d2 ⇒ daïng toïa ñoä B theo t2. + Tìm vectô chæ phöông laàn löôït cuûa d1 vaø d2.a a1 2 → → , + AB laø ñoaïn vuoâng goùc chung d1, d2. ⇔ tìm ñöôïc t1 vaø t2 AB a AB a → → → → ⊥ ⊥ ⎧ ⎨ ⎪ ⎩ ⎪ 1 2 + Khi ñoù d(d1, d2) = AB Vaán ñeà 6 GOÙC Cho 2 ñöôøng thaúng d vaø d’ coù phöông trình : d : x x a y y b z z c − = − = −0 0 0 d’ : x x a y y b z z c − = − = −0 0 ' ' 0 ' Cho 2 maët phaúng α vaø β coù phöông trình : α : Ax + By + Cz + D = 0 β : A’x + B’y + C’z + D’ = 0 1. Goùc giöõa hai ñöôøng thaúng d vaø d’ : cos ' ' ' ' ' ' ϕ = + + + + + + aa bb cc a b c a b c2 2 2 2 2 2 2. Goùc giöõa hai maët phaúng α vaø β : 8
  • 49. cos ' ' ' ' ϕ = + + + + + + AA BB CC' A B C A B C'2 2 2 2 2 2 3. Goùc giöõa ñöôøng thaúng d vaø maët phaúng α : sinϕ = + + + + + + Aa Bb Cc A B C a b c2 2 2 2 2 2 Chuù yù : - d ⊥ d’ ⇔ aa’ + bb’ + cc’ = 0 - α ⊥ β ⇔ AA’ + BB’ + CC’ = 0 - d song song (hoaëc naèm treân) maët phaúng α ⇔ aA + bB + cC = 0 Vaán ñeà 7 VÒ TRÍ TÖÔNG ÑOÁI CUÛA HAI MAËT PHAÚNG Cho hai maët phaúng α vaø β coù phöông trình : α : A1x + B1y + C1z + D1 = 0 β : A2x + B2y + C2z + D2 = 0 Goïi n A laàn löôït laø phaùp vectô cuûa 2 maët phaúng treân vaø M laø moät ñieåm treân maët phaúng α. B C n A B C1 1 1 1 2 2 2 → → = =( , , ), ( , , 2 ) - α caét β ⇔ vaø khoâng cuøng phöông.n1 → n2 → - α song song β ⇔ n vaø n cuøng phöông M 1 2 → → ∉ ⎧ ⎨ ⎪ ⎩⎪ β - α truøng β ⇔ n vaø n cuøng phöông M 1 2 → → ∈ ⎧ ⎨ ⎪ ⎩⎪ β Neáu A2, B2, C2, D2 ≠ 0 thì ta coù caùch khaùc : - α caét β ⇔ A1 : B1 : C1 ≠ A2 : B2 : C2 - α song song β ⇔ A A B B C C D D 1 2 1 2 1 2 1 2 = = ≠ - α truøng β ⇔ A A B B C C D D 1 2 1 2 1 2 1 2 = = = Vaán ñeà 8 VÒ TRÍ TÖÔNG ÑOÁI CUÛA 2 ÑÖÔØNG THAÚNG - Caùch 1 : Xeùt heä phöông trình toïa ñoä giao ñieåm cuûa hai ñöôøng thaúng d1 vaø d2. + Heä coù moät nghieäm duy nhaát : d1 caét d2. + Heä coù voâ soá nghieäm : d1 vaø d2 truøng nhau. + Heä voâ nghieäm : cuøng phöông : d1 // d2.a vaøad d1 → → 2 2 khoâng cuøng phöông : d1 vaø d2 cheùo nhau.a vaøad d1 → → - Caùch 2 : + Tìm vectô chæ phöông a cuûa d1 vaø d2.ad d1 2 → → , + Tìm ñieåm A ∈ d1 vaø B ∈ d2. a) a v cuøng phöôngaøad d1 → → 2 A d d d A d d d ∈ ≡ ∉ 2 1 2 2 1 2 : : / / 9
  • 50. b) a v khoâng cuøng phöông ta coù:aøad d1 → → 2 0 0 i) neáu thì d1,d2 caét nhau.1 2 , .d da a AB⎡ ⎤ =⎣ ⎦ ii) neáu thì d1,d2 cheùo nhau.1 2 , .d da a AB⎡ ⎤ ≠⎣ ⎦ Vaán ñeà 9 VÒ TRÍ TÖÔNG ÑOÁI GIÖÕA ÑÖÔØNG THAÚNG VAØ MAËT PHAÚNG - Caùch 1 : Xeùt heä phöông trình toïa ñoä giao ñieåm cuûa ñöôøng thaúng d vaø maët phaúng α. + Heä voâ nghieäm : d // α. + Heä coù nghieäm duy nhaát : d caét α + Heä voâ soá nghieäm : d ⊂ α - Caùch 2 : Tìm vectô chæ phöông cuûa d, phaùp vectô cuûa α vaø tìm ñieåm A ∈ d.a → n → + a ≠ 0 ( khoâng vuoâng goùc ) : d caét α.n → → . a → n → + a = 0 ( )n → → . a n → → ⊥ A d A d ∉ ∈ ⊂ α α α α : / / : Ví duï 1: Laäp phöông trình maët phaúng chöùa ñöôøng thaúng (D) 2 0 3 2 3 x z x y z − =⎧ ⎨ − + − =⎩ 0 vaø vuoâng goùc vôùi maët phaúng (P) : x – 2y + z + 5 = 0 Giaûi Phöông trình tham soá cuûa (D) vieát 2 7 3 2 2 x t y t z t =⎧ ⎪ ⎪ = −⎨ ⎪ =⎪⎩ Maët phaúng (Q) chöùa (D) vaø vuoâng goùc (P) seõ ñi qua ñieåm M( 0, 3 2 − , 0 ∈ (D) vaø coù caëp vectô chæ phöông laø a) = ( 2, 7 2 , 1 (vectô chæ phöông cuûa (D) vaø = (1, –2, 1) (phaùp vectô cuûa (P)). ) n Do ñoù, moät phaùp veùctô cuûa ( Q) laø 1 2 1 1 2 1 1 2 ; ;7 7 1 21 2 2 2 n ⎛ − − ⎞ ⎜ ⎟= = ⎜ ⎟⎜ ⎟ ⎝ ⎠ = (– 11, 2, 15) 10
  • 51. Vaäy phöông trình (Q) vieát –11x + 2 ( y 3 2 + ) + 15z = 0 11x – 2y - 15 z – 3 = 0.⇔ Caùch khaùc: Pt maët phaúng (Q) chöùa (D) vaø vuoâng goùc (P) coù daïng: x-2z = 0 (loaïi) hay m(x-2z) +3x -2y+z -3= 0. Vaäy pt (Q) coù daïng: (m+3)x –2y +(1 –2m)z – 3 = 0. (Q) vuoâng goùc vôùi (P) neân ta coù: m + 3 + 4 + 1- 2 m= 0 ⇒ m = 8. Vaäy pt mp (Q) laø: 11x – 2y - 15 z – 3 = 0. Ví duï 2: Xaùc ñònh caùc tham soá m vaø n ñeå maët phaúng 5x + ny + 4z + m = 0 thuoäc chuøm maët phaúng coù phöông trình : (3x – 7y + z – 3) + β (x – 9y – 2z + 5) = 0α Giaûi Chuøm maët phaúng coù phöông trình (3x – 7y + z – 3) + β (x – 9y – 2z + 5) = 0α chöùa ñöôøng thaúng (D) coù phöông trình : 3 7 3 9 2 5 x y z x y z − + − =⎧ ⎨ − − + =⎩ 0 0 Ñeå maët phaúng (P) : 5x + ny + 4z + m = 0 thuoäc chuøm maët phaúng treân thì (P) chöùa (D) nghóa laø chöùa 2 ñieåm A 1 18 ,0, 7 7 ⎛ ⎞ ⎜ ⎟ ⎝ ⎠ , B 31 9 0 10 10 , , ⎛ ⎜ ⎝ ⎠ ⎞ ⎟ ∈ (D). Ñieàu kieän ñeå (P) chöùa A, B thì m, n thoûa heä phöông trình : 5 18 4 0 7 7 31 9 5 0 10 10 . m . .n m ⎧ + + =⎪⎪ ⎨ ⎪ + + = ⎪⎩ ⇒ 11 5 m n = −⎧ ⎨ = −⎩ Ví duï 3: ( ÑH KHOÁI A-2002) Trong khoâng gian vôùi heä toïa ñoä Ñeâcac vuoâng goùc Oxyz cho hai ñöôøng thaúng: Δ1 : vaø Δ2 : ⎩ ⎨ ⎧ =+−+ =−+− 04z2y2x 04zy2x ⎪ ⎩ ⎪ ⎨ ⎧ += += += t21z t2y t1x a) Vieát phöông trình maët phaúng (P) chöùa ñöôøng thaúng Δ1 vaø song song vôùi ñöôøng thaúng Δ2. 11
  • 52. b) Cho ñieåm M (2; 1; 4). Tìm toïa ñoä ñieåm H thuoäc ñöôøng thaúng Δ2 sao cho ñoaïn thaúng MH coù ñoä daøi nhoû nhaát. BAØI GIAÛI: a) (P) chöùa Δ1 vaø // Δ2 1 aΔ = (2, 3, 4); 2aΔ = (1, 1, 2); Δ1 qua M (0, −2, 0) Maët phaúng (P) coù pvt [ ]21 a,a ΔΔ =(2, 0, −1) (P) : 2x – z = 0 b) M (2, 1, 4); H ∈ Δ2; MH min ⇔ MH ⊥ Δ2 C1 : Goïi (Q) laø maët phaúng qua M vaø vuoâng goùc vôùi Δ2. Pt (Q) : x + y + 2z – 11 = 0; {H} = (Q) ∩ Δ2 ⇒ H (2, 3, 3) C2 : MH = (−1 + t, 1 + t, −3 + 2t), vôùi H ∈ Δ2 Do MH . 2aΔ = 0 ⇒ t = 1. Vaäy ñieåm H (2, 3, 3). Ví duï 4: ( ÑH KHOÁI B-2002) Cho hình laäp phöông ABCDA1B1C1D1 coù caïnh baèng a. a) Tính theo a khoaûng caùch giöõa hai ñöôøng thaúng A1B vaø B1D . b) Goïi M,N,P laàn löôït laø caùc trung ñieåm cuûa caùc caïnh BB1, CD,A1D1 .Tính goùc giöõa hai ñöôøng thaúng MP vaø C1N . BAØI GIAÛI: Choïn heä truïc toïa ñoä Axyz sao cho ta coù : A (0, 0, 0); A1 (0, 0, a); B (a, 0, 0); B1 (a, 0, a) C (a, a, 0); C1 (a, a, a); D (0, a, 0); D1 (0, a, a) Suy ra M (a, 0, 2 a ); N ( 2 a , a, 0); P (0, 2 a , a) a) BA1 = (a, 0, −a) DB1 = (−a, a, −a) Goïi (P) laø mp qua B1D vaø (P) // A1B ⇒ (P) coù phaùp vectô n = (1, 2, 1) ⇒ Pt (P) : x + 2y + z – 2a = 0 ⇒ d (A1B, B1D) = d (B, (P)) = 6 a b) MP = (−a, 2 a , 2 a ) . NC1 = (− 2 a , 0, −a) Ta coù : MP . NC1 = 0 ⇒ MP ⊥ C1N. Vaäy goùc giöõa MP vaø C1N laø 900 . Ví duï5 ( ÑH KHOÁI D-2002): Trong khoâng gian vôùi heä toïa ñoä Ñeâcac vuoâng goùc Oxyz, cho maët phaúng (P): 2x – y + 2 = 0 vaø ñöôøng thaúng dm : (m laø tham soá) ⎩ ⎨ ⎧ =++++ =−+−++ 02m4z)1m2(mx 01my)m1(x)1m2( Xaùc ñònh m ñeå ñöôøng thaúng dm song song vôùi maët phaúng (P). BAØI GIAÛI: 1 vectô chæ phöông cuûa (dm) laø : a = (−2m2 + m + 1, −(2m +1)2 , - m(1 – m)) 1 pvt cuûa (P) laø n = (2, −1, 0) ycbt ⇔ a . n = 0 ⇔ −4m2 + 2m + 2 + (4m2 + 4m + 1) = 0 ⇔ 6m + 3 = 0 ⇔ m = 2 1 − 12
  • 53. Ví duï 6 ( ÑH KHOÁI A-2003): Trong khoâng gian vôùi heä toïa ñoä Ñeâcac vuoâng goùc Oxyz cho hình hoäp chöõ nhaät ABCD.A’B’C’D’ coù A truøng vôùi goác toïa ñoä, B(a;0;0), D(0; a; 0), A’(0; 0; b) ( a > 0, b > 0). Goïi M laø trung ñieåm CC’. a. Tính theå tích khoái töù dieän BDA’M theo a vaø b. b. Xaùc ñònh tyû soá a b ñeå hai maët phaúng (A’BD) vaø (MBD) vuoâng goùc vôùi nhau. BAØI GIAÛI: A (0, 0, 0); B (a, 0, 0); C (a, a, 0); D (0, a, 0) A’ (0, 0, b); C’ (a, a, b); M (a, a, b 2 ) a) ; ;= −BD ( a,a,0) = −BA' ( a,0,b) = b BM (0,a, ) 2 ⇒ ⎡ ⎤ =⎣ ⎦ 2 BD,BA' (ab,ab,a ) ⇒ V= ⎡ ⎤ = +⎣ ⎦ 2 21 1 a b 2 BD,BA' .BM (a b ) 6 6 = = 2 2 3a b a b 12 4 (ñvtt) b) (A’BD) coù vectô phaùp tuyeán hay⎡ ⎤ =⎣ ⎦ 2 BD,BA' (ab,ab,a ) =n (b,b,a) (MBD) coù vectô phaùp tuyeán ⎡ ⎤ = −⎣ ⎦ 2ab ab BD,BM ( , , a ) 2 2 hay = −m (b,b, 2a) Ta coù : (A’BD) ⊥ (MBD) ⇔ =m . n 0 ⇔ b2 + b2 – 2a2 = 0 ⇔ a = b (a, b > 0) ⇔ = a 1 b Ví duï 7 ( ÑH KHOÁI B-2003): Trong khoâng gian vôùi heä toïa ñoä Ñeâcac vuoâng goùc Oxyz cho hai ñieåm A(2;0;0), B(0;0;8) vaø ñieåm C sao cho (0;6;0)AC = . Tính khoaûng caùch töø trung ñieåm I cuûa BC ñeán ñöôøng thaúng OA. BAØI GIAÛI: A (2; 0; 0); B (0; 0; 8). = (0; 6; 0) ⇔ ⇔ C (2; 6; 0). I trung ñieåm BC ⇒ I (1; 3; 4)AC =⎧ ⎪ =⎨ ⎪ =⎩ C C C x 2 y z 0 6 Pt tham soá OA : =⎧ ⎪ =⎨ ⎪ =⎩ x t y 0 z 0 (α) qua I ⊥ = (2; 0; 0) : 2(x – 1) = 0 ⇔ x – 1 = 0OA Toïa ñoä {H} = OA ∩ (α) thoûa : ⇔ = = =⎧ ⎨ − =⎩ x t,y 0,z 0 x 1 0 =⎧ ⎪ =⎨ ⎪ =⎩ x 1 y 0 z 0 . Vaäy H (1; 0; 0). d(I, OA) = IH = − + − + −2 2 (1 1) (0 3) (0 4)2 = 5. Ví duï 8 ( ÑH KHOÁI D-2003): Trong khoâng gian vôùi heä toïa ñoä Ñeâcac vuoâng goùc Oxyz cho ñöôøng thaúng 3 2 : 1 0 x ky z dk kx y z + − + = − + + = ⎧ ⎨ ⎩ 0 Tìm k ñeå ñöôøng thaúng dk vuoâng goùc vôùi maët phaúng (P): x – y – 2z + 5 =0 BAØI GIAÛI: 1n = (1, 3k, −1); = (k, −1, 1)2n 13
  • 54. = (3k – 1, −k – 1, −1 – 3k2 )da = (1, −1, 2)Pn − dk ⊥ (P) ⇔ cuøng phöôngda Pn ⇔ − − − − − = = − − 2 3k 1 k 1 1 3k 1 1 2 ⇔ =⎧ ⎪ ⎨ = ∨ = −⎪⎩ k 1 1 k 1 k 3 ⇔ k = 1 Ví duï9 ( ÑH KHOÁI A-2004): Trong khoâng gian vôùi heä toïa ñoä Oxyz cho hình choùp S.ABCD coù ñaùy ABCD laø hình thoi, AC caét BD taïi goác toïa ñoä O. Bieát A(2; 0; 0), B(0; 1; 0), S(0; 0; 2 2 ). Goïi M laø trung ñieåm cuûa caïnh SC. a) Tính goùc vaø khoaûng caùch hai ñöôøng thaúng SA, BM. b) Giaû söû maët phaúng (ABM) caét ñöôøng thaúng SD taïi ñieåm N. Tính theå tích khoái choùp S.ABMN. BAØI GIAÛI: Caùch 1: S M C N H D O B A GT ⇒ SO ⊥ (ABCD); SA = SC = 2 3 a) Ta coù OM // SA ⇒ Goùc (SA, MB) laø OMB OB ⊥ (SAC) ⇒ OB ⊥ OM ΔOBM coù tg OB OMB OM = ⇒ 1 tgOMB 3 = ⇒ OMB =300 Veõ OH ⊥ SA ⇒ OH ⊥ OM vaø OH ⊥ OB ⇒ OH ⊥ (OMB) Vì SA // OM ⇒ SA // (OMB) ⇒ d (SA, MB) = d(H, (OMB)) = OH = 2 6 3 . b) (ABM) ∩ SD = N ⇒ N laø trung ñieåm SD Ta coù: SBMN SBCD V SM SN . V SC SD = 1 4 = ⇒ VSMNB = SBCD SABCD 1 1 V V 4 8 = Töông töï: VSABN = SABCD 1 V 4 Vaäy: VSABMN = VSMNB + VSABN = SABCD 3 V 8 = 3 1 (ñvtt) 1 1 . . AC.BD.SO .4.2.2 2 2 8 3 2 16 = = Caùch 2: a) O laø trung ñieåm BD ⇒ D (0; −1; 0) O laø trung ñieåm AC ⇒ C (−2; 0; 0) M laø trung ñieåm SC ⇒ M ( 1;0; 2)− 14
  • 55. =(2; 0;-SA 2 2 ); BM ( 1; 1; 2)= − − Goïi ϕ laø goùc nhoïn taïo bôûi SA vaø BM cosϕ = − + − + + + 2 0 4 4 8 1 1 2 = 3 2 ⇒ ϕ = 300 Goïi (α) laø mp chöùa SA vaø // BM ⇒ PT (α) : 2x z 2 2 0+ − = Ta coù d(SA, BM) = d(B, α) = 2 6 3 . b) Pt mp(ABM): 2x 2 2y 3z 2 2 0+ + − = Pt tham soá SD: ⎧ = ⎪ = − +⎨ ⎪ =⎩ x 0 y 1 z 2 2t t (t ∈ R). N laø giao ñieåm cuûa SD vaø mp (ABM) ⇒ N 1 (0; ; 2) 2 − BS (0; 1;2 2)= − ; BA (2; 1;0)= − 3 BN (0; ; 2) 2 = − ; BM ( 1; 1; 2)= − − BS,BN (2 2;0;0)⎡ ⎤ =⎣ ⎦ ; BS,BN .BA 4 2⎡ ⎤ =⎣ ⎦ BS.BN .BM 2 2⎡ ⎤ = −⎣ ⎦ VSABMN= VSABN + VSBNM = 1 1 .4 2 .2 2 2 6 6 + = (ñvtt) Ví duï 10 ( ÑH KHOÁI D -2004): Trong khoâng gian vôùi heä toïa ñoä Oxyz cho hình laêng truï ñöùng ABCA1B1C1. Bieát A(a;0;0); B(−a;0;0); C (0; 1; 0); B1(−a; 0; b) a > 0, b > 0. a) Tính khoaûng caùch giöõa 2 ñöôøng thaúng B1C vaø AC1 theo a, b. b) Cho a, b thay ñoåi nhöng luoân thoûa maõn a + b = 4. Tìm a, b ñeå khoaûng caùch giöõa 2 ñöôøng thaúng B1C vaø AC1 lôùn nhaát. BAØI GIAÛI: a) C1 (0; 1; b) Goïi (α) laø maët phaúng chöùa B1C vaø song song vôùi AC1 ;1B C (a;1; b)= − 1C A (a; 1; b)= − − Suy ra: 1 1B C,C A ( 2b;0; 2a)⎡ ⎤ = − −⎣ ⎦ Suy ra ptrình (α): .− + − + − =b(x 0) 0(y 1) a(z 0) 0 ⇔ bx + az = 0. Ta coù: d=d(B1C, AC1)=d(A, α)= 2 2 2 2 ab ab a b a b = + + . b) Caùch 1: Ta coù: d= 2 2 ab ab ab 2ab 2a b ≤ = + a b 4 2 2 2 2 2 + ≤ = = Max d ⇔ d = 2 ⇔ ⇔ a = b = 2 a b a b 4 a 0,b 0 =⎧ ⎪ + =⎨ ⎪ > >⎩ 15
  • 56. Caùch 2: d = ab 16 2ab− , ñaët x = ab, ñk 0 < x ≤ 4. vì x = ab 2 a b 4 2 +⎛ ⎞ ≤ =⎜ ⎟ ⎝ ⎠ Xeùt f(x) = x 16 2x− f’(x) = 3 16 x (16 2x) − − > 0 ∀x ∈ (0; 4] ⇒ d ñaït max khi x = ab = 4 ⇒ a = b = 2 (vì a + b = 4) Ví duï 11 ( ÑH KHOÁI B-2004): Trong khoâng gian vôùi heä toïa ñoä Oxyz cho ñieåm 3 2 : 1 1 4 x t d y t z t = − +⎧ ⎪ = −⎨ ⎪ = − +⎩ A (-4; -2; 4) vaø ñöôøng thaúng Vieát phöông trình ñöôøng thaúng Δ ñi qua ñieåm A, caét vaø vuoâng goùc vôùi ñöôøng thaúng d. BAØI GIAÛI: Caùch 1: A (−4; −2; 4) (d) : ⎪ ⎨ x 3 2t y 1 t z 1 4t = − +⎧ = − ⎪ = − +⎩ Laáy M (−3+2t; 1 – t; −1 + 4t) ∈ (d) ⇒ = (1 + 2t; 3 – t; −5 + 4t)AM Ta coù: AM ⊥ (d) ⇔ (vôùidAM. a 0= da =(2; −1; 4)). ⇔ 2 + 4t – 3 + t – 20 + 16t = 0 ⇔ 21t = 21 ⇔ t = 1. Vaäy ñöôøng thaúng caàn tìm laø ñt AM qua A coù VTCP AM =(3;2;−1) ⇒ phöông trình (Δ) : x+4 y 2 z 4 3 2 + − = = −1 . Caùch 2: Goïi (α) laø mp qua A chöùa d ,Goïi (β) laø mp qua A vaø ⊥ d ⇒ d qua B (−3; 1; −1); = (2; −1; 4)da (α) qua A (−4; −2; 4) (α) coù 1 caëp VTCP : ⇒da (2; 1;4 AB (1;3; 5) ⎧ = −⎪ ⎨ = −⎪⎩ ) ( )n α = (−7; 14; 7) = −7(1; −2; −1) Pt mp (α) : x – 2y – z + 4 = 0 ( ) d ( ) qua A (-4; -2; 4) ( ) (d) n a (2; 1;4)β β β ⎧⎪ ⎨ ⊥ → = = −⎪⎩ Pt (β) : 2x – y + 4z – 10 = 0 Pt (Δ) : x 2y z 4 0 2x y 4z 10 0 − − + =⎧ ⎨ − + − =⎩ Ví duï 12 ( ÑH KHOÁI A-2005): Trong khoâng gian vôùi heä toïa ñoä Oxyz cho ñöôøng thaúng: d : x 1 y 3 z 3 1 2 1 − + − = = − vaø maët phaúng (P) : 2x + y – 2z + 9 = 0 a)Tìm toïa ñoä ñieåm I thuoäc d sao cho khoaûng caùch töø I ñeán maët phaúng (P) baèng 2. b)Tìm toïa ñoä giao ñieåm A cuûa ñöôøng thaúng d vaø maët phaúng (P). Vieát phöông trình tham soá cuûa ñöôøng thaúng Δ naèm trong maët phaúng (P), bieát Δ ñi qua A vaø vuoâng goùc vôùi d. BAØI GIAÛI: a) Phöông trình tham soá cuûa d : ⎧ ⎪ ⎨ ⎪ ⎩ = − = − + = + x 1 t y 3 2t z 3 t (t∈ R) 16
  • 57. I ∈ d ⇔ I (1–t ; –3+2t ; 3+t) Ta coù : d (I, (P)) = 2 ⇔ − − + − − + = + + | 2 2t 3 2t 6 2t 9 | 2 4 1 4 ⇔ Suy ra : I (3 ; -7 ; 1) hay I (-3 ; 5 ; 7). t |1 t | 3 t 4 = −⎡ − = ⇔ ⎢ =⎣ 2 b) Theá phöông trình d vaøo phöông trình (P) ta ñöôïc t = 1. Theá t = 1 vaøo phöông trình d, ta ñöôïc x = 0; y = -1; z = 4 Suy ra A (0; -1 ; 4) Vectô chæ phöông cuûa d : = −a ( 1;2;1) Vectô phaùp tuyeán cuûa (P): = −n (2;1; 2) Suy ra vectô chæ phöông cuûa Δ : = − −[a,n] ( 5; 0; 5) hay (1; 0; 1) Maët khaùc Δ ñi qua A neân phöông trình tham soá cuûa Δ laø : ⎧ ⎪ ⎨ ⎪ ⎩ = = − = + x t' y 1 z 4 t' (t’∈ R) Ví duï 13 ( ÑH KHOÁI B-2005): Trong khoâng gian vôùi heä toïa ñoä Oxyz cho hình laêng truï ñöùng ABC.A1B1C1 vôùi A(0; -3; 0), B(4; 0; 0), C(0; 3; 0), B1(4; 0; 4). a) Tìm toïa ñoä caùc ñænh A1, C1. Vieát phöông trình maët caàu coù taâm laø A vaø tieáp xuùc vôùi maët phaúng (BCC1B1). b) Goïi M laø trung ñieåm cuûa A1B1 . Vieát phöông trình maët phaúng (P) ñi qua hai ñieåm A, M vaø song song vôùi BC1. Maët phaúng (P) caét ñöôøng thaúng A1C1 taïi ñieåm N. Tính ñoä daøi MN. BAØI GIAÛI: a) Hình chieáu cuûa A1 xuoáng mp (Oxy) laø A ⇒ A1(0; -3; 4) Hình chieáu cuûa C1 xuoáng mp (Oxy) laø C ⇒ C1(0; 3; 4) Caëp veùc tô chæ phöông cuûa (BCC1B1) laø : BC ( 4;3;0)= − 1BB (0;0;4)= Suy ra veùc tô phaùp tuyeán cuûa (BCC1B1) laø : = (12; 16; 0) hay = (3; 4; 0)1n BC,BB⎡= ⎣ ⎤ ⎦ m Maët khaùc (BCC1B1) qua B neân coù phöông trình: 3(x – 4) + 4y + 0z = 0 ⇔ 3x + 4y – 12 = 0 Baùn kính maët caàu laø : R = d (A, (BCC1B1)) = 0 12 12 24 59 16 − − = + Suy ra phöông trình maët caàu laø : x2 + (y + 3)2 + z2 = 576 25 b) M laø trung ñieåm cuûa A1B1 ⇒ M (2; 3 2 − ; 4) Mp (P) coù caëp veùc tô chæ phöông 3 AM (2; ;4) 2 = vaø 1BC ( 4;3;4)= − ⇒ veùc tô phaùp tuyeán cuûa mp (P): = = (−6; −24; 12) hay (1; 4; −2)Pn 1AM;BC⎡ ⎣ ⎤ ⎦ Maët khaùc (P) ñi qua A neân coù phöông trình : x + 4(y + 3) – 2z = 0 ⇔ x + 4y – 2z + 12 = 0 A1C1 ñi qua A1 vaø coù veùc tô chæ phöông 1 1A C = (0; 6;0) hay (0; 1; 0) neân coù phöông trình : (t ∈ R) x 0 y 3 z 4 =⎧ ⎪ = − +⎨ ⎪ =⎩ t 17
  • 58. Theá phöông trình A1C1 vaøo phöông trình (P) ta ñöôïc t = 2 Theá t = 2 vaøo phöông trình (A1C1) ta ñöôïc x = 0, y = −1, z = 4 ⇒ N (0; −1; 4) vaø MN = 2 2 23 1 (0 2) ( 1 ) (4 4) 2 2 − + − + + − = 7 Ví duï 14 ( ÑH KHOÁI D-2005): Trong khoâng gian vôùi heä toïa ñoä Oxyz cho hai ñöôøng thaúng : d1 : x 1 y 2 z 1 3 1 2 − + + = = − vaø d2: x y z 2 0 x 3y 12 0 + − − =⎧ ⎨ + − =⎩ a) Chöùng minh raèng d1 vaø d2 song song vôùi nhau. Vieát phöông trình maët phaúng (P) chöùa caû hai ñöôøng thaúng d1 vaø d2. b) Maët phaúng toïa ñoä Oxz caét hai ñöôøng thaúng d1, d2 laàn löôït taïi caùc ñieåm A, B. Tính dieän tích tam giaùc OAB (O laø goác toïa ñoä). aBAØI GIAÛI: a) d1 qua N (1; −2; −1) vaø coù 1 vectô chæ phöông laø =(3; −1; 2) bd2 qua B (12; 0; 10) vaø coù 1 vectô chæ phöông laø =(3; −1; 2) Ta coù : = vaø = (11, 2, 11) khoâng cuøng phöông vôùia b NB a . Vaäy d1 // d2 Mp (P) qua N vaø coù phaùp vectô : =[n a NB, ] = (−15; −11; 17) Phöông trình (P) laø: −15(x–1) – 11(y+2) + 17(z+1) = 0 ⇔ 15x + 11y – 17z – 10 = 0 b) A(−5, 0, −5); B (12, 0, 10) ⇒ = (0, −10, 0)OA,OB⎡ ⎣ ⎤ ⎦ 1 OA,OB 2 ⎡ ⎣⇒ Dieän tích (ΔOAB) = ⎤ ⎦ = 5 (ñvdt). * * * 18
  • 59. CHUYEÂN ÑEÀ 10: HÌNH CAÀU TOÙM TAÉT COÂNG THÖÙC (1) Phöông trình maët caàu 1) Phöông trình maët caàu (S) coù taâm I(a, b, c) baùn kính R laø (x – a)2 + (y – b)2 + (z – c)2 = R2 2) Daïng toång quaùt cuûa phöông trình maët caàu laø x2 + y2 + z2 – 2ax – 2by – 2cz + d = 0 seõ coù taâm I(a, b, c) baùn kính R = 2 2 2 a b c d+ + − neáu ta coù ñieàu kieän a2 + b2 + c2 – d > 0 3) Ñieàu kieän tieáp xuùc giöõa maët phaúng (P) vaø maët caàu (S) coù taâm I baùn kính R laø khoaûng caùch töø I ñeán (P) baèng baùn kính R. Ví duï 1: Laäp phöông trình maët caàu coù taâm I(2, 3, –1) caét ñöôøng thaúng (d) 5 4 3 20 3 4 8 0 x y z x y z − + + =⎧ ⎨ − + − =⎩ 0 taïi hai ñieåm A vaø B sao cho AB = 16 Giaûi Goïi (P) laø maët phaúng qua I vaø vuoâng goùc ñöôøng thaúng (d). Ta coù phöông trình tham soá ñöôøng (d) laø 14 1 2 2 2 x t y t z t = −⎧ ⎪ ⎪ = −⎨ ⎪ = −⎪⎩ 5 Goïi (P) laø maët phaúng qua I(2, 3, –1) vaø vuoâng goùc ñöôøng thaúng (d) neân coù phaùp vectô laø a = 1 1, , 1 2 ⎛ −⎜ ⎝ ⎠ ⎞ ⎟ . Vaäy phöông trình (P) vieát (x – 2) + 1 2 (y – 3) - (z + 1) = 0 ⇔ 2x + y – 2z – 9 = 0 Giao ñieåm K giöõa (d) vaø (P) coù toïa ñoä ( t – 14, 1 2 t – 25 2 , –t ) thoûa phöông trình (P). Vaäy ta coù 1
  • 60. 2(t – 14) + ( 1 2 t – 25 2 ) +2t – 9 = 0 Suy ra t = 11. Vaäy ta coù K (–3, –7, –11). Khoaûng caùch töø I ñeán (d) laø IK = 25 100 100+ + = 15 Do ñoù baùn kính maët caàu laø R = 2 2 4 AB IK + = 225 64+ Neân phöông trình maët caàu vieát laø : (x – 2)2 + (y – 3)2 + (z + 1)2 = 289 Ví duï 2: Laäp phöông trình maët caàu coù taâm thuoäc ñöôøng thaúng (d) 2 4 7 0 4 5 14 0 x y z x y z + − − =⎧ ⎨ + + − =⎩ vaø tieáp xuùc vôùi hai maët phaúng coù phöông trình (P) : x + 2y – 2z – 2 = 0 ; (Q) : x + 2y – 2z + 4 = 0 Giaûi Ta coù (P) // (Q) neân khi goïi A, B laø giao ñieåm cuûa (d) vôùi (P) vaø (Q) thì taâm I maët caàu tieáp xuùc vôùi (P) vaø (Q) phaûi laø trung ñieåm ñoaïn AB vaø baùn kính maët caàu baèng khoaûng caùch töø I ñeán (P). Ta coù toïa ñoä A laø nghieäm cuûa heä A(2, 1, 1) 2 4 7 0 4 5 14 2 2 2 0 x y z x y z x y z + − − =⎧ ⎪ + + − =⎨ ⎪ + − − =⎩ 0 0 ⇒ Ta coù toïa ñoä B laø nghieäm cuûa heä B(–4, 5, 5) 2 4 7 0 4 5 14 2 2 4 0 x y z x y z x y z + − − =⎧ ⎪ + + − =⎨ ⎪ + − + =⎩ ⇒ Vaäy taâm maët caàu laø I(–1, 3, 3) vaø baùn kính R = 1 Neân phöông trình maët caàu vieát thaønh (x + 1)2 + (y – 3)2 + (z – 3)2 = 1. Ví duï 3 ( ÑH KHOÁI D –2004) Trong khoâng gian vôùi heä toïa ñoä Oxyz cho 3 ñieåm A (2; 0; 1); B(1;0;0); C (1; 1; 1) vaø maët phaúng (P): x + y + z – 2 = 0. Vieát phöông trình maët caàu ñi qua 3 ñieåm A, B, C vaø coù taâm thuoäc maët phaúng (P). Giaûi 2
  • 61. Caùch 1: x2 + y2 + z2 + 2ax + 2by + 2cz + d = 0. Maët caàu qua A, B, C vaø coù taâm thuoäc maët phaúng (P) neân ta coù: ⇔ + + = −⎧ ⎪ + = −⎪ ⎨ + + + = −⎪ ⎪ + + = −⎩ 4a 2c d 5 2a d 1 2a 2b 2c d 3 a b c 2 = −⎧ ⎪ =⎪ ⎨ = −⎪ ⎪ =⎩ a 1 b 0 c 1 d 1 ⇔ x2 + y2 + z2 – 2x – 2z + 1 = 0 Caùch 2: Goïi I(x; y; z) laø taâm maët caàu Giaû thieát cho: 2 2 IA IB IC I (P) ⎧ = =⎪ ⎨ ∈⎪⎩ 2 2 ⇔ ⎧ − + + − = − + + ⎪⎪ − + + = − + − + −⎨ ⎪ + + − = ⎪⎩ 2 2 2 2 2 2 2 2 2 2 2 (x 2) y (z 1) (x 1) y z (x 1) y z (x 1) (y 1) (z 1) x y z 2 0 ⇔ ⇔ ⇒ I (1; 0; 1) + − =⎧ ⎪ + =⎨ ⎪ + + − =⎩ 2x 2z 4 0 y z 1 x y z 2 0 x 1 y 0 z 1 =⎧ ⎪ =⎨ ⎪ =⎩ Baùn kính R = IB = 1 Suy ra phöông trình maët caàu: (x – 1)2 + y2 + (z –1)2 =1 Ví duï4 ( Ñeà Döï Tröõ KHOÁI D -2002) Trong khoâng gian vôùi heä toïa ñoä Ñeàcac vuoâng goùc Oxyz cho ñöôøng thaúng d : vaø maët caàu ⎩ ⎨ ⎧ =−−+ =+−− 04z2y2x 01zy2x2 (S) : x2 + y2 + z2 + 4x – 6y + m = 0. Tìm m ñeå ñöôøng thaúng d caét maët caàu (S) taïi hai ñieåm M, N sao cho khoaûng caùch giöõa hai ñieåm ñoù baèng 9. Giaûi Phöông trình maët caàu (S) : (x + 2)2 + (y – 3)2 + z2 = 13 – m ÑK : m < 13 (S) coù taâm I(−2; 3; 0), R = 13 m− . Vì MN = 9 ⇒ HM = HN = 9 2 (IH ⊥ MN) (d) cho x = 0 ⇒ ⇒ 2y z 1 0 2y 2z 4 0 − − + =⎧ ⎨ − − =⎩ y 1 z 1 =⎧ ⎨ = −⎩ ⇒ A(0; 1; −1) (d) coù ⇒ = 3(2; 1; 2) 1 2 n (2, 2, 1) n (1,2, 2) → → ⎡ = − −⎢ ⎢ = −⎢⎣ a → AI ⎯→ = (−2; 2; 1), [ AI ⎯→ , ] = (9; 18; − 18) = 9(1; 2; − 2)a → IH = d(I, d) = ⎯→ → → ⏐ ⏐ + + = = + +⏐ ⏐ [ AI ,a] 9 1 4 4 3 3 4 1 4a . Δ vuoâng IHN ta coù : IM2 = IH2 + HN2 ⇔13 – m = 9 + 81 117 4 4 = 3
  • 62. ⇔ m = 65 4 − . Ví duï 5 ( ÑEÀ DÖÏ TRÖÕ KHOÁI D -2003) Trong khoâng gian vôùi heä toïa ñoä Ñeàcac vuoâng goùc Oxyz cho maët phaúng (P) : 2x + 2y + z – m2 – 3m = 0 (m laø tham soá) vaø maët caàu (S) : (x – 1)2 + (y + 1)2 + (z – 1)2 = 9 Tìm m ñeå maët phaúng (P) tieáp xuùc vôùi maët caàu (S). Vôùi m tìm ñöôïc haõy xaùc ñònh toïa ñoä tieáp ñieåm cuûa maët phaúng (P) vaø maët caàu (S). Giaûi Maët caàu (S) coù taâm I(1; −1; 1), baùn kính R = 3. Maët phaúng P tieáp xuùc vôùi (S) ⇔ d(I: P) = R ⇔ 1443m3m122 2 ++=−−+− ⇔ m2 + 3m – 1 = 9 hay m2 + 3m – 1 = −9 ⇔ m2 + 3m – 10 = 0 hay m2 + 3m + 8 = 0 (VN) ⇔ m = −5 hay m = 2 ⇒ (P) : 2x + 2y + z – 10 = 0 Phöông trình ñöôøng thaúng Δ qua I vaø ⊥ (P) : x 1 2t y 1 2 z 1 t = +⎧ ⎪ Δ = − +⎨ ⎪ = +⎩ t Theá vaøo phöông trình mp (P) ⇒ 2(1 + 2t) + 2(−1 + 2t) + 1 + t – 10 = 0 ⇒ t = 1 ⇒ Tieáp ñieåm M cuûa P vaø (S) laø M(3; 1; 2). Caùch khaùc IM2 = 9 ⇔ 4t2 + 4t2 + t2 = 9 ⇒ t = ± 1 ⇒ M(3; 1; 2) hay M(-1; -3; 0).Vì M∈ P ⇒ M(3; 1; 2) PHAÏM HOÀNG DANH-TRAÀN MINH QUANG –TRAÀN VAÊN TOAØN ( TRUNG TAÂM LUYEÄN THI CLC VÓNH VIEÃN ) 4